You are on page 1of 342
SCHAUM’S OUTLINE OF THEORY AND PROBLEMS or ELECTROMAGNETICS Second Edition JOSEPH A. EDMINISTER Professor Emeritus of Electrica! Engineering ‘The University of Akron SCHAUM’S OUTLINE SERIES McGRAW-HILL New York San Francisco Washington, D.C. Auckland Bogoté Caracas Lisbon London Madrid Mexico City Milan Montreal New Dehli San Juan Singapore Sydney Tokyo Toronto JOSEPH A. EDMINISTER is currently Director of Corporate Relations for the College of Engineering at Cornell University. In 1984 he held an IEEE Congressional Fellowship in the office of Congressman Dennis E. Eckart (D-OH). He received BEE, MSE and JD degrees from the University of Akron. He served as professor of electrical engineering, department head of electrical engineering, assistant dean and acting dean of engineering, all at the University of Akron. He is an attorney in the state of Ohio and a registered patent attorney. He taught electric circuit analysis and electromagnetic theory throughout his academic career. He is a Professor Emeritus of Electrical Engineering from The University of Akron, ‘Appendix B is jointly copyrighted © 1995 by McGraw-Hill, Inc. and MathSoft, Ine. Schaum's Outline of Theory apd Problems of ELECTROMAGNETICS, ‘Copyright © 1993, 1979 by The McGraw-Hill Companies, Inc.All rights reserved. Printed inthe United Sus of America. Except as pernited under the Copyright At of 1976, no par of this publication may be reproduced er disuibuted in any form cr by fay means stored in 8 data base ‘orreuieval syste, without the price writen penisson of the publisher. 567891011 12 13 14 15 16 1718 19 20PRS PRS 98 ISBN O-07-021234 - 1 (Formerly published under ISBN 0-07-018993-5.) Spommoring Editor: David Beckwith Production Supervisor: Al Rihner Eddting Supervisor: Panty Andrews Front Mater Editor: Maureen Walker Libcary of Congress Cataloging-in-Publication Data Edminister, Joseph. ‘Schaum's outline of theory and problems of electromagnetics / by Joseph A. Eéminister —2nd ed. ._em.—{Schaum’s outline series) Includes index. ISBN 0.07-0189935 1. Electromagnetism. 1. Tie. IL. Title: Theory and problems of electromagnetics. I. Series. OCT0.E35 1995, $37°.02'02—4e20 tas. ap McGraw-Hill 82 ADivision of The McGrew Hill Companies Preface ‘The second edition of Schaum’s Outline of Electromagnetics offers three new chapters—in transmission lines, waveguides, and antennas. These have been included to make the book a more powerful tool for students and practitioners of clectromagnetic field theory. I take pleasure here in thanking my colleagues M. L. Kult and K. F. Lee for their contribution of this valuable material. The basic approach of the first edition has been retained: “As in other Schaum's Outlines the emphasis is on how to solve problems. Each chapter consists of an ample set of problems with detailed solutions, and a further set of problems with answers, preceded by a simplified outline of the principles and facts needed to understand the problems and their solutions. Throughout the book the mathematics has been kept as simple as possible, and an abstract approach has been avoided. Concrete examples are liberally used and numerous ‘graphs and sketches are given. I have found in many years of teaching that the solution of most problems begins with a carefully drawn sketch.” Once again it is to my students—my former students—that I wish to dedicate this book. Josern: A. Epsanister VECTOR ANALYSIS 11 Introduction 1.2 Vector Notation 1.3 Vector Algebra 1.4 Coordinate ‘Systems. 1.5 Differential Volume, Surface, and Line Elements ‘COULOMB FORCES AND ELECTRIC FIELD INTENSITY 2.1 Coulomb's Law 2.2 Electric Field Intensity 2.3 Charge Distributions 2.4 Standard Charge Configurations ELECTRIC FLUX AND GAUSS’ LAW. 3.1 Net Charge in a Region 3.2 Electric Flux and Flax Density "33 Gauss? Law 3.4 Relation Between Flux Density and Electric Field Intensity 3.5 Special Gaussian Surfaces DIVERGENCE AND THE DIVERGENCE THEOREM .. 4.1 Divergence 4.2 Divergence in Cartesian Coordinates 4.3 Divergence of D 44 The Del Operator 4.5 The Divergence Theorem a THE ELECTROSTATIC FIELD: WORK, ENERGY, AND POTENTIAL . 5.1 Work Done in Moving « Point Charge 5.2 Conservative Property of the Electrostatic Field 5.3 Electric Potential Between Two Points 5.4 Potential of Point Charge 5.5 Potential of a Charge Distribution 5.6 Gradient 5.7 Rela- tionship Between E and V5.8 Energy in Static Electric Fields Chapter CURRENT, CURRENT DENSITY, AND CONDUCTORS 6:1 Introduction 6:2 Charges in Motion 63 Convention Current’ Density 3 64 Conduction Current Density J 6.5 Conductivity ¢ 6.6 Current J 67 Resistance R 6.8 Current Shect Density K 6.9 Continuity of Current 6.10 Conducior-Dielectric Boundary Conditions CAPACITANCE AND DIELECTRIC MATERIALS 7.1 Polarization B and Relative Permittvity ¢, 7.2 Capacitance 7.3 Multiple- Dielectric Capacitors 7.4 Energy Stored in a Capacitor 7.5 Fixed-Voltage D and E 7.6 Fixed-Charge D and E 7.7 Boundary Conditions at the Interface of ‘Two Dielectrics LAPLACE’S EQUATION 811 Introduction 8.2 Poisson's Equation and Laplace's Equation 8.3 Explicit Forms of Laplace's Equation 84 Uniqueness Theorem 8.5 Mean Value ‘and Maximum Value Theorems 86 Cartesian Solution in One Variable 8.7 Cartesian Product Solution 8.8 Cylindrical Product Solution 8.9 Spherical Product Solution a Chapter 9 CONTENTS AMPERE’S LAW AND THE MAGNETIC FIELD. 9.1 Introduction 9.2 Biot-Savart Law 9.3 Ampere’s Law 9.4 Curt 9.5 Rela- tionship of J and H__9.6 Magnetic Flux Density B 9.7 Vector Magnetic Poten- tial A 9.8 Stokes’ Theorem Chapter 10 FORCES AND TORQUES IN MAGNETIC FIELDS... 10.1 Magnetic Force on Particles 10.2 Electric and Magnetic Fields Com- bined 10.3 Magnetic Force on a Current Element 10.4 Work and Power 10.5 Torque 10.6 Magnetic Moment of a Planar Coit Chapter 11 INDUCTANCE AND MAGNETIC CIRCUITS 11.1 Inductance 11.2 Standard Conductor Configurations andSelfInductance 11.4 Internal inductance 11.5 Mutuallnductance 11.6 Mag- netic Circuits 11.7 The B-H Curve 11.8 Ampare’s Law for Magnetic Circuits 11.9 Cores with AirGaps 11.10 Multiple Coils 11.11 Parallel Magnetic Circuits Chapter 12 DISPLACEMENT CURRENT AND INDUCED EME ..... 1 192 12.1 Displacement Current 12.2 Ratio of J, to Jp 12.3 Faraday’s Law and Lenz's Law 124 Conductors in Motion Through Time-Independent Fields 12.5 Conductors in Motion Through Time-Dependent Fields Chapter 13 MAXWELL’S EQUATIONS AND BOUNDARY CONDITIONS 13.1 Introduction 13.2 Boundary Relations for Magnetic Fields 13.3 Current Sheet at the Boundary 13.4 Summary of Boundary Conditions 13.5 Maxwell's Equations. ELECTROMAGNETIC WAVES: 14.1 Introduction 14.2 Wave Equations 14.3 Solutions in Cartesian Coord- {nates 14.4 Solutions for Partially Conducting Media 14.5 Solutions for Perfect Dielectrics 14.6 Solutions for Good Conductors; Skin Depth 14.7 Interface Conditions at Normal Incidence 14.8 Oblique Incidence and Snell's Laws 14.9 Perpendicular Polarization 14.10 Parallel Polarization 14.11 Standing Waves 14.12 Power and the Poynting Vector ‘TRANSMISSION LINES 15.1 Introduction 15.2 Distributed Parameters tages and Currents 15.4 Sinusoidal Steady-State Excitation 15.5 The Smith Chart 15.6 Impedance Matching 15.7 Single-Stub Matching 15.8 Double- Stub Matching 15.9 Impedance Measurement 15.10 Transients in Lossless Lines Captor 16 WAVEGUIDES ---.-eeseee 16.1 Introduction 16.2 Transverse and Axial Fields 16.3 TE and TM Modes; ‘Wave Impedances 16.4 Determination of the Axial Fields 16.5 Mode Cutoff Frequencies 166 Dominant Mode 16.7 Power Transmitted in a Lossless Waveguide 16.8 Power Dissipation in a Lossy Waveguide Chapter 17 CONTENTS ANTENNAS 17.1 Introduction 17.2 Current Source and the E and H Fields 17.3 Electric (Hertzian) Dipole Antenna 17.4 Antenna Parameters 17.5 Small Circular- Loop Antenna 17.6 Finite-Length Dipole 17.7 Monopole Antenna 17.8 Self- and Mutual Impedances 17.9 The Receiving Antenna 17.10 Linear Arrays 17.11 Reflectors Chapter 1 Vector Analysis L1_ INTRODUCTION Vectors are introduced in physics and mathematics courses, primarily in the cartesian coordinate system. Although cylindrical coordinates may be found in calculus texts, the spherical coordinate system is seldom presented. All three coordinate systems must be used in electromagnetics. AS the notation, both for the vectors and the coordinate systems, differs from one text to another, a thorough understanding of the notation employed herein is essential for setting up the problems and obtaining solutions. 1.2, VECTOR NOTATION In order to distinguish vectors (quantities having magnitude and direction) from scalars (quantities having magnitude only) the vectors are denoted by boldface symbols. A unit vector, one Of absolute value (or magnitude or length) 1, will in this book always be indicated by a boldface, lowercase a. The unit vector in the direction of a vector A is determined by dividing A by its absolute val Ay A lal A By use of the unit vectors a,, a, a, along the x, y, and z axes of a cartesian coordinate system, an arbitrary vector can be written in component form: A=A,9, +A, +48, In terms of components, the absolute value of a vector is defined by IAl=A = VATS Ag+ AT 13 VECTOR ALGEBRA 1. Vectors may be added and subtracted. ALB=(A,9,+4,8, +4,8,) + (Bet, + By, + Ba.) = (A. +B,)a, + (A, * B,)a, + (A,@ B,)a, 2. The associative, distributive, and commutative laws apply. A+(B+O=(A+B)+C K(A+B)=KA+KB (Ki +h )A A+B=B+A 3. The dot product of two vectors is, by definition, AsB=ABoos@ (read “A dot B") where @ is the smaller angle between A and B. In Example 1 it is shown that A-B=A,B,+4,B,+4,B, which gives, in particular, |A|=VA-A. 1 A+ A 2 VECTOR ANALYSIS [cHap. 1 EXAMPLE 1. The dot product obeys the distributive and scalar multiplication laws BHO=AsB+A+C A-KB=K(A-B) This being the case, A-B=(A,a, + 4,8, + A.t,)- (5,8, + B,8, + B.8,) = A.B.,+ 8.) + A,B, +8) + A,B (0,8) + A,Bi(Qea,) +--+ AB,(Q 2) However, 0,*8,=8,:8,=8,28,=1 because the cos 6 in the dot product is unity when the angle is zero. And when @= 50", 086 is zer0; hence all other dot products of the unit vectors are zero. Thus AsB=A,B,+4,B, + A.B, 4, The cross product of two vectors is, by definition, AXB=(ABsin Oa, (read “A cross B") where 6 is the smaller angle between A and B, and a, is a unit vector normal to the plane determined by A and B when they are drawn from a common point. There are two normals to the plane, so further specification is needed. The normal selected is the one in the direction of advance of a right-hand screw when A is turned toward B (Fig. 1-1). Because of this direction requirement, the commutative law does not apply to the cross product; instead, AXB=-BXA Expanding the cross product in component form, AXB=(A,9, + Aa, + 4.4.) (Bye, + Ba, + Bya,) = (A,B, ~ A.B, )a, + (A,B, ~ AxB,)a, + (A,B, — A,By)a, which is conveniently expressed as a determinant: cap. 1) VECTOR ANALYSIS 3 EXAMPLE 2. Given A=2a,+48,—38, and B=a,—a, find A: AsB=(2)(1) + 4-1) +(-31) sau AxB=|2 4-3] =—30,-35,-60, 1-10 1.4 COORDINATE SYSTEMS ‘A problem which has cylindrical or spherical symmetry could be expressed and solved in the familiar cartesian coordinate system. However, the solution would fail to show the symmetry and in most cases would be needlessly complex. Therefore, throughout this book, in addition to the cartesian coordinate system, the circular cylindrical and the spherical coordinate systems will be used. All three will be examined together in order to illustrate the similarities and the differences. ‘A point P is described by three coordinates, in cartesian (x, y, z), in circular cylindrical (r, ¢, 2), and in spherical (r, 0, $), as shown in Fig. 1-2. The order of specifying the coordinates is important and should be carefully followed. ‘The angle ¢ is the same angle in both the cylindrical and spherical systems. But, in the order of the coordinates, @ appears in the second position in cylindrical, (r, g, z), and the third position in spherical, (r, 8, ¢). The same symbol, r, is used in both cylindrical and spherical for two quite different things. In cylindrical coordinates r measures the distance from the z axis in a plane normal to the z axis, while in the spherical system r measures the distance from the origin to the point. It should be clear from the context of the problem which ris intended. 8 2.0.8) () Cartesian (©) Cylindrical Fig. 12 ‘A point is also defined by the intersection of three orthogonal surfaces, as shown in Fig. 1-3. In cartesian coordinates the surfaces are the infinite planes x=const, y=const., and z const. In cylindrical coordinates, z=const. is the same infinite plane as in cartesian; const. is a half plane with its edge along the z axis; r=const. is a right circular cylinder. | These three surfaces are orthogonal and their intersection locates point P. In spherical coordinates, = const. is the same half plane as in cylindrical; r=const. is a sphere with its center at the origin; @=const. is a right circular cone whose axis is the z axis and whose vertex is at the in. Note that is limited to the range 0= 65. Figure 1-4 shows the three unit vectors at point P. In the cartesian system the unit vectors have fixed directions, independent of the location of P. This is not true for the other two systems (except in the case of a,). Each unit vector is normal to its coordinate surface and is in the direction in which the coordinate increases. Notice that all these systems are right-handed: a Xa, aXe, aXe, % 4 VECTOR ANALYSIS. (CHAP. 1 re const = const (0) Cartesian (©) Cyindricat (© Spherical Fig 13 “The component forms of a vector in the three systems are A=, +42, +A,a, (cartesian) A= Aa, + Agag+A.e, (cylindrical) A=AB, +Aoto+Agae (spherical) It should be noted that the components A,, A,, Ag, et are functions of the coordinates in that particular system, 5 % 4 } PY —» , Jt (e) Cartesian (6) Cyindsical (© Spherical . are not generally constants but more often Fig. 14 1.5 DIFFERENTIAL VOLUME, SURFACE, AND LINE ELEMENTS ‘There are relatively few problems in electromagnetics that can be solved without some sort of integration—along a curve, over a surface, or throughout @ volume. Hence the corresponding differential elements must be clearly understood. When the coordinates of point P are expanded to (x + dr, y + dy, z-+dz) or (r-+dr, p+ dg, 2+dz) or (r+ dr, 6+ d0, @ + do), a differential volume du is formed. To the first order in infinitesimal quantities the differential volume is, in all three coordinate systems, a rectangular box. The value of du in each system is given in Fig. 1-5. CHAP. 1] ‘VECTOR ANALYSIS 5 dy= dx dvds d= rdrdeds Sind drd0 db (e) Cartesian () Cylindrical (©) Spherical Fig. 1S From Fig. 1-5 may also be read the areas of the surface elements that bound the differential volume. For instance, in spherical coordinates, the differential suiface element perpendicular to a, is dS = (rd0)(r sin @dg) =?" sin 0d0dp ‘The differential line element, dé is the diagonal through P. Thus dO = dx? +dy?+ dz? (cartesian) dO=dP +P dg? dz? (cylindrical) d@=dP + Pde? +P sin? dg? (spherical) Solved Problems 1.1. Show that the vector directed from M(x1, ys, 2:) to N(Q&z, yz» 22) in Fig. 1-6 is given by Mie + (92 — Yad + Moxyed3022) Moxy ¥1.21) Fig 16 ‘The coordinates of M and N are used to write the two position vectors A and B in Fig. 1-6. Anza ty tam, Bane, ty8, +28, ‘Then B-A=(@:-x)a,+0:- We, +(e — 208, 6 VECTOR ANALYSIS ICHAP. 1 1.2. Find the vector A directed from (2, ~4, 1) to (0, ~2,0) in-eartesian coordinates and find the uni¢ veetor along A. a nete-matt-2-Coh+0-Due tat IAP = (-2) + (2+ (-17 =9 ~AL 2,42, -1 eenigin ah hgh ah 1.3. Find the distance between (5, 321/2, 0) and (5, 2/2, 10) i cylindrical coordinates. {(5.39/2,0) Fig. 47 ‘Then B-A=10a,+10a, and the required distance between the points is {B- Al = 10v2 ‘The cylindrical coordinates of the points cannot be used to obtain a vector between the points in the same manner as was employed in Problem 1.1 in cartesian coordinates. 14, Show that A=4a,-2a,—a, and B=0,+4a,—4a, are perpendicular. ‘Since the dot product contains cos @, a dot product of zero from any two nonzero vectors implies that 0= 90 A = (4)(1) + (294) + (-1)(-4) =0 15. Given A=2a,+4a, and B=6a,—4a,, find the smaller angle between them using (a) ig the cross product, (6) the dot product. ae 8 @ AxB=|2 4 0 | =—t60, +88, +128, 0 6 ~4 1Al= VOy+@)'+ OF =4.47 [Bh = VOF + OF + (= 4y = 7.21 IAXBI= VITO + OF Oa = 21.54 ‘Then, since [A XB] =|Al |B|sin 8, 21.54 sn 0 Gana) = 0.668 or OAL. CHAP. 1] VECTOR ANALYSIS 7 ® A-B= (2)(0) + (4)(6) + OM -4) = 24 __24 (ALIBI @-47)07-21) 0s 6: 07S or 6= 41" (ya, +2xa,, find the vector at (2,2,1) and its projection on B, 16. Given ‘cig where B=Sa,—a,+28,. ry (2,2, 1) = (2- Na, + (2)(2)a, aatds, {As indicated in Fig. 1-8, the projection of one vector on a second vector is obtained by expressing the tunit vector in the direction of the second vector and taking the dot product ‘Thus, at 2.2.1), Given A=a,+s,, B=9,+2a,, and” C=2a,+2,, find (AXB)X€ and compare it with Ax (Bx). aaa Axpe|i 1 0|=2-25,-—, 102 nae Then (axe)xe~]2 -2 -1]=-25, +40, 021 A similar calculation gives AX(BXC)=2a,~2s, +3e,. Thus the parentheses that indicate ‘hich cross product is to be taken fist are essential in the vector triple product and C of Problem 1.7, find A+BXC and compare it with Ax 1.8. Using the vectors A, B-C. From Problem 1.7, BXC= 4a, —a,+2a,. Then A-BXC= (1-4) + (1-1) +2) ‘Also from Problem 1.7, AXB=2a,—2a,—a,. Then AXB-C= (240) + (-2)2)+ (1) =-5 Parentheses are not needed in the scalar triple product since it has meaning only when the cross product is taken first. In general, it can be shown that ALA A B, BB, AGG A-BXC= 8 VECTOR ANALYSIS fcHar. 1 AAs long as the vectors appear in the same cyclic order the result is the same. ‘The scalar triple products ‘ot in this cyclic order have a change in sign. 1.9, Express the unit vector which points from z= on the z axis toward (r, @, 0) in cylindrical coordinates. See Fig. 1-9. AO Fig. 19 ‘The vector R is the difference of two vectors: R=ra—he, raha, RI VP ‘The angle ¢ does not appear explicitly in these expressions. Nevertheless, both Rand a, vary with through »,. a 1.10. Express the unit vector which is directed toward the origin from an arbitrary point on the plane z=~5, as shown in Fig. 1-10. Fig. 1-10 Since the problem is in cartesian coordinates, the two-point formula of Problem 1.1 applies. R= -x0, ys, +50, 1.11. Use the spherical coordinate system to find the area of the strip a= @10cm), E=6a,V/m; and for z<10cm, E=~6a, V/m. 6v/m Solved Problems |. Two point charges, Q)=SOuC and Q,=10uC, are located at (—1,1,—3)m and ziz (3, 1,0) m, respectively (Fig. 2-8). Find the force on Q,. ry R= 2.02 y aneoRa 1-650 10°)(10) a 2.1.0) 8, 1,13) Fig. 28 CHAP. 2] COULOMB FORCES AND ELECTRIC FIELD INTENSITY 19 ‘The force has a magnitude of 0.18N and a direction given by the unit vector -0.88, -0.6a,. In ‘component form, F,=0.144a, — 0.1088, N 2.2, Refer to Fig. 2-9. Find the force on a 100 uC charge at (0,0,3) m if four like charges of a 20 nC are located on the x and y axes at 4m. 1004 0,3) (0,4,0) Fig. 2-9 Consider the force due to the charge at y=4, 00-0109 (45,438) ‘4x0 6G S ‘The y component will be canceled by the charge at_y=—4, Similarly, the x components due to the ‘other two charges will cancel. Hence v-4(2)n)<17—.N 23. Refer to Fig. 2-10. Point charge Q1=300uC, located at (1, force —3)m, experiences a F, = 8, — 88, +48, 2 2 @-3,-2) at ig. 2-10 due to point charge Qz at (3, ~3, -2)m. Determine Qs. Ry = 2a, +24, Note that, because 20 (COULOMB FORCES AND ELECTRIC FIELD INTENSITY {cnar. 2 the given force is along R,, (see Problem 1.21), as it must be. aneR?* = ba, $a, 1D, (~2a, 24 ay Ba, 88, #48 e190 “Teo 7) Solving, Q2=—40 uC. 2.4, Find the force on a point charge of 50 uC at (0,0, 5)m due to a charge of S00: nC that is a uniformly distributed over the circular disk r=Sm, z=Om (see Fig. 2-11). (0,0,5) ‘The charge density In cylindrical coordinates, $0 Tae TD) (VTE [Before integrating, note that the radial components wll cancel and that a, is constant. Hence f° fox 1o-9102 10-95% dre rf f 4n(10- "(36K + 25) -1 7 aR) 1600 N 2.5. Repeat Problem 2.4 for a disk of radius 2m. Z Reducing the radius has two effects: the charge density is increased by a factor (5)? or 85 a hile the integral over r becomes [eign nots inseator [ati e= 0.0m ‘The resulting force is 0143; F = 6.25)(5EGsz) (16.564, N) = 25.270, N CHAP. 2) COULOMB FORCES AND ELECTRIC FIELD INTENSITY a 2.6. Find the expression for the electric field at P due to a point charge Q at (x1, Yu. 7). Repeat with the charge placed at the origin. AAs shown in Fig. 212, ReGe—a)at(y—wa He -n “Grea + +20 When the charge is atthe origin, O x tyy ten, ane (ety rey bb this expression fails to show the symmetry of the field. In spherical coordinates with Q at the origin, 2 Be and now the symmetry is apparent. ‘2.7, Find E at the origin due to a point charge of 64.4nC located at (—4,3,2)m in cartesian coordinates. ‘The electric field intensity due 0/8 point charge Q at the origin in spherical coordinates is 2 barrie In this problem the distance is Vm and the vector from the charge ‘© the origin, where Bis to be Pietra Sonar nach ) = v0 2=35=2) vim ax (saute a, — 34-28) ax(QO*736M\29), ve 28, Find E at (0,0,5)m due to Q,=0.354C at (0,4,0)m and Q2=-0.55uC at 3,0, 0) m (see Fig. 2-13). R= —4a, +58, Ri=—3a, +50, asso (=e 458) FRO 7s6mye \ Var = 48.08, +60.00, Vim 2 COULOMB FORCES AND ELECTRIC FIELD INTENSITY [cHap. 2 Be gR10- 736m) = 74.99, = 1249, Vim and E=E, +E," 74.98, — 48.08, ~ 64.98, Vim 29. Charge is distributed uniformly slong an infinite straight line with constant density ‘Pc. Develop the expression for E at the general point P. Cylindrical coordinates will be used, with the line charge as the z axis (see Fig. 2-14). At P. Since for every dQ ats there is another charge dQ at —z, the z components cancel. Then pur de = maar be ‘anes. lap oneer™ 2.10, On the line described by x=2m, y=—4m_ there is a uniform charge distribution of density pe=20nC/m. Determine the electric field E at (-2, -1,4)m. With some modification for cartesian coordinates the expression obtained in Problem 2.9 can be used with this uniform line charge. Since the line is parallel to a, the field has no 2 CHAP. 2) COULOMB FORCES AND ELECTRIC FIELD INTENSITY 2B ‘component. Referring to Fig. 2-15, Ra -te, +35, 20x 10% /—4a, +34) _ and en IEA) neg 5. Vin Mg. 215 2413. As shown in Fig. 2-16, two uniform line charges of density pe=4nC/m lie in the a *=0 plane at y= 44m. Find E at (4,0, 10) m. Rg. 216 ‘The line charges are both parallel to a,; their fields are radial and parallel to the xy plane. For either line charge the magnitude of the field at P would be =f Bim Bas be = Evi va ‘The field due to both line charges is, by superposition, 18 1 =2( ems = 8, Vim 2.12. Develop an expression for E due to charge uniformly distributed over an infinite plane with 2 density p,. a COULOMB FORCES AND ELECTRIC FIELD INTENSITY IcHAP. 2 ‘The cylindrical coordinate system will be used, with the charge in the z=0 plane as shown in Fig 217, pride dg +z.) Wen eft ae) Fig. 247 Symmetry sbout the axis results in cancellation ofthe radial components. predrdg Ee Clete a= fe ares ‘This result is for points above the xy plane. Below the xy plane the unit vector changes to ~a,. The ‘generalized form may be writen using a,, the unit normal vector: enfes ‘The electric field is everywhere normal to the plane of the charge and its magnitude is independent of the distance from the plane, 2.13. As shown in Fig. 2-18, the plane y=3m contains a uniform charge distribution of density p, =(10~*/62) C/m*. Determine E at all points. ¢ ie G30 wo Fory>3m, and for y<3m, B= -300, Vim CHAP. 2] COULOMB FORCES AND ELECTRIC FIELD INTENSITY 25 2.14. ‘Two infinite uniform sheets of charge, each with density p,, are located at x=+1 (Fig. 2.19). Determine E in all regions. Mig. 219 ‘Only parts of the two sheets of charge are shown in Fig. 2-19. Both sheets result in KE fields that are directed along x, independent of the distance. Then (lem, <1 +E} 0 l1 245. Repeat Problem 2.14 with pon x=—1 and —p,on x=1. 0 x<-l E,+E.=4 (p/eos, 9 -1, E-»(p,/2¢,)a,, the field due to a uniform plane sheet. Charge lies on the circular disk r4m, Y2apL = Darl) p="4, (im) CHAP. 3] ELECTRIC FLUX AND GAUSS' LAW a 3.16. ‘The volume in spherical coordinates described by ra? For a gaussian surface such as 2 in Fig. 317, Qu=$D- as $arp= Disa) and poe Fer polos cus te arg dt, Sacip= DG) whene D=Ba, ro If a point charge Q=na’p is placed at the origin, the D field for r>a will be the ‘same. This point charge is the same as the total charge contained in the volume. A parallel-plate capacitor has a surface charge on the lower side of the upper plate of +p, (Cim*). The upper surface of the lower plate contains —p, (C/m*). Neglect fringing and use Gauss’ law to find D and E in the region between the plates. Al fux leaving the positive charge on the upper plate terminates on the equal negative charge on the lower plate. The statement neglect fringing insures that all lux is normal to the plates. For the special gaussian surface shown in Fig. 3-18, One= [D-as+[ p-as+ | Deas =0+[ paseo or pA=D ds=DA where A is the area. Consequently, D=pa,(Cim) and E=2a, (Vim) Both are directed from the positive to the negative plate. 3.19. 3.20, 323, 304, 328. [Lay Mig. 3418 Supplementary Problems Find the net charge enclosed in a cube 2m on an edge, parallel to the axes and centered at the origin, if the charge density is p=S0stc0s(Zy) (acim?) ‘Ans, 849 uC Find the charge enclosed in the volume 1a? Ans. Q, 0" ‘A uniform tine charge with p,=34C/m lies along the x axis. What flux crosses a spherical surface ‘centered at the origin with r=3m? Ans. 18 uC {fa point charge Q is at the origin, find an expression for the flux which crosses the portion of a sphere, cenueastteoign daciteaty w=geh am PSQ quar. 3.26, 3.28. 3M. 335. 336. 337. a 31 ELECTRIC FLUX AND GAUSS’ LAW 45 ‘A point charge of O (C) is at the center ofa spherical coordinate system. Find the flux Wf which crosses, ‘an area of 4m on a concentric spherical shell of radius 3m. Ans. Q/9 (C) ‘An area of 40.2 m? on the surface of a spherical shell of radius 4m is crossed by 10 C of flux in an inwatd direction. What point charge at the origin is indicated? Ans. —50 nC ‘A uniform line charge ¢ lies along the x axis. What percent of the flux from the line crosses the strip ofthe y=6 plane having -152=1? Ans. 5.26% ‘A point charge, Q=3nC, jis located at the origin of a cartesian coordinate system. What flux W crosses the portion of the 2=2m plane for which —4a. Obtain D in all regions. Ans, Wasa'p={[FO 7Se Given that D=S00e"a, (uC/m’), find the flux W crossing surfaces of area 1 m* normal to the x axis and located at x=1m, x=5m, and x= 10m. Ans 452 pC, M3 yC, 184 uC Given that D=Sr°a, + 10za, (C/m’), find the net outward fux crossing the surface of a cube 2m nan edge centered atthe origin. The edges ofthe cube are parallel to the axes. Ans. 80C Given that D=30e""4,~ 25, (Cim') in cylindrical coordinates, find the outward flux crossing the right circular eylinder described 0, and 2=Sb(m). Ans. 1296*(C) De tromen, 288s, i thal carn, fd theft cring he perio of te £90. pede by 1 SORES Sante TS cali Ans, $$ In cyclindrical coordinates, the disk r a). ans. (0) 229, (ey 2, wnere = [" [ ote. eorardp A point charge, Q=2000pC, is at the origin of spherical coordinates. A concentric spherical distribution of charge at r=Im has a charge dersity p,=40x pC/m’. What surface charge density on a concentric shell at_ r=2m would result in D=0 for r>2m? Ars. 2 pCa Be & g ELECTRIC FLUX AND GAUSS’ LAW [cHap. 3 Given a charge distribution with density =r (C/m?) in spherical coordinates, use Gauss’ law t0 find D. Ans. (Sr/4)a, (C/m*) ‘A uniform charge density of 2C/m? exists in the volume 2x 54m (cartesian coordinates), Use Gauss’ law to find D in all regions. Ans. ~2a, C/m, 2(x —3)a, (C/m, 2a, C/m? ‘Use Gauss’ law to find D and E in the region between the concentric conductors of a cylindrical capacitor. The inner cylinder is of radius a. Neglect fringing. Ans. p.(a/?), pa(a/éor) ‘A conductor of substantial thickness has a surface charge of density p,. Assuming that Y=0 within the conductor, show that D=+p, just outside the conductor, by constructing a small special ‘gaussian surface. Chapter 4 Divergence and the Divergence Theorem 4.1 DIVERGENCE ‘There are two main indicators of the manner in which a vector field changes from point to point throughout space. The first of these is divergence, which will be examined here. It is a scalar and bears a similarity to the derivative of a function. The second is curl, a vector which will be ‘examined when magnetic fields are discussed in Chapter 9. ‘When the divergence of a vector field is nonzero, that region is said to contain sources or sinks, ‘sources when the divergence is positive, sinks when negative. In static electric fields there is a correspondence between positive divergence, sources, and positive electric charge Q. Electric flux W by definition originates on positive charge. Thus, a region which contains positive charges contains the sources of W. The divergence of the electric flux density D will be positive in this region. A similar correspondence exists between negative divergence, sinks, and negative electric charge. Divergence of the vector field A at the point P is defined by re AU Here the integration is over the surface of an infinitesimal volume Av that shrinks to point P. 4.2 DIVERGENCE IN CARTESIAN COORDINATES The divergence can be expressed for any vector field in any coordinate system. For the development in cartesian coordinates a cube is selected with edges Ax, Ay, and Az parallel to the x, y, and z axes, as shown in Fig. 4-1. Then the vector field A is defined at P, the corner of the cube ‘with the lowest values of the coordinates x, y, and z. A=A,a, +4 +A.a, Fig 41 In order to express $A dS for the cube, all six faces must be covered. On each face, the direction of dS is outward. Since the faces are normal to the three axes, only one component of A will cross any two parallel faces. 47 48 DIVERGENCE AND THE DIVERGENCE THEOREM [cHae. 4 In Fig. 42 the cube is turned such that face 1 is in full view; the x components of A over the faces to the left and right of 7 are indicated. Since the faces are small, [ A+dS~—A,(x) Ay Az f A-dS~A,(x-+ Ax) dy Az a ae aA - A.0)+ Aas] Ay Az Ag@) Ante #0) a so that the total for these two faces is Arar ay a2 ‘The same procedure is applied to the remaining two pairs of faces and the results combined. OA, | OA, | aA, 4 Pr , BAL 2 (4+ yt ge) Bray Dividing by Ax Ay Az=Av and letting Av->0, one obtains 3A, | aA, BA, iv A = Seg Sy. ax ay ae The same approach may be used in cylindrical (Problem 4.1) and in spherical coordinates. 1a 2oays Lads, OA ray faces (cartesian) (cylindrical) 1 aay rsin 6 3@ (spherical) EXAMPLE 1. Given he vector fel) A=Se (sin), fd dv Aat 2 and div Al. EXAMPLE 2._In cylindrical coordinates a vecor field is given by A=rsin ga, +r*cos oa, +2re-%a,. Find iv A at (I, 2/2, 0). 1a 1a any = -» div A232 (sin +42 re 9)+ Ze (Qre) =2sin@ —rsin o ~ 10re and div Alunano= sin in —10(3)¢ 2 2 cHap. 4] DIVERGENCE AND THE DIVERGENCE THEOREM 0 EXAMPLE 3. In spherical coordinates a vector field is given by A=(S/r?)sin Oa, +rcot Gay + sin @ 008 ay. Find div A. sin 0)+ 2 3 sin cot 0) +15 3 (sin 8 08 9) = ‘in 636 sin 036 sing 1 diva=3, 4.3, DIVERGENCE OF D From Gauss’ law (Section 3.3), fv-as ae ae ae In the limit, fv-as = diva tin See ae aD fim ay oP This important result is one of Maxwell’s equations for static fields: divD=p and divE=' if ¢ is constant throughout the region under examination (if not, diveE=p). Thus both E and D fields will have divergence of zero in any isotropic charge-free region. EXAMPLE 4. In spherical coordinates the region ra contains a uniform charge density p, while for r>a the charge density is zero. From Problem 2.54, E=E,,, where E.=(pr/3e) for rsa and E,=(pa'[3er') for r>a, ‘Then, for ra, 44 THE DEL OPERATOR ‘Vector analysis has its own shorthand, which the reader must note with care. At this point a vector operator, symbolized ¥, is defined in cartesian coordinaies by a), Hd, ©), oy tae In the calculus a differential operator D is sometimes used to represent d/dx. The symbols Vand Jf are also operators; standing alone, without any indication of what they are to operate on, they look strange. And so V, standing alone, simply suggests the taking of certain partial derivatives, each followed by a unit vector. However, when V is dotted with a vector A, the result is the divergence of A. BAe, By ax * 8y a.,8 28 vea=(Sa+ Site Hereafter, the divergence of a vector field will be written V- A. 30 DIVERGENCE AND THE DIVERGENCE THEOREM IcHap. 4 Warning! The del operator is defined only in cartesian coordinates. When V- A is written for the divergence of A in other coordinate systems, it does not mean that a del operator can be defined for these systems. For example, the divergence in cylindrical coordinates will be written as 212 44 tthe Ae WARE ANT ag * Ge (see Section 4.2). This does not imply that ule OA Sorat et a in cylindrical coordinates. In fact, the exreion would give false results when used in ‘WV (the gradient, Chapter 5) or VX A (the curl, Chapter 9). 4.5 THE DIVERGENCE THEOREM Gauss’ law states that the closed surface integral of ID dS is equal to the charge enclosed. If the charge density function p is known throughout the volume, then the charge enclosed may be obtained from an integration of p throughout the volume. Thus, $0-ds~ | pdv~ Que But p=V-D, andso fo-s= [0-4 This is the divergence theorem, also known as Gauss" divergence theorem. It is a three-dimensional analog of Green's theorem for the plane. While it was arrived at from known relationships among D, Q, and p, the theorem is applicable to any sufficiently regular vector field. livergence theorem fara [aa Cf course, the volume w is that which is enclosed by the surface S. EXAMPLE 5. The region r0, (cos xa,—sinxe,), find V- A. tone deere t 2 t-erains) eosin) seins) =0 cHap. 4) DIVERGENCE AND THE DIVERGENCE THEOREM 33 45. Given A=x7e,+yza,+xya,, find V-A. 3ay, 2 oy, 2 VAR ZONE 02) +2 tate oe 46. Given A=(7+y7"%a,, find VA at (2,2, 0). 2 VeAm-3@? + @x) and V-Alaay = -8.84% 107 4.7, Given A=rsin ga, +27.c0s ga, +22%,, find V-A. 212, 12 2 Ven SU? in 9) +35 (roo) +5 (27) =2sing—2sin +42 =42 48. Given A=10sin? pa, +ra, +[(2"/r) cos? ga, find V+A at (2, 6,5). _1Osin? 6 +22 cos? vA and V+Alagn=5 49. Given A=(5/r")a, + (10/sin 6)a,—1°o sin a,, find V- A. 1a a , Fan 5360+ ran 05g P08) —F VARS 56+ 4.10, Given A=Ssin Gay +Ssinga,, find V- A at (0.5, 2/4, 1/4). wha 2 (ssint 0+ 2 (5 sin @) = 10.84 5 EX a SA in Ora ts =e ES and V Alasnany = 24.14 4.11. Given that D= poze, in the region -1=z<1 in cartesian coordinates and D= (Gozlizi)a, elsewhere, find the charge density v-D=p For -1=7=1, a Zloor) = Po and for z<-1 or 2>1, ° ‘The charge distribution is shown in Fig. 4-5, 4 DIVERGENCE AND THE DIVERGENCE THEOREM [cHar. 4 4.12, Given that _D=(10r°/4)a,(C/m®) in the region O03m, 1a p= 73 (e10/4)-0 4.13, Given that p= 2.(.-c0s30, in spherical coordinates, find the charge density 1a 22 32 2 a-cos3n]-Bcinsr 4.14. In the region 01m, D=(-4x 10~/)a, (C/m?), in spherical coordinates. Find the charge density in both regions. For 01m, 1a . p=3S(-4x10)=0 418. In the region r=2, D=(5r7/4)a, and for r>2, D=(20/r%)a,, in spherical coordinates. Find the charge density. For r=2, (Sr'f8) = 5r “Far and for r>2, 1a p= 35 e0=0 4.16, Given that D=(10x"/3)a, (C/m?), evaluate both sides of the divergence theorem for the volume of a cube, 2m on an edge, centered at the origin and with edges parallel to the axes. fv-as~[ (w-pyav ‘Since D has only an x component, D+ dS is zero on all but the faces at x=1m and x=—1m (see * 10(1), * 10(—1) Fig 44), goa ff 2 3 1a, -dy dz (-,) CHAP. 4) DIVERGENCE AND THE DIVERGENCE THEOREM 5 Fig. 46 Now forthe right side of the divergence theorem. Since V+D= 10s’, Loma ff ‘fp . op * J c10e) ae aya = f Jf [oZ] are Ze 4.17. Given that A=30e~’a,—2za, in cylindrical coordinates, evaluate both sides of the Pe divergence theorem for the volume enclosed by r=2, z=0, and z=5 (Fig. 47). faces=[o-ara Mg 47 Itis noted that A,=0 for 2=0 and hence A+ dS is zero over that part of the surface. faras=[ [axe a20pden+ [" ['-2065)-rardga, = 6be-*(2n)(5)— 10(2n)(2) = 129.4 Foe the right side of the divergence theorem: vant coe) +2 (22) = 00 a Jov-aave lf" [ Cae -ecrapde tins AB Given that Do (10-7/4)a, (C/m?)_ in cylindrical coordinates, evaluate both sides of the divergence theorem for the volume enclosed by r=Im, r=2m, z=0 and z=10m (Gee Fig. 48). fv-as=[(v-D)av 56 DIVERGENCE AND THE DIVERGENCE THEOREM ICHAP. 4 ‘Since D has no z component, D+ dS is zero for the top and bottom. On the inner cylindrical surface dS is in the direction ~a,. fr-as=[" [Bere -Wdpde(-a) +[[ Per-wdoae, ws fo-vyw-[[" fowreraete-norc 419. Given that D=(5r7/4)a, (C/m?) in spherical coordinates, evaluate both sides of the a divergence theorem for the volume enclosed by r=4m and O=2/4 (see Fig. 49). fp-as-{o-pydv ‘Since D has only a radial component, DS has a nonzero value only on the surface r= 4m. fo-ase[" [2's oranoaeage sic CHAP. 4) DIVERGENCE AND THE DIVERGENCE THEOREM 7 43h. 432. 43. 434, 435, For the right side of the divergence theorem: and Jor-vyao=["[" [nya odrdoag 509.16 Supplementary Problems Develop the divergence in spherical coordinates. Use the delta-volume with edges Ar, r A8, and rsin 8.46. ‘Show that VE is zero for the field of a uniform sheet charge. ‘The feld of an electric dipole with the charges at +1d/2.on the z axis is Qa Tae ‘Show that the divergence of this field is zero. E (2008 bu, + sin Ons) Given A=e%a,+2cosya,+2sinza,, find V+A at the origin. Ans. 7.0 Given A=@r+y?46—y")a,, find eA, Ans. 3-2y An2oa 424, 432's,, find VA st 2,-1.3). Ans. 80 Axdrya, — ay", +Ssinza,, find VA at(2,2,0). Ans. 5.0 270s? $s, +37'sin za, +42 in? a,, find +A. Ans. 40 Given Given Given Given A= (10/P)a,+5e%a,, find VA at 2,6, 1). Ans. -2.60 Given A=Scosra.+(32e*/na,, find VA at (x, 9.2). Ans. 1.59 Given A=108,+5sin Oso, find VA. Ans. (2+ c0s 6)(10/r) Given A=ra,—Peot Gay, find V+A. Ans. 3-7 Given A= [(10sin? O)/r}a, (N/m), find V+ A at (2m, x/4rad,n/2 rad). Ans. 1.25 N/m Given A=P'sina,+13¢a9+2ra,, find V-A. Ans. arsin 9+ (: Show that the divergence of E is zero if E=(100/r)a, + 40a, Inthe region ab, oon For r 2) = (E0St/r)a,. Find p in both regions. Ans. —e-**,0 In the region r<2 (cylindrical coordinates), D=(10r+(7/3)}a., and for r>2, D= [3/(128r)Ja,. Find p in both regions. = Aas. 20+7,0 Given D=10sin 6a,+200s ay, find the charge density. Ans. 28 (84+ 20006) Given a Fea™ in spherical coordinates, find the charge density. Ans. 3(7+3)/(2-+ 19° Given endear 2 ines conns, dhe che ensiy. Ane Inte rpon 11 (her cont, p-(%-2), and for r>1, D=[5/(63r)}a,. Find the charge density in both regions. Ans. 4—r7,0 The region r52m_(epherical coordinates) has a eld E= (Sex 10-%/e,)a, (W/m). Find the net charge enclosed by the shell r=2m. Ans. 5.03% 10°C Given that D-=(S+'/4)a,_in spherical coordinates, evaluate both sides of the divergence theorem for the volume enclosed between r=1 and f=2. Ans, 753 Given that D=(10r'/4)a, in cylindrical coordinates, evaluate both sides of the divergence theorem for the volume enclosed by r=2, z=0, and z=10. Ans. 800x Given that D= 10sin 62, +205 6a,, evaluate both sides of the divergence theorem for the volume enclosed by the shell r=2. Ans. 40x? Chapter 5 The Electrostatic Field: Work, Energy, and Potential 5.1 WORK DONE IN MOVING A POINT CHARGE A charge Q experiences a force F in an electric field E. In order to maintain the charge in equilibrium a force F, must be applied in opposition (Fig. 5-1): F=QE F,=-QE rer Fig 54 Work is defined as a force acting over a distance. ‘Therefore, a differential amount of work dW. is done when the applied force F, produces a differential displacement di of the charge; i.e. moves the charge through the distance @ o Fig. 55 CHAP. 5] THE ELECTROSTATIC FIELD: WORK, ENERGY, AND POTENTIAL 68 From the calculus, the change in V from M to Nis given by wv Vp wv av a aes Mays Mae ax tay * ae Now, the del operator, introduced in Section 4.4, operating on V gives av av, av w= wv ae Fay tas . It follows that aV=WV-de ‘The vector field VV (also written grad V) is called the gradient of the scalar function V. It is seen that, for fixed |drl, the change in V in a given direction de is proportional to the projection of ‘WV in that direction. Thus VV lies in the direction of maximum increase of the function V. Another view of the gradient is obtained by allowing the points M and NV to lie on the same equipotential (if V is a potential) surface, V(x,y,z)=¢, [see Fig. 5-5(6)). Then dV = (0, which implies that VV is perpendicular to dr. But dr is tangent to the equipotential surface; indeed, for a suitable location of N, it represents any tangent through M. ‘Therefore, VV must be along the surface normal at M. Since VV is in the direction of increasing V, it points from Vix,y,2)=¢, t0 V(%y,2)=Co where €2>¢). The gradient of a potential function is «a vector field that is everywhere normal to the equipotential surfaces. ‘The gradient in the cylindrical and spherical coordinate systems follows directly from that in the cartesian system. It is noted that each term contains the partial derivative of V with respect to distance in the direction of that particular unit vector. (cartesian) Cylindrical) 1 "+ T50" * rein ap? While VV is written for grad V in any coordinate system, it must be remembered that the del ‘operator is defined only in cartesian coordinates. (spherical) 5.7 RELATIONSHIP BETWEEN E AND V From the integral expression for the potential of A with respect to B, the differential of V may bbe written dV =~E-dl (On the other hand, av =Wede Since di=dr is an arbitrary small displacement, it follows that E=-w ‘The electric field intensity E may be obtained when the potential function V is known by simply taking the negative of the gradient of V. The gradient was found to be a vector normal to the equipotential surfaces, directed to a positive change in V. With the negative sign here, the E field is found to be directed from higher to lower levels of potential V. 64 THE ELECTROSTATIC FIELD: WORK, ENERGY, AND POTENTIAL — [CHAP. 5 EXAMPLE 4. In spherical coordinates and relative to infinity, the potential in the region r>0 surrounding a point charge Qis V=Q/4negr, Hence, wary BE )enaon in agreement with Coulomb's law. (V is obtained in principle by integrating E; so it is not surprising that differentiation of V gives back E.) a 5.8 ENERGY IN STATIC ELECTRIC FIELDS Consider the work required to assemble, charge by charge, a distribution of m=3 point charges. ‘The region is assumed initially to be charge-free and with E=O throughout. Referring to Fig. 5-6, the work required to place the first charge. Q1, into position 1 is zero. Then, when Q, is moved toward the region, work equal to the product of this charge and the potential due to Q, is required. The total work to position the three charges is We = Wi + Wa + Ws + (Q2Va.1) + (QsVs + QsVs.2) ‘The potential V,, must be read “the potential at point 2 due to charge Q, at position 1.” (This rather unusual notation will not appear again in this book.) ‘The work We is the energy stored in the electric field of the charge distribition. (See Problem 5.17 for a comment on this identification.) Now if the three charges were brought into place in reverse order, the total work would be Wea Wy + We + (Q2V2.s) +(QMVis+ QiVi2) When the two expressions above are added, the result is twice the stored energy: 2We = O(Vi2 + Vis) + OAV2, + Ve,s) + OsVs.1 + V5.2) ‘The term Q,(V,2+ Vi.) was the work done against the fields of Q, and Qs, the only other charges in the region. Hence, Vi2+Vis= Vi, the potential at position 1. Then 2We = O1V, + O2V2+ QsV5 and {for a region containing n point charges. For a region with a charge density p (C/m?) the summation becomes an integration, We 1 =} [ovdv Other forms (see Problem 5.12) of the expression for stored energy are We=3 fo-ea wee} [ectan 2 CHAP. 5] THE ELECTROSTATIC FIELD: WORK, ENERGY, AND POTENTIAL 65 In an electric circuit, the energy stored in the field of a capacitor is given by We = 40V =4CV? where C is the capacitance (in farads), V is the voltage difference between the two conductors making up the capacitor, and Q is the magnitude of the total charge on one of the conductors. EXAMPLE 5. A parallel-plate capacitor, for which C= ¢A/d, has a constant voltage V applied across the plates (Fig. 5-7). Find the stored energy in the electric field ‘With fringing neglected, the field is E=(V/d)a, between the plates and E=0 elsewhere Wea 5 | cetde ‘Asan alternate approach, the total charge on one conductor may be found from Dat the surface via Gauss’ law (Section 3.3). ng p=”, 7” pa ns¥4 Q=DIA=—F Joy: 1 cave) 1 2 Solved Problems 5.1. Given the electric field E=2xa,—d4ys, (V/m), find the work done in moving a point charge +2C (a) from (2, 0,0)m to (0,0,0) and then from (0,0,0) to (0,2, 0); (b) from 5 (2,0,0) to (0, 2, 0) along the straight-line path joining the two points. (See Fig. 5-8.) (@) Along the x axis, y=dy=dz=0, and dW = ~2(2xn,)+ (dr a,)= 4x dr 6 THE ELECTROSTATIC FIELD: WORK, ENERGY, AND POTENTIAL (CHAP. 5 (0.2.01 ta] oa om Fig. 58 Along the y axis, x=dredz=0, and dW = ~2(~4ya,)-(dya,) = By dy The wana fieares fyayaaes (6). The straight-line path has the parametric equations xe2-m y= 220 where 0=1=1. Hence, aW = 2122-29, — 42a] [(-2 da, + 2a.) (+ at and was [ (endre2ey 5.2. Find the work done in moving a point charge Q=5 uC from the origin to (2m, 2/4, 27/2), 1 coordinates, in the field 10 rsin 0 B= Sea + ap (V/m) In spherical coordinates, l= dra, + rdbay +rsin Odga, ‘Choose the path shown in Fig. 5-9. Along segment f, d= dp dW =-QE-dl=(-5 «10 (Se Along segment 11, dr=d0 and aw Fig. 59 GHAP, 5] THE ELECTROSTATIC FIELD: WORK, ENERGY, AND POTENTIAL a ‘Along segment II, dr=dg=0, and dW=-QE-di=0 In this case, the field does 117.9 u5 of work on the moving charge. 5.3. Given the field E=(k/r)a, in cylindrical coordinates, show that the work needed to move a point charge Q from any radial distance r to a point at twice that radial distance is independent of r. ‘Since the field has only a radial component, dW=-OE- dt For the limits of tegration use 7, and 2r,. 10 ["* independent of 7, 5.4. For a line charge pe=(10"%/2)C/m on the z axis, find Vas, where A is (2m, 7/2, 0) and Bis (4m, x, 5m). =e where B=5 ta, ince the field due to the line charge is completely in the radial direction, the dot product with dl results in E,dr. 10 2 ~[ aren He 24 8.5. In the field of Problem 5.4, find Vac, where rp=4m and rc=10m. Then find Vac and compare with the sum of Vay and Voc. Vac = ~Sfinr}2= —9{In 4—In 10) = 8.25 V Vago ~Sfin r¥g-= -9(1n2— In 10) = 18.49 Van + Vac 6.24 V + 8.25 V = 14.49 V= Vac 5.6. Given the field E=(-16/r?)a, (V/m)_ in spherical coordinates, find the potential of point 2m, x, 2/2) with respect to (4m, 0, 7). ‘The equipotential surfaces are concentric spherical shells. Let r=2m be A and B. Then Aline charge pe=400pC/m_ lies along the x axis and the surface of zero potential passes through the point (0, 5, 12) m in cartesian coordinates (see Fig. 5-10). Find the potential at 2,3, -4)m. 58. THE ELECTROSTATIC FIELD: WORK, ENERGY, AND POTENTIAL — [CHAP. 5 With the line charge along the x axis, the x coordinates of the two points may be ignored. naVOr=5m y= VE+I= 13m Pe dpa Pe thn, Breer ~~ ames ry SY Find the potential at 1=Sm_ with respect to rg=15m_ due to a point charge Q= 500 pC at the origin and zero reference at infinity. Due to a point charge, “To find the potential diference, the ero reference is not needed sto 10 (1 ‘107/362 \S “The zero reference at infinity may be used to find Vz and Vi. se Benes ‘Then Van = Vo— Vig = 0.60V Forty nanocoulombs of charge is uniformly distributed around a circular ring of radius 2m. Find the potential at a point on the axis Sm from the plane of the ring. Compare with the result where all the charge is at the origin in the form of a point charge. With the charge in Here 9-2 Oe and (oc Fig, 511) R2VBm, dem Qmydp v- [7 cosas 4n(10°°/362)V29 CHAP. 5] THE ELECTROSTATIC FIELD: WORK, ENERGY, AND POTENTIAL o 40 = o,rae Fig. 5-11 If the charge is concentrated at the origin, 40x10 Feeley OY 5.10. Five equal point charges, Q=20nC, are located at x=2,3,4,5,6m. Find the poten- 4g _ tial at the origin. = S.1L. Charge is distributed uniformly along a straight line of finite length 21 (Fig. 5-12). Show that for two external points near the midpoint, such that r, and 72 are small compared to the length, the potential Viz is the same as for an infinite line charge. moxwepaa it Gegtaeg tg) 2 % -L Fig. S12 7 THE ELECTROSTATIC FIELD: WORK, ENERGY, AND POTENTIAL — [CHAP. 5 ‘The potential at point 1 with zero reference at infinity is pide , are 2b, aA Peeing eve rR, [ln (+ WEF) - Inn) Similarly, the potential at point 2 is Now if Ln and Ln, ‘Then Which agrees with the expression found in Problem 5.7 for the infinite line. 5.12. Charge distributed throughout a volume v with density p gives rise to an electric field with energy content 1 We= 2 [ov du Stow tat neue xresion forthe seed nt ! 2 ler Figure 5-13 shows the charge-containing volume v enclosed within large sphere of radius R._ Since p vanishes outside v, 1 1 ct $f 00H Volumew Fig. 513 GAP. 5} THE ELECTROSTATIC FIELD: WORK, ENERGY, AND POTENTIAL, n 5.13. 5.14, a The vector identity V+VA=A+¥V+V(V-A), applied to the integrand, HEI WM] Ore This expression holds for an arbitrarily large radius R; the plan isto let +e. “The fist integral on the right equals, by the divergence theorem, 1 Fara” Now, as the enclosing sphere becomes very large, the enclosed volume charge looks like 2 point charge. Thus, at the surface, D appears as k,/A* and V appears as ky/R. So the integrand is Gecreasing as 1/R?. Since the surface area increases only as R*, it follows that fim f | _,VD-ds=0 “The remaining integral gives, ia theft, eo! f w-svya0=! feo-E)d0 And since D= eR, the stored energy is ako given by afm Wend feed oe wend [Pav Given the potential function V=2:+4y (V) in free space, find the stored energy in a |-m? volume centered at the origin. Examine other 1-m? volumes. Bawa Mae Ma) any (vim ‘This field is constant in magnitude (£=~V20V/m) and direction over all space, and so the total stored energy is infinite. (The field could be that within an infinite parallel-plate capacitor. It would take an infinite amount of work to charge such 2 capacitor.) Nevertheless, itis possible to speak of an energy density for this and other fields. The expression wend etree sess tht shy women cy cnet ,we w= lek For the present field, the energy density is constant: 1 07 > w=} 620) = oH and s0 every Lem? volume contains (10°*/362) J of energy. ‘Two thin conducting half planes, at @=0 and @=./6, are insulated from each other along the z axis. Given that the potential function for 0=$=2/6 is V= (-@0g/x)V, find the energy stored between the half planes for 0.1a with the zero reference at infinity. Find an expression for the stored energy that this Char. 5] THE ELECTROSTATIC FIELD: WORK, ENERGY, AND POTENTIAL B 5.19. sai. 522. sas, potential represents 0 rd. Ans. (Qdsin)/(4xeor’) Repeat Problem 5.31 with the charges on the z axis. Ans. (Qd cos O)/(Azes") Find the charge densities on the conductors in Problem 5.14. +60€ Ans, 2A im?) on = 0, =ZE(Cim') on =F ‘A uniform line charge p¢=2nC/m lies in the z=0 plane parallel to the x axisat_y=3m. Find the potential difference V,g for the points A(2m,0,4m) and B(0, 0,0) Ans. -18.4V A uniform sheet of charge, p,=(1/6n)nC/m?, is at x=0 and a second sheet, p, (6x) aCe, is at x=10m. Find Vag, Vacs and Vac for A(10m,0,0), B(4m,0,0), and (0,00). Ans. —36V, -24V, -60V Given the cylindrical coordinate electric fields E=(S/rje, (V/m) for O2m, find the potential difference Vsq for A(1‘m,0, 0) and B(4 m, 0, 0). ‘Ans. 847 A parallet-plate capacitor 0.5 m by 1.0m, has a separation distance of 2em and a voltage difference of 10V. Find the stored energy, assuming that =<, Ans. 1.1m CHAP. 5] THE ELECTROSTATIC FIELD: WORK, ENERGY, AND POTENTIAL, » 538. & é ‘The capacitor described in Problem 5.37 has an applied voltage of 200 V. (@) Find the stored energy. (@) Hold a, (Fig. 516) at 2em and the voltage difference at 200V, while increasing d, to 22em. Find the final stored energy. (Hint: AW, = (AC)V"] Ans. (@) 44 hs (b) 4.2 Wh Find the energy stored in a system of three equal point charges, @=2nC, arranged in a line with (0.5 m separation between them. Ans. 180nJ Repeat Problem 5.39 if the charge in the center is -2nC. Ans. ~108 nd Four equal point charges, Q=2nC, are to be placed at the corners of a square $m on a side, one at atime. Find the energy in the system after each charge is positioned. ‘Ans. 0, 1085, 22-5, S855 Given the electric field E=—Se"""a, in cylindrical coordinates, find the energy stored in the volume described by r=2a and O=2=Sa, Ans. 7.89x 10a" Given a potential V~3x"+4y" (V), find the energy stored in the volume described by 0x = fm, O=y=1m, and Osz<1m. Ans 167ph Chapter 6 Current, Current Density, and Conductors 6.1 INTRODUCTION Electric current is the rate of transport of electric charge past a specified point or across a specified surface. The symbol J is generally used for constant currents and é for time-variable currents. The unit of current is the ampere (1 A=1C/s; in the SI, the ampere is the basic unit and the coulomb is the derived unit). ‘Ohm's law relates current to voltage and resistance. For simple de circuits, 1= V/R. However, when charges are suspended in a liquid or a gas, or where both positive and negative charge carriers are present with different characteristics, the simple form of Ohm's law is insufficient. Consequently, the current density J (A/m?) receives more attention in electromag- netics than does current J. 6.2 CHARGES IN MOTION Consider the force on a positively charged particle in an electric field in vacuum, as shown in Fig. 6-1(a). This force, F=+QE, is unopposed and results in constant acceleration. Thus the charge moves in the direction of E with a velocity U that increases as long as the particle is in the E, field. When the charge shown in Fig. 6-1(6), it collides repeatedly with particles in the medium, resulting in random changes in direction. But, for constant E and a homogeneous medium, the random velocity components cancel out, leaving a constant average velocity, known as the drift velocity U, along the direction of E. Conduction in metals takes place by movement of the electrons in the outermost shells of the atoms making up the crystalline structure. According to the eleciron-gas theory, these electrons reach an average drift velocity in much the same way as charged particle moving through a liquid or gas. The drift velocity is directly proportional to the electric field intensity, U=yE where 1, the mobility, has the units m?/V-s, Each cubic meter of a conductor contains on the order of 10% atoms. Good conductors have one or two electrons from each atom free to move upon application of the field. The mobility 41 varies with temperature and the crystalline structure of the solid. The particles in the solid have a vibratory motion which increases with temperature. This makes it more difficult for the charges to move. Thus, at higher temperatures the mobility jis reduced, resulting in a smaller drift velocity (or current) for a given E. In circuit, analysis this phenomenon is accounted for by stating a resistivity for each material and specifying an increase in this resistivity with increasing temperature. : (@) Vacuum CHAP. 6] CURRENT, CURRENT DENSITY, AND CONDUCTORS 7 63 CONVECTION CURRENT DENSITY J A set of charged particies giving rise to a charge density p ina volume w is shown in Fig. 6-2 to have a velocity U to the right. The particles are assumed to maintain their relative positions within the volume. As this charge configuration passes a surface 5 it constitutes a convection current, with density pU (Alm) Of course, if the cross section of 1 varies or if the density p is not constant throughout v, then J will not be constant with time. Further, J will be zero when the last portion of the volume crosses S. Nevertheless, the concept of @ current density caused by a cloud of charged particles in motion is at times useful in the study of electromagnetic field theory. 6.4 CONDUCTION CURRENT DENSITY 3 (Of more interest is the conduction current that occurs in the presence of an electric field within a conductor of fixed cross section. ‘The current density is again given by J=pU (Alm) which, in view of the relation U=E, can be written J=ok where o= py is the conductivity of the material, in siemens per meter (S/m). In metallic conductors the charge carriers arc electrons, which drift in a direction opposite to that of the electric field (Fig. 6-3). Hence, for electrons, both p and u are negative, which results in 2 positive conductivity 0, just as in the case of positive charge carriers. It follows that J and E have the same ditection regardless of the sign of the charge carriers. It is conventional to treat electrons moving to the left as positive charges moving to the right, and always to report p and 41 as positive. The relation =. is often referred to as the poin form of Ohm's law. The factor o takes into account the density of the electrons free to move (p) and the relative ease with which they move through the crystalline structure (jt). As might be expected, o is a function of temperature. 28) Be U 3) |e == not > Fig. 63 B CURRENT, CURRENT DENSITY, AND CONDUCTORS (CHAP. 6 EXAMPLE 1. What electric field intensity and current density correspond to a drift velocity of 6.0 x 10-* m/s in a silver conductor? For silver o=61.7MS/m and 4=5.6%10 *m7/V-s. U_60x10* 7 on Seg T= B07 10-* W/m J= 0B = 6.61 x 10" Alm? E 68 CONDUCTIVITY o In a liquid or gas there are generally present both positive and negative ions, some singly charged and others doubly charged, and possibly of different masses. A conductivity expression ‘would include all such factors. However. if itis assumed that all the negative ions are alike and so {00 the positive ions, then the conductivity contains two terms as shown in Fig. 6-4(a). In a metallic conductor, only the valence electrons are free to move. In Fig. 6-4(b) they are shown in motion to the left. ‘The conductivity then contains only one term, the product of the charge density of the electrons free to move, p,, and their mobility, 4. 6 oF 6 76 Sot re EP E 058} ~© O- e_, EQ E o=pu tou, O= Deby = Belle + Pnbin oii or conte casement Fig. 6-4 ‘A somewhat more complex conduction occurs in semiconductors such as germanium and silicon, In the crystal structure each atom has four covalent bonds with adjacent atoms. However, at room temperature, and upon influx of energy from some external source such as light, electrons can move out of the position called for by the covalent bonding. This creates an electron-hole pair available for conduction. Such materials are called intrinsic semiconductors. Electron-hole pairs have a short lifetime, disappearing by recombination. However, others are constantly being formed and at all times some are available for conduction. As shown in Fig 6-4(c), the conductivity o consists of two terms, one for the electrons and another for the holes. In Practice, impurities, in the form of valence-three or valence-five elements, are added to create P-type and n-type semiconductor materials. The intrinsic behavior just described continues, but is far overshadowed by the presence of extra electrons in n-type, or holes in p-type. materials. ‘Then, in the conductivity 0, one of the densities, pe oF pp , will far exceed the other. EXAMPLE 2. Determine the conductivity of intrinsic germanium at room temperature ‘At 300K there are 25x10" electron-hole pairs per cubic meter. The electron mobility is u,= 0.38m'/V-s_ and the hole mobility is 44 =0.18m'/V-s. Since the material is not doped, the numbers of electrons and holes are equal. else + Ha) = (2.5 % 10")(1.6 x 10°")(0.38 + 0.18) ~ 2.24 S/m CHAP. 6] CURRENT, CURRENT DENSITY, AND CONDUCTORS n 66 CURRENT I Where current density J crosses a surface S, as in Fig. 6-5, the current I is obtained by integrating the dot product of J and dS. ansas i= [sas Of course, J nced not be uniform over S and S need not be a plane surface. Fig. 65 3. Find the current in the circular wire shown in Fig. 6-6 if the current density is J= 15(1—e-"™ Ja, (Alm). The radius of the wire is 2 mm. ‘A cross section of the wire is chosen for S. Then dl=s-d8 = 151 ea, -rdrdpa, we reff [7 sae marae $1.33 x10 A = 0.133 mA Any surface $ which has a perimeter that meets the outer surface of the conductor all the way around will have the same total current, f= 0.133 mA, crossing it 80 CURRENT, CURRENT DENSITY, AND CONDUCTORS IcHaP. 6 6.7 RESISTANCE R Ifa conductor of uniform cross-sectional area A and length ¢, as shown in Fig. 6-7, has a voltage difference V between its ends, then wv E ind gz and =F assuming that the current is uniformly distributed over the area A. The total current is then AV paw ¥ é ‘Since Ohm's law states that V=IR, the resistance is é a @ (Note that 1S"'=10; the siemens was formerly known as the mho.) This expression for resistance is generally applied to all conductors where the cross section remains constant over the length ¢ However, if the current density is greater along the surface area of the conductor than in the center, then the expression is not valid. For such nonuniform current distributions the resistance is given by v v R= vy fa-as fop-as If E is known rather than the voltage difference between the two faces, the resistance is given by fea R= fon-as ‘The numerator gives the voltage drop across the sample, while the denominator gives the total current BR ccwwie 4 rintne roisnce fervent met and outer ced ses th Hos shown in Fi 68 where the material is silver for which o = 6.17 x 10” S/m. ee etm, Har. 6 CURRENT, CURRENT DENSITY, AND CONDUCTORS at Then (5°=0.0873 rad), (Chae “PP reoen Ins sa aay = hil 10*A=10.1 na (0.05)(0.0873) a 68 CURRENT SHEET DENSITY K At times current is confined to the surface of a conductor, such as the inside walls of a waveguide. For such a current sheet it is helpful to define the density vector K (in A/m), which s the rate of charge transport per unit transverse length. (Some books use the notation 4). Figure 6-9 shows a total current of J, in the form of a cylindrical sheet of radius r, flowing in the positive z direction. In this case, at each point of the sheet. For other sheets, K might vary from point to point. Fig. 69 In general, the current flowing through a contour C within a current sheet is obtained by integrating the normal component of K along the contour. r= [ Kae EXAMPLE 5. A thin conducting sheet lies in the z=0_ plane for 0x 10" ke/m* and the atomic weight is 26.98 kg/kmol. Then = (6.02% 10°)( 525) SQ _ (6.02% 10 clectrons/m')(10-? m)(1.6 x 10" C/electron) _ a ws ARKA (2.70 10") = 6.02 x 10 electrons/m? and What is the density of free electrons in a metal for a mobility of 0.0046m"/V-s and a conductivity of 29.1MS/m? CHAP. 61 CURRENT, CURRENT DENSITY, AND CONDUCTORS 87 6.7. 68. Since o= up, o_ 291x106 a 0.0086 633x110" Lex10 = 6.33 10° Cim? and N= 3.96% 10 electrons/m* Find the conductivity of n-type germanium (Ge) at 300 K, assuming one donor atom in each 10" atoms. The density of Ge is 5.32 x 10° kg/m? and the atomic weight is 72.6 kg/kmol. ‘The carriers in an n-type semiconductor material are electrons. Since Lkmol of a substance ‘contains 6.02 x 10 atoms, the carrier density is given by (ows BE om = 4.41 10" electrons/m? ‘The intrinsic concentration n, for Ge at 300K is 2.5% 10"m™?. The mass-action law, NN, = I then gives the density of holes: assy aa TO Because N,% Ny, conductivity will be controlled by the donated electrons, whose mobility at 300K is 10" atoms. 42x 10" holes/m* = Neetie= (4.41 x 10")(1.6 X 10-)(0.38) = 26.8S/m ‘A conductor of uniform cross section and 150m long has a voltage drop of 1.3V and a current density of 4.65% 10° A/m?. What is the conductivity of the material in the conductor? Since E=V/é and J=0E, sesxio'eo(23) or ons 10 sin A table of resistivities gives 10.40hm circular mils per foot for annealed copper. What is the corresponding conductivity in siemens per meter? A circular mit isthe area of a circle with a diameter of one mil (10~ seen af (272) o0eug)f ‘The conductivity is the reciprocal of the resistivity. 07 «10% mi? = 5.7810" Sim An AWG #20 aluminum wire has a resistance of 16.7ohms per 1000 feet. What conductivity does this imply for aluminum? From wie tables, a #20 wire has a diameter of 32 mil. pe » Aza 0.0254) = 5.19 10°? m £€ = (1000 £1)(12in/fe)(0.0254 mfin) = 3.05 * 10% m 88 CURRENT, CURRENT DENSITY, AND CONDUCTORS [cHar. 6 Then from R= é/0A, oss 10° TaN E.19% 10) ~ 2 MS/m 6.10, In a cylindrical conductor of radius 2mm, the current density varies with the distance from the axis according to ry F=10e" (A/m) Find the total current J. to frase foes [’[Wwemaae =2x(10°)| a5 Ga oor) =751mA 6.11. Find the current crossing the portion of the y=0 plane defined by -O.1sx5 O1m and ~0.002<7<0.002m if S=10 xia, (Am?) iefrase[" fein, 6.12. Find the current crossing the portion of the x=0 plane defined by —x/4sy= x/4m and -0.0157=0.01m if J 100cos2ya, (A/m?) 1 fa-as= ff 100cosaya ay dea, =208 drdza, =4mA, in spherical coordinates, find the current crossing the spheri- dS =P sinOd0dou, ct f 10%(0.02)' sin? 640.49 =3.95 A 6.14, Show that the resistance of any conductor of constant cross-sectional arca A and length ¢ is given by R=€/0A, assuming uniform current distribution. ‘A constant cross section along the length ¢ results in constant £, and the voltage drop is vafeanee If the current is uniformly distributed over the +A, 1=[svas-s4~06A where o is the conductivity. Then, since R= V/t, é nok CHAP. 6] CURRENT, CURRENT DENSITY, AND CONDUCTORS 89 6.15. Determine the resistance of the insulation in a length ¢ of coaxial cable, as shown in Fig. 6-14. | | 3 Fig. 614 Assume a total current J from the inner conductor to the outer conductor. Then, at 2 radial distance 7, and so et ‘The voltage difference between the conductors is then 1 tenn sre and the resistance is voto RT" inot a 6.16. A current sheet of width 4m lies in the z= plane and contains a total current of 10 A in a direction from the origin to (1,3,0)m. Find an expression for K. ‘At each point ofthe shect, the direction of K isthe unit vector 2135 vio and the magnitude of Kis A/m. Thus 10/a, + 38 aig") 6.17. As shown in Fig. 6-15, a current Ir follows a filament down the z axis and enters a thin conducting sheet at z=0. Express K for this sheet. fi, “Che » Fig. 615 % CURRENT, CURRENT DENSITY, AND CONDUCTORS IcHar. 6 Consider a circle in the 2=0 plane. The current /-on the sheet spreads out uniformly over the circumference 277. The direction of K is a,. Then xe 6.18, For the current sheet of Problem 6.17 find the current in a 30° section of the plane (Fig. 6-16) hp en + AA i 4 Fig. 6-16 tf ee=[feraonte However, integration is not necessary, since for uniformly distributed current a 30° segment will contain 30°/360" or 1/12 of the total. 6.19. A current 1(A) enters a thin right circular cylinder at the top, as shown in Fig. 6:17. Express K if the radius of the cylinder is 2m. ' cS Pig. 6:17 ‘On the top, the current is uniformly distributed over any circumference 2x7, so that 1 K=55a (Alm) Down the side, the current is uniformly distributed over the circumference 2(0.02m), so that L oie 1) (Alem) |. A cylindrical conductor of radius 0.05 m with its axis along the z axis has a surface charge density p,=polz (C/m?). Write an expression for E at the surface. Since D,= Pr Ex= Pleo. At (0.05, 0, 2), E-E, 6.21. A conductor occupying the region x=5 has a surface charge density me We Write expressions for E and D just outside the conductor. CHAP. 6] CURRENT, CURRENT DENSITY, AND CONDUCTORS on ‘The outer normal is ~a,. Then, just outside the conductor, D=D,¢ ey +z 6.22. Two concentric cylindrical conductors, r,=0.01m and, =0.08m, have charge densities ya =40pC/m? and P,,, such that D and E fields exist between the two cylinders but are zero elsewhere. See Fig. 6-18. Find py and write expressions for D and E between the cylinders. Fig. 618, By symmetry, the field between the cylinders must be radial and a function of r only. Then, for n1. In terms of the capacitance C and the voltage V this stored ‘energy is given by We = CV? and the energy increase relative to free space is reflected in C, which is directly proportional to €, 7.8 FIXED-VOLTAGE D AND E A parallel-plate capacitor with free space between the plates and a constant applied voltage V, as shown in Fig. 7-6, has a constant electric field intensity E. With fringing neglected, v CHAP. 7] CAPACITANCE AND DIELECTRIC MATERIALS 99 Fig. 746 Now, when a dielectric with rel permittivity ¢, fils the space between the plates, Ey D=eDy because the voltage remains fixed, whereas the permittivity increases by the factor €, EXAMPLE 4. A parallel-plate capacitor with free space between the plates is connected to a constant source of voltage. Determine how Wz, C, Q, and p, change as a dielectric of ¢,=2 is inserted between the plates. Insertion of the diclectric causes additional charge in the amount Q, to be pulled from the constantwoltage 7.6 FIXED-CHARGE D AND E ‘The parallel-plate capacitor in Fig. 7-7 has a charge +Q on the upper plate and —Q on the lower plate. This charge could have resulted from the connection of a voltage source V which was subsequently removed. With free space between the plates and fringing neglected, =2 POA In this arrangement there is no way for the charge to increase or decrease, since there is no conducting path to the plates. ‘Thus, when a dielectric material is inserted between the plates, D=D, E Fig. 7-7 EXAMPLE 5. A charged parallel-plate capacitor in free space is kept electrically insulated as a dielectric of relative permittivity 2 is inserted between the plates. Determine the changes in We, C, and V. Relationship Explanation We = $Wen D-E=1Dp-Ey vem V=Ed C=2C, c=0lv (See Problem 7.20.) 100 CAPACITANCE AND DIELECTRIC MATERIALS, [cHar. 7 7.7 BOUNDARY CONDITIONS AT THE INTERFACE OF TWO DIELECTRICS If the conductor in Figs. 6-11 and 6-12 is replaced by a second, different, dielectric, then the same argument as was made in Section 6.10 establishes the following two boundary conditions: (1) The tangential component of E is continuous across a dielectric interface. In symbols, Dy _Da ea (2) The normal component of D has a discontinuity of magnitude \p,| across a dielectric interface. If the unit normal vector is chosen to point into dielectric 2, then this condition can be written Ey=Eq and Pe Da- =p, and en Eg — €2Ene Generally the interface will have no free charges (p,=0), so that Dar=Da and en Ens saEn D, and the angles 0, and 6 Fig. 78 ‘The interface is a z= const. plane. The x and y components are tangential and the 2 components are normal. By continuity of the tangential component of E and the normal component of D: 2a,— 3+ Sa, 2a, Jat Exe, = deja, — 66,8, + 10€,, Dasa, + Dy, + Weve, ‘The unknown components are now found from the relation Ds= €yénEs. Dasa, + Dy, + Wega, = 2e6 28, ~ Rend, + Gob rE a, from which Da =2edea= We Dy Bex =—I5eq ‘The angles made with the plane of the interface ate essiest found from IEslcos(9C"— 6) Ea~a, = | eos (90° 6,) 5=VIRsin 8, i sin = 542" c cHae. 7] CAPACITANCE AND DIELECTRIC MATERIALS 101 ‘A useful relation can be obtained trom En Dyl €o6 VER+E, VEn+En Ea__ Daltsta In view ofthe continuity relations, division ofthese two equations gives tan 8, tan 8, Solved Problems Find the polarization P in a dielectric material wtih €,=2.8 if D=3.0x 10~’aC/m’. Assuming the material to be homogeneous and isotropic, Payee Since D=e6E and z.=<, P= (S=!)o~19sx10"ac 72. Determine the value of E in a material for which the electric susceptibility is 3.5 and P=2.3x 10%aC/m?. Assuming that P and E ae in the same direction, Ee pa7.2« 10aV/m yee 7.3. Two point charges in a dielectric medium where €,=5.2 interact with a force of 8.6%10°N. What force could be expected if the charges were in free space? Coulomb's Iaw, F = Q,0,/(4xeye4°), shows that the force is inversely proportional to «,. In free space the force will have its maximum Value. 7.4, Region 1, defined by x<0, is free space, while region 2, x>0, is a dielectric material for which «,=2.4. See Fig. 7-9. Given os = Dy 3a, —49, +68, C/m? find E, and the angles @, and 8. ‘The x components are normal to the interface: D, and E, are continuous. Data ys oe, Beda tet, ws D, = 38, + Dyas, + Dat, E,= Eat. —Lay toa, 102 15. 76. CAPACITANCE AND DIELECTRIC MATERIALS [cuar. 7 Fig. 7.9 Tren Di= ev€aF2 gives 3a, + Dt, + Da = ec biak~ Am, +668, whence Fann B yates ‘To find the angles: (D,| cos (90° — @,) 3= Valin 0, =26 Sinilay, 0 In the free-space region x<0 the electric ficld intensity is Ei—3a, +5a,—3a, Vim. ‘The region x>0 is a dielectric for which ¢.~3.6. Find the angle 6» that the field in the dielectric makes with the x=0 plane. ‘The angle made by By is found from Ey-a, = [B,c0s (AP — 6) 3= Va3 sin 8 Oame ‘Then, by the formula developed in Example 6, tan 0,=— tan 0, =0.1428 and 6)=8.13" A dielectric free-space interface has the equation 3x +2y+z=12m. The origin side of the interface has ¢;=3.0 and K,=2a,+5a, Vim. Find Ey. ‘The interface is indicated in Fig. 7-10 by its intersections with the axes, ‘The unit normal vector on. the free-space side is 3a vis “The projection of Ey on a, is the normal component of Eat the interface. u Evan CHAP. 7] CAPACITANCE AND DIELECTRIC MATERIALS 103 Fig. 7-10 Then u Eq oth g, = 2.360, +1578, +0.79a, Vie E, =F) —E.1= 0.360, — 1.57e, + 4.2, Di CE En = € (1.088, + 47a, +2378, Eat Mas = 7.088, + 4.714, + 2.378, and finally BE, Eqg + Ea" 6.722, +3.14a, +6.588, V/m 7.7. Figure 7-11 shows a planar dielectric slab with free space on either side. Assuming a constant field Ey within the slab, show that E, =, fe Fig. 7-11 By continuity of E, across the two interfaces, Eo= En By continuity of D, across the two interfaces (no surface charges), Dyy= Dm a6 50 Ey Eu Consequently, Ey=F,. (@)_ Show that the capacitor of Fig. 7-4(a) has capacitance €o6aAs , CofnA2 c CHG + GROG, (6) Show that the capacitor of Fig. 7-5(a) has reciprocal capacitance 1 1 1 oa Ca GeenAld, * &endld, C,* Cy 104 CAPACITANCE AND DIELECTRIC MATERIALS, [cHaP. 7 (a) Because the voltage difference V is common to the two diclectrics, DD LY fea Gon ae v, E.=E= 58, and ‘where a, is the downward normal to the upper plate. Since D,=p., the charge densities on the two Sections of the upper plate are ¥, Y putes — pam beta and the tot hare i elas, a) ‘Thus, the capacitance of the system, C.,=Q/V, has the asserted form. (6) Let +0 be the charge on the upper plate. Then’ 2 Pos + PaAs=V(' p=2., A everywhere between the plates. $0 that o o Bea eed The voltage differences across the two dielectrics are then = Ed, = 2 = Ot VBA Eade mt Veni nno( aa ala) + From this itis seen that 1/C.,=V/@_ has the asserted form. 7.9. Find the capacitance of a coaxial capacitor of length L, where the inner conductor has radius a and the outer has radius b. See Fig, 7-12. a Fig. 7212 With fringing neglected, Gauss’ law requires that Dé=1/r between the conductors (see Problem 6.2). Al r=a, D=p,, where p, is the (assumed positive) surface charge density on the inner conductor. Therefore, D=p.2a, and the voltage difference between the conductors is van =f (Ln) tn CHAP. 7] CAPACITANCE AND DIELECTRIC MATERIALS 105 ‘The total charge on the inner conductor is Q=p,2xal.), and so 2meceL (la) 7.10. In the capacitor shown in Fig. 7-13, the region between the plates is filled with a dielectric having ¢,=4.5. Find the capacitance. Fig. 743 With fringing neglected, the D field between the plates should, in eylindrical coordinates, be of the form D=Daay, where D, depends only on r. Then, if the voltage of the plate @ =a with respect to the plate =O is Vo, ue foa--f( Thus, Dy=—€o,Volra, and the charge density on the plate g=a is Me 85) (dong) ~ 22 [dp = Patt p=D, ‘The total charge on the plate is then given by = fas f [Maras afl O_eehyn woe My Hence ce ‘When the numerical values are su! tuted (with «converted to radians), one obtains C= 7.76pF. 7.11. Referring to Problem 7.10, find the separation d which results in the same capacitance when the plates are brought into parallel arrangement, with the same dielectric in between. ‘With the plates parallel 106 CAPACITANCE AND DIELECTRIC MATERIALS: (char. 7 so that eoeh(n—n) __ a(a=n) (exe hlain(ir)) In (edn) Notice that the numerator on the right is the difference of the are lengths at the two ends of the capacitor, while the denominator is the logarithm of the ratio of these arc lengths. For the data of Problem 7.10, ar,=0.087 mm, a7,=2.62 mm, 7.42. Find the capacitance of an isolated spherical shell of radi ‘The potential of such a conductor with a zero reference at infinity is (see Problem 2.34) 2 neva 2. Then Ca Ga Arcoa 7.13. Find the capacitance between two spherical shells of radius a separated by a distance d> a ‘As an approximation, the result of Problem 7.12 for the capacitance of a single spherical shell, nea, may be used. From Fig. 7-14 two such identical capacitors appear to be in series. 3.3 nF Similarly, C,=31.0nF. Then eit 4 a a Pr quar. 7] CAPACITANCE AND DIELECTRIC MATERIALS, 107 7.18. Repeat Problem 7.14 if the two dielectrics each occupy one-half of the space between the cix Plates but the interface is parallel to the plates. Eo _ S06 _ (885410 0-5) _ 5a 1 ap a da 72 Similarly, C,=124nF. Then 7.16. In the cylindrical capacitor shown in Fig. 7-16 each dielectric occupies one-half the volume. Find the capacitance. Fig. 7-16 ‘The diclectric interface is parallel to D and E, so the configuration may be treated as two capacitors in parallel. Since each capacitor carries half as much charge as a full cylinder would carry, the result of Problem 7.9 gives “Wala where rang= Mn+ €n)- The two dielectrics act like a single dielectric having the average relative permittivity 7.17. Find the voltage across each dielectric in the capacitor shown in Fig. 7-17 when the applied a voltage is 200 V. = 50) 5000, 107 1000¢y 108 7.18, 719. CAPACITANCE AND DIELECTRIC MATERIALS ICHAP. 7 in the capacitor is now found from @_CV_(2.77x 10-9200) d,=9,- 825 ‘ = 5.54 10-7 Cf’ Then, Bye Peaiasxiwtvim B= 2 = 6.25% 10°V/m foe © from which Vi = Bad, = 187.5 V Find the voltage drop across each dielectric in Fig. 7-18. whe 5.0. The inner conductor is at_m=2em and the outer at 7 electric interface halfway between. €1=20 and eg 25em, with the ‘The voltage division is the same as it would be for full right circular cylinders. The segment shown, with angle a, will have a capacitance a/2x times that of the complete coaxial capacitor. From Probiem 7.9, f) Battal — at41.5% 10") 2a! to .25/2.0) 144.210") (F) W; and VY+¥=V, it follows that G Since Q= CVs Y= “gg (t= 74V 3 c Ls “ere iseaa (100) = 26 ‘A free-space parallel-plate capacitor is charged by momentary connection to a voltage source V, which is then removed. Determine how W,, D, E.C, and V change as the plates are moved apart to a separation distance d,=2d, without disturbing the charge. Har. 7] CAPACITANCE AND DIELECTRIC MATERIALS, 109 7.20. 721. Explain physically the energy changes found in (a) Problem 7.19, (6) Example 5. (@) External work (in the amount W,,) is done on the system in forcing apart the oppositely charged plates. ‘This work shows up as an increase in internal energy (stored in the E field). (&) The charged plates draw the dielectric slab into the gap. Thus the system performs work (in the amount Wr.) on the surroundings—specifically, on whatever is guiding the slab into position. "The internal energy suffers a corresponding decreas. A parallel-plate capacitor with a separation d=1.0cm has 29KV applied when free space is the only dielectric. Assume that air has a dielectric strength of 30kV/em. Show why the air breaks down when a thin piece of glass (¢,~6.5) with a dielectric strength of 290 kV/em and thicknesses d,=0.20cm_ is inserted as shown in Fig. 7-19. Ait Glas, Fig. 739 “The problem becomes one of two capacitors in series, ‘Then, as in Problem 7.18, 3250 280 (29) =27.93kV Was 5m) OD =27.958 so that 2193KV _ TBbem ~ 3tOKV/em which exceeds the dielectric strength of air. Find the capacitance per unit length between a cylindrical conductor of radius a and a ground plane parallel to the conductor axis and a distance h=6.0m from it. ‘A useful technique in problems of this kind is the meihod of images. ‘Take the mirror image of the ‘conductor in the ground plane, and let this image conductor carry the negative of the charge distribution fon the actual conductor. Now suppose the ground plane is removed. It is clear that the electric field Of the two conductors obeys the right boundary condition at the actual conductor, and, by symmetry, thas an equipotential surface (Section 5.6) where the ground plane was. Thus, this field is she field in the region between the actual conductor and the ground plane. Approximating the actual and image charge distributions by line charges +p, and ~py, respectively, at the conductor centers, one has (sce Fig. 7-20): Sem Potent ata de 4p,= ~(#24) na Potent at pin P ue tp, ~ (=) nee) 10 1B. 7A, 725. CAPACITANCE AND DIELECTRIC MATERIALS [cHar. 7 ‘The potential due to pe is not constant over =a, the surface of the actual conductor. But itis very nearly so if ah. To this approximation, then, the total potential of the actual conductor is Be Pe Beg * Bes Similarly, the potential of the image conductor is ~V,, Thus, the potential difference between the conductors is 2¥,, so that the potential difference between the actual conductor and the ground plane ay Pe tna Le In 2h =P In in (2h — a) nea" * ome nea is {QV,)= Vz. “The desired capacitance per unit length is then c 2xeo L in hla) For the given values of a and h, C/L=9.0pF/m. “The above expression for C/L is not exact, but provides a good approximation when ah (the practical ease). An exact solution gives aes (an Cheeta C/o he souse-inage st (ore get, fo ay pra ini ‘conductors with centef-to-center separation 2h) is one-half the value found above (same charge, twice the voltage). That is, with d=2h, c wn in(d/a) Supplementary Problems Find the magnitudes of D, P, and ¢, for a dielectric material in which E=0.1SMV/m and y,= 425, Ans. 697 yClm*, 5.64 C/mt, 5.25 Ina dielectric material with ¢,=3.6, D=285nC/m?. Find the magnitudes of E, P, and x, Ans. 8.94kV/m, 206.nC/m?, 2.6 Ginn B=-3644 =, Viv in he spin <0, whee #20, fat Ee regen 2° rw =65 Ane eden, Vim cur. 7] CAPACITANCE AND DIELECTRIC MATERIALS, ut 726, 721. 728. 729. 730. 73h. 732. 723. 734. 138. 136. 731. 738. Given that _D=2a,—48, +158, Cli? in the region x>0, which is free space, find P in the region x<0, which is a dielectric with €=5.0. Ans. 1.60, ~ 16a, +68, C/m? Region 1, 21m, has ¢=30. Find E;, Pr, and 6, an 2 +m) (Win), 7554424, Cin 25.02 ‘The plane interface between two dielectrics is given by x+z=S. On the side including the origin, D,=(4.5a,+3.28,)10-? and €=43, while on the other side, ¢2=1.80. Find Ey, Ez, Dz, and 6. Ans. 1.45 x 10", 3.37 x 10", 5.37% 1077, #3.08° A dielectric interface is described by 4y +3z= 12m, The side including the origin is free space where D,=a, +3a,+2a, 4C/m’. On the other side, €=3.6. Find D, and 6. Ans, 5.14 uClm, 44.4° Find the capacitance of parallel-plate capacitor with a dielectric of ¢,=3.0, area 0.92m", and separation 45mm, Ans. 5.43 nF A parallel-plate capacitor of 8.0 nF has an area 1.51 m? and separation 10mm. What separation would bbe required to obtain the same capacitance with free space between the plates? Ans. 1.67 mm. Find the capacitance between the inner and outer curved conductor surfaces shown in Fig. 7-21. Neglect fringing. Ans. 6.86 pF Find the capacitance per unit length between a cylindrical conductor 2.75 inches in diameter and & parallel plane 28 ft from the conductor axis. Ans. 8.99 pF/m (note units) Double the conductor diameter in Problem 7.33 and find the capacitance per unit length Ans. 10.1 pF/m Find the capacitance per unit length between two parallel eylindrical conductors in air, of radius 1.5.0m and with a center-to-center separation of 85cm. Ans. 6.92 pF/m A parallel-plate capacitor with area 0.30m? and separation 5.5mm contains three dielectrics with imerfaces normal to E and D, as follows: €,=3.0, d\=10mm; €,=40, d;=20mm; 1760, d)=2Smm. Find the capacitance. Ans. 2.128F ‘With a potential of 1000 V applied to the capacitor of Problem 7.36, find the potential potential gradient (electric field intensity) in each dielectric. ‘Ans. 261, 267 kW/m; 400 V, 200kV/my; 333 V, 133 kV/m iflerence and Find the capacitance per unit length of a coaxial conductor with outer radius 4 mm and inner radius 0.5 mm ifthe dielectric has €=5.2. Ans. 139 pF/m 12 139. 740, Tal. 742. 148, 144, 745. 7.46. 142. CAPACITANCE AND DIELECTRIC MATERIALS, [cHar. 7 Find the capacitance per unit length of a cable with an inside conductor of radius 0.75cm and a cylindrical shield of radius 2.25 cm if the dielectric has ¢,=270. Ans. 137pF/m ‘The coaxial cable in Fig. 7-22 has an inner conductor radius of 0.5 mm and an outer conductor radius of ‘Smm. Find the capacitance per unit length with spacers as shown. Ans. 45.9 pF/m ass fo [==10 mem 1-50 mm— Fig. 722 ‘A paralle-plate capacitor with free space between the plates is charged by momentarily connecting it to constant 200-V source. After removal from the source a dielectric of ¢, completely filling the space. Compare the values of W,, D, E, p,, V, and C after insertion of the dielectric to the values before. Partial Ans. V.=3V; A parallel-plate capacitor has its dielectric changed from €,=20 to €2=60. Its noted that the stored energy remains fixed: Wi=W,. Examine the changes, if any, in V, C, D, E, Q, and p,. Partial Ans. pan=V3p,, ‘A paralet-plate capacitor with free space between the plates remins connected to a constant voltage source while the plates are moved closer together, from separation dto fd." Examine the changes in O, purC,D, E, and We. Partial Ars. De= 2D, ‘A parallel-plate capacitor with free space between the plates remains connected to @ constant voltage source while the plates are moved farther apart, from separation d to 2d. Express the changes in D, E,Q,p.. Cand We. Partial Ans. D, 1D, A parallel plate capacitor has free space as the dielectric and a separation d. Without disturbing the charge Q, the plates are moved closer together, to d/2, with 2 dieledic of €,=3 completely filing the space between the plates. Express the changes in D, E,V, C, and We. | Paria! Ans. V= A parallel-plate capacitor has free space between the plates. Compare the voltage gradient in this free space to that in the free space when a sheet of mica, ¢,=5.4, fils 20% of the distance between the plates. Assume the same applied voltage in each case." Ans. 0.84 A shielded power cable operates at 2 voltage of 12.5kV on the inner conductor with respect to the cylindrical shield. “There are two insulations; the first has €,,=6.0 and is from the inner conductor at_r=O.8em to r=1.0cm, while the second has €2=3.0 and is from r=1.0cm to r= 3.0¢m, the inside surface of the shield. Find the maximum voltage gradient in each insulation, ‘Ans. 0.645 MV/m, 1.03 MV/m CHAP. 7) CAPACITANCE AND DIELECTRIC MATERIALS 3 7.48. A shielded power cable has a polyethylene insulation for which ¢,= 2.26 and the dielectric strength is. zig 18.1MV/m. What is the upper limit of voltage on the inner conductor with respect to the shield when GB. the inner conductor has a radius 1em and the inner side of the concentric shield is at radius of Sem? Ans. 0.376MV 749. For the coaxial capacitor of Fig. 7-16, a=3em, 6=12em, €,=2.50, €2=40. Find Ey, Es, 1D, , and D, if the voltage difference is $0V. Partial Ans. E,= £(36.1/r)a, (V/m) 7-50. In Fig 7-23, the center conductor, r=mm, is at 100V with respect fo the outer conductor at n=100mm. The region 1 woot) -ae] navel“) (00V. Find the voltage at @ =20", av, s0=-e.4i( senor) 0.268 Solving gives @ = 17.41° 8.13. With reference to Problems 8.11 and 8.12 and Fig. 8-11, find the charge distribution on the 1g conducting plane at 6,=90°. “The polis bid by ting 6-97, Soo ant. tae n(n?) ‘in (ean 5*) —__=100 = 10", and ¥,=100-V_ in the expres- Vv =100, 126 LAPLACE’S EQUATION [CHAP. 8 Fig. 11 ‘On the plane 8=91F, sin @=1 the direction of D requires that the surface charge on the plane be negative in sign. Hence, pn 2S" Cm) 8.14, Find the capacitance between the two cones of Fig. 812. Assume free space. ta aL Fe If fringing is neglected, the potential function is given by the expression of Problem 8.11 with 0,=75, 0,= 10S" Thus In (tan 52.5%) 129192) ont from which 1.890, sin 8 = GE “The charge density on the upper plate is then CHAP. 8] LAPLACE’S EQUATION 127 ‘so that the total charge on the upper plate is LEK, - = fas f° [OLE ints dra = 12.28%, andthe capacitance fs C= Q/Y,= 12.28, 8.15. The region between two concentric right circular cylinders contains uniform charge density p. Use Poisson's equation to find V. Neglecting fringing, Poisson’s equation reduces 10 raZ) Note that static problems involving charge distributions in space are theoretical exercises, since no ‘means exist to hold the charges in position against the coulomb forces. 8.16. The region rated 25292 has a charge density p=10"*cos (2/20) (C/m’). Elsewhere the charge density is zero. Find V and E from Poisson's equation, and compare with the results given by Gauss? law. is not a function of x or y, Poisson's equation is av. 10"" cos (2/0) ae € Inering ie, v =tdesseled oy = (282) 4a, vray But, by the symmetry of the charge distribution, the field must vanish on the plane 2= 0. ‘Therefore A=0 and +Ar+B (VY) ea WTAE, (Im) ‘A special gaussian surface centered about 2=0 is shown in Fig. 8-13. D cuts only the top and bottom surfaces, each of area A. Furthermore, since the charge distribution is symmetrical about 2 = 0, D must be antisymmetrical about z=0, $0 that Dip=Da,, Dioana = D(—2,): pf a+of asf f| 10-*c0s 2/2) de dy de 2DA = 2apA10™*sin (2/22) 128 8.17. i: LAPLACE'S EQUATION ICHaP. 8 or D=z10*sin(z/z) for O<20. Since the potential is not a function of y, the equipotential surfaces extend to += in the y Girection. Because sinaz=0 for z=nx/a, where =0,1,2,..., the planes 2=0 and z=afa are at rero potential. Because sinhax=0 for x=0, the plane x=0 is also at zero potential. The V=0 equipotential is shown as a heavy broken line in Fig. 8-14. ‘The V=Y% equipotential has the equation Ye=Vlsinn axysinaz) or sina = When values of z between zero and 2x/a are substituted, the corresponding x coordinates are readily CHAP. 8) LAPLACE'S EQUATION 129 Fig. 816 obtained. For example: 1s7 | 1.02 | 0.67 | 0.49 | 0.28 | 0.10 2 | ys7 | 212 | 247 | 265 | 2.86 | 3.04 ax | ass [10 | 125 [1.50 | 200 | 300 ‘The equipotential, which is symmetrical about 2 8:14, Because vis periodic in z, and because V(—x, ~2 with replicas of the strip shown in Fig, §-14 (2a, is shown as a heavy curve in Fig. V(x, z), the whole xz plane can be filled 8.19. Find the potential function for the region inside the rectangular trough shown in Fig. 8-15. i ver Fig. 815 ‘The potential is a function of x and 2, of the form (see Section 8.7) V =(C,cosh az + C,sinhaz(C; cosax + C,sinax) ‘The conditions V=0 at x=0 and r=0 require the constants C, and C, to be zero. Then since V=0 at x=c, a=nn/c, where n is an integer. Replacing CC, by C, the expression becomes, v= Cink in ‘or more generally, by superposition, ‘The final boundary condition requires that v= 3 (os 130 LAPLACE’S EQUATION [CHAP. 8 ‘Thus the constants b, =C, sinh (nind/e) are determined as the coefficients in the Fourier sine series for f(x)#¥, inthe range 0 10's, Vim for 2>0 and the region con- tains a dieletric for which ¢,=45. Ans. 200V, 266nC/m? In cylindrical coordinates. V=7S.0V at r=Smm and V=0 at r=60mm. Find the voltage at_r=130mm. if the potential depends only ony. Ans. —23.34V Concentric, right circular, conducting cylinders in free space at r=Smm and r=25mm have voltages of zero and Vs. respectively. If E=-8.28% 10's, Vim at r=1Smm, find ¥q and the charge density on the outer conductor. Ans. 200, +44 nC/mn® For concentric conducting cylinders, V=75V at r=1mm and V=0 at in the region between the cylinders, where €,=3.6 Ans. (798/r)a,(pC/m") 20mm. Find D Conducting planes at @=10" and =O" in cylindrical coordinates have voltages of 75 V and zero, respectively. Obtain D in the region between the planes, which contains a material for which 621.65. Ans. (-6.28/r)a, (nC/m’) ‘Two square conducting planes S0)cm on a side are separated by 2.0 cm along one side and 2.5em along. the other (Fig. 8-17). Assume a voltage difference and compare the charge density at the center of one plane to that on an identical pair with a uniform separation of 2.0em. Ans. 0.89 Fig. 8:17 ‘The voltage reference is at r=1Smm in spherical coordinates and the voltage is Vo at r= 200mm. Given E=~334.7a, Vim at r= 110mm, find Yj. The potential is a function of r only. Ans. 250 In spherical coordinates, V=865V at r=S0em and E=748.2a,V/m at r=85em. Deter- mine the location of the voltage reference ifthe potential depends only on. Ans. r= 250em wit of 6, ero reference at infinity and V=45.0V_ at_r=0.22m_ in spherical coordinates, a dielectric TL occupies the region 0.221.00m. Deter- mine Dat r=1.0040m. Ans. 8.55V/m, 14.7V/m In Fig. 8:18 the cone at @=45° has a voltage V with respect to the reference at @=30"°. At r= Fig. 8:18 132 LAPLACE'S EQUATION [cHar. 8 025m and @=30%, Ans. 1255 2.30% 102, V/m. Determine the voltage difference V. 8.32, In Problem 831 determine the surface charge densities on the conducting cones at 30" and 45°, if 833. €,=2.45 between the cones. Ans. Find E in the region between the two cones shown in Fig. 819. Ans. 0.288¥, (vim) 220° H>0 veo Big. 8:19 In ‘cylindrical coordinates, p= 111/r (pC/m’). Given that V=0 at r=10m and V= SOV at r=3.0m due to this charge configuration, find the expression for E. Ans, (125-2), (vim) Determine E in spherical coordinates from Poisson's equation, assuming a uniform charge density p. Specialize the solution found in Problem 8.35 to the case of a uniformly charged sphere. ‘Ans. See Problem 2.54 Assume that a potential in cylindrical coordinates is a function of + and z but not, V= RONZ(2). Write Laplace's equation and obtain the separated differential equations in r and z. Show that the solutions to the equation in r are Bessel functions and that th solutions in z are exponentials or hyperbolic functions. Verify that the first five Legendre polynomials are Pfcos 6) = ¥(3.c0s? 8-1) 165.08’ 0 — 30s 6) (35 cost 6 — 30.005" 8+ 3) and graph them against {=cos@. Ans. See Fig. 820. CHAP. 8} LAPLACE’S EQUATION 133, 8.39. Obtain E for Problem 8.18 and plot several values on Fig. 8-14. Note the orthogonality of E. and the cequipotential surfaces. Ans. E=—Voa{(cosh ax)(sin az)a, + (sinh ax)(cos az)a,] 840, Given V=V(cosharysinay), where a>0, determine the shape and location of the surfaces on 0 and V=Vo Make a sketch similar to Fig. 8-14. Ans. See Fig. 8-21 » v=o Fig. 8:22 4 LAPLACE'S EQUATION (cHar. 8 8.41, From the potential function of Problem 8.40, obtain E and plot several values on the sketch of the quipotental surfaces, Fig. 821. Ans. E=—Vel(sinh ar)sin aye, + (cosh (oo ] 8.42, Use a superposition of the product solutions found in Problem 8.17 to obtain the potential function for the semicircular strip shown in ig. 822. Ans. = 3 SOI Chapter 9 Ampére’s Law and the Magnetic Field 9. INTRODUCTION A. static magnetic field can originate from either a constant current or a permanent magnet. This chapter will treat the magnetic fields of constant currents. ‘Time-variable magnetic fields, which coexist with time-variable electric fields, will be examined in Chapters 12 and 13. 9.2 BIOT-SAVART LAW A differential magnetic field strength, dH, results from a differential current element Idi. ‘The field varies inversely with the distance squared, is independent of the surrounding medium, and has a direction given by the cross product of {dl and aj. ‘This relationship is known as the Biot-Savart law: 1dlX ap dH= Tae (Alm) The direction of R must be from the current element to the point at which dHH is to be determined, as shown in Fig. 9-1 Current elements have no separate existence. All elements making up the complete current filament contribute to Hand must be included. ‘The summation leads to the integral form of the Biot-Savart law: _ffaixae “PS axk™ A dosed line integral is required to ensure that all current elements are included (the contour may close at =). EXAMPLE 1. An infinitely long, straight, filamentary current 1 along the 7 axis in cylindrical coordinates is 135 136 AMPERE'S LAW AND THE MAGNETIC FIELD [cHa. 9 shown in Fig. 9-2. A point in the 2=0 plane is selected with no loss in generality, In differential form, dea, X(ra,~ 20,) ane tay de ray ere “The variable of integration is 2 Since a, docs not change with 2, it may be removed from the integrand before integrating. da= lf agae P al * Fig. 92 ‘This important result shows that H is inversely proportional to the radial distance. The direction is seen to be in agreement with the “right-hand rule” whereby the fingers of the right hand point in the direction of the field when the conductor is grasped such that the right thumb points in the direction of the current. EXAMPLE 2. An infinite current sheet lies in the 7=0 plane with K=Ka,, as shown in Fig. 9.3. Find H. ONG gn te Hal Fe 93 ‘The Biot—Savart law and considerations of symmetry show that H has only an x component, and is not a function of x or y. Applying Ampére’s law to the square contour 12341, and using the fact that H must be cur. 9] AMPERE'S LAW AND THE MAGNETIC FIELD 37 antisymmetric in z, fH Qa) +04 (4120) +0= (KI(20) oH ‘Thus, for all 2>0, H=(K/2)a,. More generally, for an arbitrary orientation of the current sheet, H=iKxa, Observe that His independent of the distance from the sheet. Further the directions of H above and below the sheet can be found by applying the right-hand rule to a few of the current elements in the sheet. 93 AMPERE’S LAW The line integral of the tangential component of the magnetic field strength around a closed path is ‘equal to the current enclosed by the path: fam ha At first glance one would think that the law is used to determine the current I by an integration, Instead, the current is usually known and the law provides a method of finding H. This is quite similar to the use of Gauss’ law to find D given the charge di In order to utilize Ampere’s law to determine Hi there must be a considerable degree of symmetry in the problem. Two conditions must be met: 1. At each point of the closed path H is either tangential or normal to the path. 2. Hhas the same value at all points of the path where H is tangential. ‘The Biot-Savart law can be used to aid in selecting a path which meets the above conditions. In most cases a proper path will be evident. BUNPLE. Use Angi’ oan Hd oil lag sigh net fet Th Bet-sart ow shws at ech pr of he i mi 92 tage sdf the same wea he fran neny=1 so that 94 CURL ‘The curl of a vector field A is another vector field. Point P in Fig. 9-4 lies in a plane area AS ‘bounded by a closed curve C. In the integration that defines the curl, C is traversed such that the 138 AMPERE'S LAW AND THE MAGNETIC FIELD ICHAP. 9 enclosed area is on the left. The unit normal a, , determined by a right-hand rule, is as shown in the figure. Then the component of the cutl of A in the direction a, is defined 2s faa as In the coordinate systems, curl A is completely specified by its components along the three unit vectors. For example, the x component in cartesian coordinates is defined by taking as the contour Ca square in the x=const. plane through P, as shown in Fig. 9-5. (curl A)+a, = Him A-dl (cul A)-a,= tim ay aro Ay AZ Fig 95 1. + A,a, +A,2, at the corner of AS closest to the origin (point 1), then pele feL =A 89+ (Ae Seay) as (a, + Mea )e-ay) +A,(-a0 = (2-28) ay as ay Oz 2A, aA, d : s ami (out A) a, = SS ‘The y and z components can be determined in a similar fashion. Combining the three components, aA, _ aA, curt A= (242 _ 242) url A F os A third-order determinant can be written, the expansion of which gives the cartesian curl of A. eye aaa a ay & JA, A, Ay the elements of the second row are the components of the del operator. This suggests (see Section 1.3) that VX A can be written for curl A. As with other expressions from vector analysis, this cua. 9} AMPERE’S LAW AND THE MAGNETIC FIELD 139 convenient notation is used for curl A in other coordinate systems, even though V is defined only in cartesian coordinates. Expressions for curl A in cylindrical and spherical coordinates can be derived in the same manner as above, though with more difficulty. (oylindrical) 2A, 8 se (spherical) Frequently useful are two properties of the curl operator: (1). The divergence of a curl is the zero scalar; that is, V-(VxA)=0 for any vector field A. (2) The curl of a gradient is the zero vector; that is, Vx (8A) =0 for any scalar function of position f (see Problem 9.20). Under static conditions, E=—VV, and so, from (2), VxE=0 9.§ RELATIONSHIP OF J AND H In view of Ampére’s law, the defining equation for (curl H), (see Section 9.4) may be rewritten as (curl H) a, = sttady && where J,=dl,/dS_ is the area density of x-directed current. ‘Thus the x components of curl Hl and the current denisty J are equal at any point. Similarly for the y and z components, so that VxH=3 ‘This is one of Maxwell's equations for static fields. If H is known throughout a region, then VX H will produce J for that region. EXAMPLE 4. A long, straight conductor cross section with redius a has a magnetic field strength H= Crf2na?)a,._ within the conductor (ra. Find J in both regions. Within the conductor, ad ‘which corresponds to a current of magnitude I in the +z direction which is distributed uniformly over the cross-sectional area xa’ 140 AMPERE’S LAW AND THE MAGNETIC FIELD (cHar. 9 9.6 MAGNETIC FLUX DENSITY B Like D, the magnetic field strength H depends only on (moving) charges and is independent of the medium. The force field associated with H is the magnetic flux density B, which is given by B=yH where = poi, is the permeability of the medium. The unit of B is the tesla, N ITI The free-space permeability uo has a numerical value of 4 x 10? and has the units henries per ‘meter, H/m; u, , the relative permeability of the medium, is a pure number very near to unity, except for a small group of ferromagnetic materials which will be treated in Chapter 11. ‘Magnetic flux, ®, through a surface is defined as o- [nas ‘The sign on ® may be positive or negative depending upon the choice of the surface normal in dS. The unit of magnetic flux is the weber, Wb. The various magnetic units are related by 1T=1Wo/m? = 1 H=1Wo/A EXAMPLE 5. Find the flux crossing the portion of the plane @=2/4 defined by 0.010, whence ‘Thus, for 7>0, Ke aa = Moe An BE ean Pe For z<0, change the sign of the above expression. 150 AMPERE'S LAW AND THE MAGNETIC FIELD [CHAP. 9 9.19. Using the vector magnetic potential found in Problem 9.18, find the magnetic flux crossing the rectangular area shown in Fig. 9-18. 0,0,2) (0,2,2) ©,0,1)} 0,2.) Fig. 9.18 Let the zero reference be at 22, so that He, A=-BG-2K fara ‘A‘is perpendicular to the contour on two sides and vanishes on the third (2=2). Thus, In the line integral = [Cae Ha —2 [ Kay=uok Note how the choice of zero reference simplified the computation. By Stokes’ theorem itis Vx A, ‘and not A itself, that determines ®; hence the zero reference may be chosen at pleasure. 9.20, Show that the curl of a gradient is zero. From the definition of curl A given in Section 9.4, itis seen that curl A is zero in a region if fara for every closed path in the region. But if A=Vf, where fis a single-valued function, fara a= fay =0 (s2e Section 5.6). Supplementary Problems 9.21. Show thatthe magnetic fed due to the fnitecurtent element shown in Fig. 9-19 is given by 1 H1= 7b Gin ay ~ sin iy 9.22, Obtain dHl at a general point (r, 8, $) in spherical coordinates, due to a differential current element / dl sete crnin de ponte zdreion an E2ESIO,, CHAP. 9] AMPERE'S LAW AND THE MAGNETIC FIELD 1st 923. 9.208, 925, 9.6, 921. 928. 929. 930, Fig. 9:19 ‘Currents in the inner and outer conductors of Fig. 9-20 ae uniformly distributed. Use Ampere's law to show that for b=r=c, enn =P) 1 nek Dar Mig. 9.20 ‘Two identical circular current loops of radius r=3m and 1=20A are in parallel planes, separated on their common axis by 10m. Find H at a point midway between the two oops. Ans. 0,908, A/m ‘A current filament of 10 in the +y direction lies along the y axis, and a current sheet, K= 20a, Alm, is located at_z=4m. Determine H at the point (2,2, 2). ‘Ans. 0398s, + 1.08, ~0.3988, A/m Show that the curl of (xa, +ya,+za)/G2+y?+2)? iszer0. (Hint: UXE: ) Given the general vector A=(—cosx)(cosy)a,, find the curl of A at the origin. Ans. 0 Given the general vector A.= (cosx)(sin ye, + (sinx)(cosy)a,, find the curl of A everywhere Ans. 0 Given the general vector A.=(sin26)a, in cylindrical coordinates, find the curl of A at (2, */4, 0). Ans. 0.54, Given the general vector A=e"*(sin}@)a, in cylindrical coordinates, find the curl of A at (0.800, 2/3, 0.500), Ans. 0.368a, + 0.2308, 152 93. 9.32, 933. 9.38. 936. 9.38. 9.38, AMPERE'S LAW AND THE MAGNETIC FIELD ICHAP. 9 Given the general vector A= (sin p)a, + (sin @)ay in spherical coordinates, find the curl of A at the point (2, 2/2,0). Ans. © Given the general vector A~2.50ay +5.00a, in spherical coordinates, find the curl of A at (2.0,2/6,0). Ans. 4.338, -2.50ay + 1.258, Given the general vector show that the curl of A is everywhere 2er0. ‘A cylindrical conductor of radius 10-*m has an internal magnetic field ma masele sci Wm ‘What is the total current in the conductor? Ans. 5.0. In cylindrical coordinates, 3=10°(cos'2r)a, in a certain region. Obtain H from this current density 1 sin r cos dr 1 and then take the curl of Hand compare with 3. Ans, H=10°(5+ S044 e041) In cartesian coordinates a constant current density, JJoa,, exists in the region -a0 Ans. W=} Joram, rsa “haa, 2<—a cul a3 Fig. 921 Compute the total magnetic flux © crossing the 2=0 plane in cylindrical coordinates for r= Sx10%m if int oa, CD) Ans, 3.16107 Wo Given that ‘a, (1) 3=2.50(in find the total magnetic flux crossing the strip 2=0, y=0, Osx<2m. Ans. 1LS9Wb ‘A coaxial conductor with an inner conductor of radius @ and an outer conductor of inner and outer radi CHAP. 9] AMPERES LAW AND THE MAGNETIC FIELD 153 940, 942, 943, 944, ban rope cri caren he er eda Fn magn ert en coun aylie pom tewentecaaon, Aw ie One uniform current sheet, K=Kes,, is at 2=b>2 and another, K=Ka(-s,), is at z= Find the magnetic flux crossing the area defined by x=const., -2 1074) ;, the force cannot be (into page) Fig. 101 EXAMPLE 2. For the particle of Example 1, find the radius of the circular path and the time required for one revolution. mU __ (1.70 x 10° (83.5 x 10°) i918” Cex x10) —O™ ne 77 a133, a us 154 CHAP. 10} FORCES AND TORQUES IN MAGNETIC FIELDS 155 10.2 ELECTRIC AND MAGNETIC FIELDS COMBINED ‘When both fields are present in a region at the same time, the force on a particle is given by F=Q(E+UxB) cig, This Lorentz force, together with the initial conditions, determines the path of the particle. 3 wauees 2 tera come argntig eof gerd 250%10°%.T m= 5.08, Vim. A proton (Q,~ 1.602 10""C, m, = 1.673 x 10-”" kg) enters the fields at the origin with an initial velocity U,=2.5%10', m/s. Describe the proton's motion and give its position after three complete revolutions. ‘The initial force on the particle is = Q(E+ UX B)=0,(Ea, — UBa,) ‘The z component (electric component) of the force is constant, and produces a constant acceleration in the z direction. Thus the equation of motion in the z direction is voted Op “The other (magnetic) component, which changes into —Q, Ba, , produces circular motion perpendicular to the 2 axis, with period After three revolutions, x=y=0 and v=} (QE )onr= 288-00 0,8" 10.3. MAGNETIC FORCE ON A CURRENT ELEMENT ‘A frequently encountered situation is that of a current-carrying conductor in an external magnetic field. Since 1=dQ/di, the differential force equation may be written 4F = dQ(U xB) = (dU xB) = (41 xB) where dl=Udt is the elementary length in the direction of the conventional current J. If the conductor is straight and the field is constant along it, the differential force may be integrated to give F=ILBsin@ ‘The magnetic force is actually exerted on the electrons that make up the current J. However, since the electrons are confined to the conductor, the force is effectively transferred to the heavy 156 FORCES AND TORQUES IN MAGNETIC FIELDS IcHaP. 10 lattice; this transferred force can do work on the conductor as a whole. While this fact provides a reasonable introduction to the behavior of current-carrying conductors in electric machines, certain essential considerations have been omitted. No mention was made, nor will be made in Section 10.4, of the current source and the energy that would be required to maintain a constant current I. Faraday’s law of induction (Section 12.3) was not applied. In electric machine theory the result ‘will be modified by these considerations. Conductors in motion in magnetic fields are treated again in Chapter 12; see particularly Problems 12.10 and 12.13. EXAMPLE 4, Find the force on a straight conductor of length 0.30m carrying a current of $.0A in the dizection, where the field is B=3.50x 10%a, —a,)T. F=(LXB) = (5.0}{(0.30)(—2,) «3.50 x 10%(a, —9,)] =ra2x10" ‘The force, of magnitude 7.42 mN, is at right angles to both the field B and the current direction, as shown in Fig. 103, ig. 103 104 WORK AND POWER ‘The magnetic forces on the charged particles and current-carrying conductors examined above result from the field. To counter these forces and establish equilibrium, equal and opposite forces, F, , would have to be applied. If motion occurs, the work done on the system by the outside agent applying the force is given by the integral w-[" Bea A positive result from the integration indicates that work was done by the agent on the system to ‘move the particles or conductor from the initial location to the final, against the field. Because the magnetic force, and hence F, , is generally nonconservative, the entire path of integration joining the initial and final locations of the conductor must be specified. EXAMPLE 5. Find the work and power required to move the conductor shown in Fig. 10-4 one full revolution in the direction shown in 0.02, if B=2.50x 10"'e,T and the current is 45.0 A. F=JQXB)=1.13% 10%, N ano FA -LSKI074N we fra =[[erissi0 arden wanes we P=wi= 0.07 CHAP. 10] FORCES AND TORQUES IN MAGNETIC FIELDS 157 i 7=063 m Fig. 104 “The negative sign means that work is done by the magnetic field in moving the conductor in the direction shown, For motion in the opposite direction, the reversed limits will provide the change of sign, and no attempt to place a sign on rd@a, should be made. 105 TORQUE ‘The moment of a force oF torque about a specified point is the cross product of the lever arm about that point and the force. The lever arm, r, is directed from the point about which the torque is to be obtained to the point of application of the force. In Fig. 10-5 the force at P has a torque about O given by T=xF where T has the units N-m. (The units N-m/rad have been suggested, in order to distinguish torque from energy.) Fig. 105 In Fig. 10-5, T lies along an axis (in the ay plane) through O. If P were joined to O by a tigid rod freely pivoted at O, then the applied force would tend to rotate P about that axis. ‘The torque T would then be said to be about the axis, rather than about point O. EXAMPLE 6. A conductor located at_x=0.4m, y=0 and 0 Ford>>a, ‘ Law Mn cen alin SE) Fig. 11-6. Cylindrical conductor parallel to a ground plane a —< Fig. 11-7. Long solenoid of small cross-sectional area S saNS “Sw 113 FARADAY'S LAW AND SELF-INDUCTANCE, Consider an open surface S bounded by a closed contour C. If the magnetic flux ¢ linking S varies with time, then a voltage v around C exists; by Faraday’s lav, ap at ‘As was shown in Chapter 5, the electrostatic potential or voltage, V, is well-defined in space and is 1m INDUCTANCE AND MAGNETIC CIRCUITS [cHap. 11 associated with a conservative electric field. By contrast, the induced voltage v given by Faraday's law is a multivalued function of position and is associated with a nonconservative field (dlectromotive force). More about this in Chapter 12. Faraday’s law holds in particular when the flux through a circuit clement is changing because the ‘current in that same element is changing: _dodi__ di di dt de In circuit theory, L is called the self-inductance of the clement and v is called the voltage of self-inductance or the back-voltage in the inductor. 4 INTERNAL INDUCTANCE Magnetic flux occurs within a conductor cross section as well as external to the conductor. This internal flux gives rise 10 an internal inductance, which is often small compared to the external inductance and frequently ignored. In Fig. 11-8(a) conductor of circular ctoss section is shown, with a current / assumed to be uniformly distributed over the area. (This assumption is valid only at low frequencies, since skin effect at higher frequencies forces the current to be concentrated at the outer surface.) Within the conductor of radius @, Ampére’s law gives Holt Taal te I H=>"50, and B Dna Co) Fig. 148 ‘The straight picce of conductor shown in Fig. 11-8(a) must be imagined as a short section of an infinite torus, as suggested in Fig. 11-8(b). The current filaments become circles of infinite radius. The lines of flux d® through the strip €dr encircle only those filaments whose distance from the conductor axis is smaller than r. Thus, an open surface bounded by one of those filaments is cut once (or an odd number of times) by the lines of db; whereas, for a filament such as 1 or 2, the surface is cut zero times (or an even number of times). It follows that d® links only with the fraction 2r?/a” of the total current, so that the total flux linkage is given by the weighted “su” af (Z)eo-[ Sa) reara et LAN bo Vy yg and Gr REA 5X 0 Hie CHAP. 11) INDUCTANCE AND MAGNETIC CIRCUITS 173 ‘This result is independent of the conductor radius. The total inductance is the sum of the external and internal inductances. If the external inductance is of the order of # x 10-7 H/m, the internal inductance should not be ignored. 1.§ MUTUAL INDUCTANCE In Fig. 11-9 a part $42 of the magnetic flux produced by the current i, through coil 1 links the Nz turns of coil 2. The voltage of mutual induction in coit 2 is given by doz v= NT? (negative sign omitted) KA % Fig. 11-9 In terms of the mutual inductance My = NeGulh do radiy yt vee Gg Ma Ge ‘This mutual inductance will be a product of the permeability 4 of the region between the coils and a geometrical length, just like inductance L. If the roles of coils 1 and 2 are reversed, diz Ue Ma ‘The following reciprocity relation can be established: Myz=Mzi EXAMPLE 4. A solenoid with N= 1000, 4=100m, and 6=S0cm_ is concentric within a second coil of M2000, r,=2.0em, and G=50cm. Find the mutual inductance assuming free-space conditions. For long coils of small cross section, H may be assumed constant inside the coil and zero for points just outside the coil. With the first coll carrying a current /;, (Alm) (athe axial direction) B= 12000h (Wo/m*) @=BA = (ug20001)(x x 10") (Wo) Since H and B are zero outside the coils, this is the only ux linking the second col Mua = My = (2000)(4x x 10°7)(2000)(x x 104) 1.6 MAGNETIC CIRCUITS In Chapter 9, magnetic field intensity Hf, ux ©, and magnetic flux density B were examined and various problems were solved where the medium was free space. For example, when Ampere’ 174 INDUCTANCE AND MAGNETIC CIRCUITS [cHaP. 11 is applied to the closed path C through the long, air-core coil shown in Fig. 11-10, the result is But since the flux lines are widely spread outside of the coil, B is small there. The flux is effectively restricted to the inside of the coil, where NI H=N é Ferromagnetic materials have relative permeabilities, in the order _ of thousands. Consequently, the flux density B=ou,H is, for a given H, much greater than would result in free space. In Fig. 11-11, the coil is not distributed over the iron core. Even so, the NI of the coil causes a flux ® which follows the core. It might be said that the flux prefers the ‘core to the surrounding space by a ratio of several thousand to one. This is so different from the free-space magnetics of Chapter 9 that an entire subject area, known as iron-core magnetics or ‘magnetic circuits, has developed. This brief introduction to the subject assumes that all of the flux is within the core. It is further assumed that the flux is uniformly distributed over the cross section of the core. Core lengths required for calculation of N/ drops are mean lengths. Fig. 14-1 Mig. tat 3.7 THE B-H CURVE A sample of ferromagnetic material could be tested by applying increasing values of Hf and ‘measuring the corresponding values of flux density B. Magnetization curves, or simply B-H curves, for some common ferromagnetic materials are given in Figs. 11-12 and 11-13. ‘The relative permeability can be computed from the B-H curve by use of i, = B/tigH. Figure 11-14 shows the extreme nonlinearity of 4, versus H for silicon steel. This nonlinearity requires that problems be solved graphically. 1L8 AMPERE’S LAW FOR MAGNETIC CIRCUITS A coil of N turns and current / around a ferromagnetic core produces a magnetomotive force (mmf) given by NI. The symbol F is sometimes used for this mmf; the units are amperes or ampere tums. Ampére’s law, applied around the path in the center of the core shown in Fig. 11-15(a), gives Fon $Hea faas[aas faa = HG + G+ 6 enon Fig. 1-12, Bat! curves, H<400 Alm eae Fig. 11-13, B-H carves, H > 400A/m 176 INDUCTANCE AND MAGNETIC CIRCUITS (CHAP. 11 HA) Fig. 11-14 Wh Ry ~ a Ry Fi ie rent ©) 2a. @ @ © Fig. 11-15 ‘Comparison with Kirchhoff’s law around a single closed loop with three resistors and an emf V, VaVrKty ‘suggests that F can be viewed as an NI rise and the Hé terms considered NI drops, in analogy to the voltage rise V and voltage drops V, , Vz and Vj. The analogy is developed in Fig. 11-15(6) and (©). Flux © in Fig. 11-15(6) is analogous to current 1, and reluctance { is analogous to resistance R. An expression for reluctance can be developed as follows, é Nt drop= He = Bal oa hence an ary If the reluctances are known, then the equation Fa N= OR, +R +H) can be written for the magnetic circuit of Fig. 11-15(b). However, 1, must be known for each CHAP. 11] INDUCTANCE AND MAGNETIC CIRCUITS 7 ‘material before its reluctance can be calculated. And only after B or # is known will the value of 4, be known. This is in contrast to the relation é ROTA (Section 6.7), in which the conductivity o is independent of the current. 11.9 CORES WITH AIR GAPS Magnetic circuits with small air gaps are very common. ‘The gaps are generally kept as small as possible, since the N/ drop of the air gap is often much greater than the drop in the core. The flux fringes outward at the gap, so that the area at the gap exceeds the area of the adjacent core. Provided that the gap length ¢, is less than % the smaller dimension of the core, an apparent area, S,,, of the air gap can be calculated. For a rectangular core of dimensions a and b, S.= (a+ G)(b + 6) If the total flux in the air gap is known, H, and H,¢, can be computed directly. For a uniform iron core of length ¢, with a single air gap, Ampére’s law reads If the flux ® is known, it is not difficult to compute the NI drop across the air gap, obtain B, , take H, {from the appropriate B-H curve and compute the NI drop in the core, Hi¢;.. The sum is the NY required to establish the flux. However, with NJ given, it is a matter of trial and error to obtain B, and ®, as will be seen in the problems. Graphical methods of solution are also available. 1.10 MULTIPLE COILS ‘Two or more coils on a core could be wound such that their mmfs cither aid one another or ‘oppose. Consequently, a method of indicating polarity is given in Fig. 11-16. An assumed direction for the resulting flux © could be incorrect, just as an assumed current in a de circuit with two or more voltage sources may be incorrect. A negative result simply means that the flux is in the ‘opposite direction. “nO @ ® Mig. 11-16 178 INDUCTANCE AND MAGNETIC CIRCUITS [onar. 11 11,11 PARALLEL MAGNETIC CIRCUITS ‘The method of solving a parallel magnetic circuit is suggested by the two-loop equivalent circuit shown in Fig. 11-17(6). The leg on the left contains an NI rise and an Ni drop. ‘The NI drop between the junctions @ and b can be written for each leg as follows: F-HG=HiG= Hl and the fluxes satisfy = 0,40; Different materials for the core parts will necessitate working with several B-H curves. An air gap in one of the legs would lead to Fi¢, + Ha, for the mmf between the junctions for that leg. ih Ie, FOL algtme 3 fas ® O) ig. 1-17 ‘The equivalent magnetic circuit should be drawn for paralle! magnetic circuit problems. It is good practice to mark the material types, cross-sectional areas, and mean lengths directly on the diagram. In more complex problems a scheme like Table 1i- can be helpful. The data are inserted directly into the table, and the remaining quantities are then calculated or taken from the appropriate B-H curve. Part | Material | Area | ¢ [@ |e |x | He Solved Problems Wl, Find the inductance per unit length of the coaxial cable in Fig. 11-2 if a a 1mm and 6=3mm. Assume y,=1 and omit internal inductance. Ea Hig? Ax" 9320.20 HHI Coin a Oe CHAP. 11] INDUCTANCE AND MAGNETIC CIRCUITS 179 2. & 13. a 4 & ns. 116. Find the inductance per unit length of the parallel cylindrical conductors shown in Fig. 11-5, where d=25ft, @=0.803in, L. 25(12) EP cosh fo (4 x 10-7) cosh = Im Fa Bont" = (4% 10") conh-"o =2.37 WH ‘The approximate formula gives L torn’, Gg tg 2a HH When d/a®10, the approximate formula may be used with an error of less than 0.5%. A cireular conductor with the same radius as in Problem 11.2 is 12.5ft from an infinite ‘conducting plane. Find the inductance. 18 xH/m. ‘This result is $ that of Problem 11.2. A conducting plane may be inserted midway between the two ‘conductors of Fig. 11-5. The inductance between each conductor and the plane is 1.18 uH/m, Since ‘they are in series, the total inductance is the sum, 2.37 H/m. ‘Assume that the air-core toroid shown in Fig. 11-4 has a circular cross section of radius 4mm. Find the inductance if there are 2500 turns and the mean radius is r = 20mm. NES _ (x x 10°7)(2500}2(0,004) 2n(0.020) L= =3.14mH ‘Assume that the air-core toroid in Fig. 11-3 has 700 turns, an inner radius of lcm, an outer radius of 2cm, and height a=1Scm. Find L using (a) the formula for square cross-section toroids; (6) the approximate formula for a general toroid, which assumes uniform H at a mean radius. « X10) FOOHO.O13) 5 5 @ a x In2= 1.02 mH oN _ (dre x 10-7(700)°0.01)(0.015) _ o oe 2n(0.015) 098 H With a radius that is larger compared to the cross section, the two formulas yield the same result. See Problem 11.26. Use the energy integral to find the internal inductance per unit length of a cylindrical conductor of radius a. Atadistance rsa from the conductor axis, whence “The inductance corresponding to energy storage within a length ¢ of the conductor is then = [ERMA te Cramear alt oT ber Pant ar = et ara or L/é=yol8x. This agrees with the result of Section 11.4, 180 INDUCTANCE AND MAGNETIC CIRCUITS [cHAP. 11 1.7, The cast iron core shown in Fig. 11-18 has an inner radius of 7em and an outer radius of Sem. Find the flux & if the coil mmf is 500 A. €= 21(0.08) = 0.508 m Ha Fp gp AIO From the B-H curve for cast iron in Fig. 11-13, B= 0.40. © = BS = (0.40X0.02)" = 0.16 mW Mt Fig. 118 11.8. ‘The magnetic circuit shown in Fig. 11-19 has a C-shaped cast steel part, 1, and a cast iron ig part, 2. Find the current required in the 150-turn coil if the flux density in the cast iron S& is 2.-045T. [eatzem—e} fe tsem Fig. 119 The calculated areas are S)=4%10"'m? and $;=3.6% 10m’, ‘The mean lengths are G=0.11+ 0.11 0.12=0.34m, 6 =0112 + 0,009 + 0.009 = 0.138 m From the B-H curve for cast iron in Fig. 11-13, H,= 1270 A/m. 1S: = (0.45)(3.6 X10 *)= 62x 10* Wo aT ‘Then, from the cast steel curve in Fig. 11-12, #,=233 A/m. ‘The equivalent circuit, Fig. 11-20, suggests the equation WG + Hab 33(0.34) + 1270(0.138) 0A Mt Fig. 11-20 CHAP. n9. 11.10. wu. ay INDUCTANCE AND MAGNETIC CIRCUITS 181 ‘The magnetic circuit shown in Fig. 11-21 is cast iron with a mean length ¢=0.44m and square cross section 0.02 x 0.02m. The air-gap length is ¢,=2mm and the coil contains 400 turns. Find the current / required to establish an ait-gap flux of 0.141 mWb. Fig. 1-21 “The Bux @ in the air gap is also the fx in the core. @_0.141 x10" 57 4x10 From Fig. 11-13, H,=850A/m, ‘Then Hie, ~ 850(0.44) = 374. For the air gap, S, = (0.02-+0.002)' = 4.84 10-*m e 0.141 x 107 Flom = ex io yasex 10> Therefore, F=Hi6,+H,6,=838A and Be 35T 'm’, and so X10") = 464 Determine the reluctance of an air gap in a de machine where the apparent area is S,=4.26x10-?m? and the gap length 5.6 mm. a= =___ 56x10? _ HS. GR X10 VA25X IT) 1.05% 10H"? ‘The cast iron magnetic core shown in Fig. 11-22 has an area S,=4cm? anda mean length 0.438m, ‘The 2-mm air gap has an apparent area S,=4.84cm?. Determine the ait-gap fiux ©. Fig. 11-22 ‘The core is quite long compared to the length of the sir gap, and cast iron isnot a particularly {00d magnetic material. As.a first estimate, therefore, assume that 600 ofthe total ampete turns are Aropped at the ar gap, i.e, H,é,= 600 A. © Mens. p= SAME 10-Y64.84 x 10°) 2x07 1.82 10 Wb 182, 11.12, 13. INDUCTANCE AND MAGNETIC CIRCUITS [chap. 11 Then B,~@/S=0.46T, and from Fig. 11-13, H,= 1340 A/m. The core drop is then 6, = 1340(0.438) = 587 A, so that His + Hel = 1187 A This sum exceeds the 1000A mmf of the coil. Consequently, values of B, lower than 0.46T. should be, ied until the sum of Fé, and HE, is 100A. The values B,=041T and b= 1.64% 10" Wb will esule in a sum very close to 1000 A. Solve Problem 11.11 using reluctances and the equivalent magnetic circuit, Fig. 11-23. Fig. 11-23, From the values of B, and H, obtained in Problem 11.11, 3.83 x 10" H/m, ‘Then, for the core, ‘ oss os R= aS BBE VERIO AOI and forthe sr xp. 2x10? - ROS” GRR MARIO AEH The crit equation, F= (RR) gives o 1000) 63 x 10 Wo “F868 x1F + 3.29 x10 ‘The corresponding flux density in the iron is 0.41, in agreement with the results of Problem LIU. While the air-gap reluctance can be calculated from the dimensions and jie, the same is not ‘true for the reluctance of the iron. The reason is that 1, for the iron depends on the values of B, and 4, Solve Problem 11.11 graphically with a plot of © versus F. Values of H, from 700 through 1100 A/m are listed in the first column of Table 11-2; the corresponding values of B, are found from the cast iton curve, Fig. 11-13, ‘The values of © and Hé are computed, and H,é, is obtained from ®é/yicS,. ‘Then F is given as the sum of H,é, and Hil. Since the air gap is linear, only two points are required. The flux @ for F=1000A. is seen from Fig. 11-24 to be approximately 1.65 x 10™* WD. ‘This method is simply a plot of the trial and error data used in Problem 11.11. However, it is helpful if several different coils or coil currents are to be examined. CHAP. 11} INDUCTANCE AND MAGNETIC CIRCUITS 183 1.14, Table 12 Bt) | oon) | Hea) | HEA) | FAD 0295 | 118x10" | 307 ae | ws 0335 | 134x10* | 350 a | at 0365 | 146x10" | 395 40 |e 0400 | 10x10 | 38 326 | (868 oa | usexio~ | 42 | ssz | 10m owe x10) Fa) Fig. 1-24 Determine the fluxes in the core of Problem 11.11 for coil mmfs of 800 and 1200 A. Use ‘graphical approach and the negative air-gap line. ‘The @ versus H,é, data for the cast iron core, developed in Problem 11.13, are plotted in Fig. 11-25. The air-gap ® versus Fis linear. One end of the negative air-gap line for the coll mmf of BOUA is at @=0, F=800A. The other end assumes H,,=800A, from which HeS.( 2.43 x10 Wo which locates this end at ®=2.43%10""Wb, F-=0. ‘The intersection of the F=800 A negative ait-gap line with the nonlinear © versus F curve for the cast iron core gives @= 1.34% 10~*Wb. Other negative air-gap lines have the same negative slope. For a coil mmf of 100A, @=1.63x10*Wb and for 120A, =1.85x 10" Wb, Solve Problem 11.11 for a coil mmf of 1000 A using the B-H curve for cast iron. ‘This method avoids the construction of an additional curve such as the & versus F curves of Problems 11.13 and 11.14. Now, in order to plot the airgap line on the B-H curve of iron, adjustments must be made for the different areas and the different lengths. Table 11-3 suggests the necessary calculations. 1000 Fra 2 Alm INDUCTANCE AND MAGNETIC CIRCUITS [cHar. 11 by Zu © sel o4 Nerve andi ine ° 1; eet BT) | He(A/m) 0.10 | 0.80%10° 030 | 2.39x10° [030 _[3sexi0" 0.61 1817 466 ‘The data from the third and fifth columns may be plotted directly on the cast iron B-H curve, as shown in Fig. 11-26. The air gap is linear and only two points are needed. ‘The answer is seen to be B,=0.41T. The method can be used with two nonlinear core parts, as well (see Problem 11.16). aa H(Alen) Fig. 11-26 CHAP. 11] INDUCTANCE AND MAGNETIC CIRCUITS 185 11.16. ‘The magnetic circuit shown in Fig. 11-27 consists of nickel-iron alloy in part 1, sig where @=1Wcm and S;=225em*, and cast steel for part 2, where 8cm and S;=3cm?. Find the flux densities B, and By . Fig. 11-27 ‘The data for part 2 of cast steel will be converted and plotted on the B-H curve for part 1 of ickeliron alloy (F/¢,= 400 A/m). Table 11-4 suggests the necessary calculations. Table 114 6 2,0) (2) cam 03 16 0 oat | 250 039 200 200 | oss | 300 a7 20 16 cas | 350 0m 70 0 on | «0 097 x0 ” o7a_| aso 104 xa 0 cas [500 un 00 ° From the graph, Fig. 11-28, B, -O1T. Then, since B.S, asx) 3x07) ‘These values can be checked by obtaining the corresponding H, and H, from the appropriate B-H ‘curves and substituting in 167 PaHG+ Hie 11.17, The cast steel parallel magnetic circuit in Fig. 11-29(a) has a coil with 500 turns. ‘The mean a lengths are &=6=10em, ¢=4em. Find the coil current if ,=0.173 mWb. 3 = 0,40, Since the cross-sectional area of the center leg is twice that of the two side legs, the flux density is the ‘same throughout the core, ie., 17310 * 3x10 * Corresponding to B=115T, Fig. 11-13 gives H=1030A/m. The NI drop between points a 186 INDUCTANCE AND MAGNETIC CIRCUITS (char. 11 an) Cet Stee, ‘Adjusted Fig, 11.29 and b is now used to write the following equation [see Fig. 11-25(b)]: F-HQ=HG=H6 or F=H(é,+6)=1030(0.14) = 144.20 F_uaz ‘Then apa eH ‘The same cast stect core as in Problem 11.17 has identical 500-turn coils on the outer legs, with the winding sense as shown in Fig. 11-30(a). If again ©,=0.173mWb, find the coil currents, ‘The flux densities are the same throughout the core and consequently H is the same. The equivalent circuit in Fig, 11-30(b) suggests that the problem can be solved on a per pole bass a Be st and H'=1090A/m (from Fig. 11-13) =H (4+ 6)=10800.14)= 142A 10.298 Each coil must have a current of 0.29 A. CHAP. 11] INDUCTANCE AND MAGNETIC CIRCUITS 187 He, @ o Fig. 11-30 11.19, The parallel magnetic circuit shown in Fig. 11-31(a) is silicon steel with the same cross-sectional area throughout, $=1.30cm?. The mean lengths are & 25cm, &=Scm. The coils have SO tums each. Given that ,=904Wb and 120 uWb, find the coil currents. ©, = © ~ , = 0.30% 10" Wo 90x10 13010 a Mig. 1131 From Fig. 11-12, H,=87A/m. Then, H,é,=21.8A. Similarly, B,=0.23T, H,=49A/m, HiG=25A; and By=092T, Hy=140 Alm, Hy6=350A. The equivalent ‘cicuit in Fig, 11-31(b) suggests the following equations for the N/ drop between points a and b: HG F= HG = B-Ab 28-R=25 from which K=19.3A and K=375A. The currents are J 3A and = 075A. Obtain the equivalent magnetic circuit for Problem 11.19 using reluctances for three legs, and calculate the flux in the core using F=19.3A and K=375A. ‘ an aS From the values of B and H found in Problem 11.19, Helin =7.93 10H Halt = 4.69 10° fem pet = 6.57 10" [Now the reluctances are calculated: = 2.43% 10H 188 INDUCTANCE AND MAGNETIC CIRCUITS [cHar. 11 %,=8.20% 10H, = 293% 10! From Fig. 1-32, K=O9+ OR, o f= OH,- 9,2, ® %149,=o) ® Pig. 11:32 Substituting ®, from (3) into (1) and (2) results in the following set of simultaneous equations in and 5: = OR + H)~ 2% or 93% (8.25% 10°) —8,(0.82 x 10) B=-OR + OLR, +H) 31.5 = ~,(0.82 10°) + 03.75 x 10") Solving, $,=89.7 hWo, ©;=303uWb, ®,=120"Wo. ‘Although the simultaneous equations above and the similarity to a two-mesh circuit problem may ‘be interesting, it should be noted that the flax densities B,, Bs, and B, had to be known before the relative permeabilities and reluctances could be computed.” But if B is known, why not find the flux directly from = BS? Reluctance is simply not of much help in solving problems of this type. Supplementary Problems 11.21, Find the inductance per unit length of a coaxial conductor with an inner radius @=2mm and an ‘outer conductor at b=9mm. Assume y= 1. Ans. 0.301 yH/m 11.22, Find the inductance per unit length of two parallel cylindrical conductors, where the conductor radius is mm and the center-to-center separation is 12mm. Ans. 0.992 uH/m 11.23, Two parallel cylindrical conductors separated by 1m have an inductance per unit length of 2.12uH/m. What is the conductor radius? Ans. Smm 11.24, An air-core solenoid with 2500 evenly spaced turns has a length of 1.5m and radius 2% 10m. Find the inductance L. Ans. 6.58mH 11.25. A square-cross-section, air-core toroid such as that in Fig. 11-3 has inner radius Sem, outer radius 7em ‘and height 1.5cm. Ifthe inductance is 495 yH, how many turns are there in the toroid? Examine the approximate formula and compare the result. Ans. 700, 704 11.26. A square-cross-section toroid such as that in Fig. 11-3 has =80em, m=8em, a=1Sem, and 700 turns. Find L using both formulas and compare the results, (See Problem 11.5.) ‘Ans, 36.3 1H (both formulas) AL27. A coil with 5000 turns, 7, =1.25em, and 4=1.0m_ has a core with p,=50. A second coil of 500 tums, r;=2.0¢m, and &=100cm is concentric with the first coil, and in the space between the coils 6 jig. Find the mutual inductance. Ans. 7.71 mH 11.28. Determine the relative permeabilities of cast iron, cast steel, silicon steel, and nickeliron alloy at a flux density of 0.4. Use Figs. 11-12 and 11-13. ‘Ans. 318, 1384, $305, 42,440 CHAP. 1] INDUCTANCE AND MAGNETIC CIRCUITS 189 11.29. An air gap of length ¢=2mm has flux density of 0.4. Determine the length of a magnetic core ‘with the same NI drop if the core is of (a) cast iron, (b) cast steel, (c) silicon steel. Ans. (a) 0.64 eny, (6) 2.77 mi; (c) 10.6m 11.30, A magnetic circuit consists of two parts of the same ferromagnetic material (4, = 4000). Part 1 50mm, S,= 106mm"; part 2has &=30mm, S,= 120mm’. The material is at a part ‘of the curve where the relative permeability is proportional to the flux density. Find the flux & if the mmfis4.0A. Ans. 26.3 4Wb LBL. A toroid with a circular cross section of radius 20mm has a mean length 280mm and a flux @= 1.50mWb. Find the required mmf ifthe core is silicon steel. Ans. 83.2 11.32, Both parts of the magnetic circuit in Fig. 11-33 are cast stecl. Part I has &=34em and S\= Gem? part Zhas €=16cm and S)=4em*, Determine the coil current 4, if L=O5A, Ni, 200 turns, N= 100 turns, and ©=120nWb. Ans. 0.65A Fig. 11-33, 11.33. ‘The silicon steel core shown in Fig. 11-34 has a rectangular cross section 10mm by 8mm and a mean Tength 150mm. The airgap length is 0.8mm and the airgap flux is 80uWb. Find the mmf, Ans. S6LZA Fig. 1134 11.34, Solve Problem 11.33 in reverse: the coil mmf is known to be 61.2 and the air-gap flux is to be determined. Use the trial and error method, starting with the assumption that 90% of the NI drop is ‘across the air gap. 11.35, ‘The silicon steel magnetic circuit of Problem 11.33 has an mmf of 600A. Determine the airgap flux. Ans. 85.2uWb 11.36, For the silicon stee! magnetic circuit of Problem 11.33, calculate the reluctance of the iron, %,, and the reluctance of the air gap, R,. Assume the flux @=804Wb and solve for F. See Fig. 11- 35. Ans. R=O33pH', R,=6.70yH"', F=S61A +5 * rT ¢ ig. 11-35 190 4.37, 11.38, 1.38. a0. mat. 42, 183. as, 14s, 11.46, INDUCTANCE AND MAGNETIC CIRCUITS [cHaP. 11 A silicon steel core such as shown in Fig. 11-34 has a rectangular cross section of area S,= 8mm? and an air gap of length ¢,=0.8mm with area S.=9Smm'. The mean length of the core is 150mm and the mmf is 600A. Solve graphically for the flux by plotting © versus F in the ‘manner of Problem 11.13. Ans. 85 zWb Solve Problem 11.37 graphically using the negative air-gap line for an mmf of 600A. Ans. 85 Wb Solve Problem 11.37 graphically in the manner of Problem 11.15, obtaining the flux density in the core. Ans. 1.06T A rectangular ferromagnetic core 40 60mm has a flux @=1.44mWb. An air gap in the core is of length ¢=2.5mm. Find the NI drop across the air gap. Ans. 1079. ‘A toroid with cross section of radius 2.em has a silicon steel core of mean length 28 em and an air gap of length 1mm. Assume the air-gap area, S,, is 10% greater than the adjacent core and find the mmf required to establish an air-gap flux of 1SmWb. Ans. 952. ‘The magnetic circuit shown in Fig. 11-36 has an mmf of SOOA. Part Lis cast steel with = 340mm and S)= 400mm; part 2is cast iron with &= 138mm and S,= 360mm. Determine thefiux®. Ans. 229 xWo Big. 11.36 Solve Problem 11.42 graphically in the manner of Problem 11.16. Ans. 229 mWb A toroid of square cross section, with )=2cm, r=3cm, and height a=1em, has a two-part core. Part 1 is silicon steel of mean length 79cm; part 2 is nickeliron alloy of mean length 7.9 cm. Find the flux that results from an mmf of 17.38A. Ans. 10 Wo Solve Problem 11.44 by the graphical method of Problem 11.15. Why is it that the plotting of the second reverse B-H curve on the firs is not as difficult as might be expected? ‘Ans, 10'Wb. ‘The mean lengths and cross-sectional areas are the same “The cast stecl parallel magnetic circuit in Fig, 11-37 has a $00.turn collin the center leg, where the cross-sectional area is twice that of the remainder of the core. The dimensions are ¢,=Imm, S,= S.=150mm', $,=300me%, &=40mm, &= 10mm, and 4= 109mm. Find the coil current requited to produce an at-gap flux of 125 4Wb. Assume that S, exceeds S, by 17%. Ans. LMA CHAP. 11] INDUCTANCE AND MAGNETIC CIRCUITS m1 11,47, ‘The cast iron parallel circuit core in Fig. 11-38 has 2 $00-turn coil and a uniform cross section of 1.5 cm? throughout. The mean lengths are 4=6=10cm and &=4em. Determine the coil current necessary to result in a flux dersity of 0.25T in leg 3. Ans. 1.OSA Q 1148, Two identical $00-turn coils have equal currents and are wound as indicated in Fig. 11-39. The cast steel core has a flux in leg 3 of 120yWb. Determine the coil currents and the flux in leg 1. Ans, 0.41.4, 0Wb “= 0.01 m 6 00 0.06 || 5 02 fe ® * or oof 6a 0.14» Fig. 1139 149, ‘Two identical coils are wound as indicated in Fig. 11-40. The silicon steel core has a cross section of ‘6cm? throughout. The mean lengths are I4cm and G=4em. Find the coil mmfs if the fluxin leg Lis O.7mWb. Ans. 385A et 2Rt- Chapter 12 Displacement Current and Induced EMF 12.1 DISPLACEMENT CURRENT In static fields the curl of H was found to be pointwise equal to the current density J... This is conduction current density; the subscript ¢ has been added to emphasize that moving charges— ‘electrons, photons, or ions—compose the current. If VXH=J. were valid where the fields and charges are variable with time, then the continuity equation would be ¥+J,=V-+(¥XH) instead of the correct ved. Hence, James Clerk Maxwell postulated that aD J+ 3 whe DS where = With the inclusion of the displacement current density Jp, the continuity equation is satisfied: @D_ 2 ap v-2--S-p)--2 ‘The displacement current ip through a specified surface is obtained by integration of the normal ‘component of Jp over the surface (just as i. is obtained from 3,). aD a4 [Fs-5 [o-s YxH=, V-E=-VIp Here, the last expression assumes that the surface S is fixed in space. EXAMPLE 1. Use Stokes’ theorem (Section 9.8) to show that i, Since the two surfaces S, and S; have the common contour C, init the circuit of Fig. 12-1 fu-a- [oxm-as=[ coxty-as -[(eeBpa-[e® ig. 121 192 CHAP. 12] DISPLACEMENT CURRENT AND INDUCED EMF 193 ‘Assuming the flux is confined to the dielectric between the conducting plates, D=0 over S;. And since no, free charges are in motion within the dielectric, J,=0 over S.. Therefore, aw sods [Pots or nin f J, or Iesould be noted that 20/21 is nonzero only over that part ofS; that ies within te dees EXAMPLE 2. Repeat Example 1, this time using circuit analysis. Refer to Fig. 12-1. The capacitance of the capacitor is where A is the plate area and d is the separation. The conduction current is then (On the other hand, the electric field in the dielectric is, neglecting fringing, E=w/d. Hence aD_edv and the displacement current is (D is normal to the plates) 12.2 RATIO of J. TO Jp Some materials are neither good conductors nor perfect dielectrics, so that both conduction ‘current and displacement current exist. A model for the poor conductor or lossy dielectric is shown in Fig. 12-2. Assuming the time dependence e for E. the total current density is a LK + Ip = 08 +5 (CE) = oF + jock from which Ale In we As expected, the displacement current becomes increasingly important as the frequency increases. Fig. 122 EXAMPLE 3. A circularcrosssection conductor of radius 1.Smm carries a eurtent i, =5.Ssin (4% 10%) (WA). What is the amplitude ofthe displacement current density, if 0=35MS/m and ¢,=1? ko 30x10 Jp we” @RIO\I0 "P36 (5.5 10° /fx(15x 10) 9.90 « 107 9.90% 10" Then Jo 7.86 «10-" w/e? 194 DISPLACEMENT CURRENT AND INDUCED EMF tcHap. 12 12.3 FARADAY'S LAW AND LENZ’S LAW The minus sign in Faraday’s law (Section 11.3) implicitly gives the polarity of the induced voltage v. To make this explicit, consider the case of a plane area S, bounded by a closed curve C, where $ is cut perpendicularly by a time-variable flux density b (Fig. 12-3). Faraday's law here takes the integral form 4 fra Leas in which the positive sense around C and the direction of the normal, dS, are corrected by the usual right-hand rule (Fig. 12-3(a)]. Now if B is increasing with time, the time derivative will be positive and, thus, the right side of the above equation will be negative. In order for the left integral to be negative, the direction of E must be opposite to that of the contour, Fig. 12-3(6). A conducting filament in place of the contour would carry a current i, also in the direction of E. As shown in Fig. 12-3(c), such a current loop generates a flux ¢” which opposes the increase in B. Lenz's law summarizes this discussion: the volkage induced by a changing flux has a polarity such that the current established in a closed path gives rise to a flux which opposes the change in fl. « ae 7 In the special case of a conductor moving through a time-independent magnetic field, the polarity predicted by Len2’s law is yielded by two other methods. (1) The polarity is such that the conductor experiences magnetic forces which oppose its motion. (2) As indicated in Fig. 12-4, a moving conductor appears to distort the flux, pushing the flux lines in front of it as it moves. This same distortion is suggested by the counterclockwise flux lines shown around the conductor. By the right-hand rule the current which would result if a closed path were provided would have the direction shown, and the polarity of the induced voltage is + at the end of the conductor where the current would leave. Figure 12-5 confirms this by comparing the moving conductor and its resulting current to a voltage source connected to a similar external circuit. Hap. 12] DISPLACEMENT CURRENT AND INDUCED EMF 195 Le XS reus 12.4 CONDUCTORS IN MOTION THROUGH TIME-INDEPENDENT FIELDS ‘The force F on a charge Q in 2 magnetic field B, where the charge is moving with velocity U, was examined in Chapter 10. F=Q(UxB) ‘A motional electric field intensity, Em, can be defined as the force per unit charge: F 2 ‘When a conductor with a great number of free charges moves through a field B, the impressed E,, creates a voltage difference between the two ends of the conductor, the magnitude of which depends on how E,, is oriented with respect to the conductor. With conductor ends a and 6, the voltage of a with respect to b is =UXB v= [ER -at= [(UxB)-al If the velocity U and the field B are at right angles, and the conductor is normal to both, then a conductor of length ¢ will have a voltage v=Beu For a closed loop the line integral must be taken around the entire loop: v=fuxE)-a Of course, if only part of the complete loop is in motion, it is necessary only that the integral cover this part, since E,, will be zero elsewhere. EXAMPLE 4. In Fig. 12-6, two conducting bars move outward with velocities U,=12.5(—%) m/s and U;=8.0a,m/s_in the field B=0.3Se,T. Find the voltage of b with respect toc. At the two conductors, EU, XB=4.38(-9,) Vim E25 U:XB=2808, Vim 196 DISPLACEMENT CURRENT AND INDUCED EMF [cHae. 12 and so i Ue= Yan + Yas # Yar = 219-4 0+ 1.40 = 3.59,V ‘Since b is positive with respect to ¢, current through the meter will be in the a, direction. This clockwise ‘current in the circuit gives tise to flux in the —a, direction, which, in accordance with Lenz’s law, counters the increase in the flux in the +a, direction due to the expansion of the circuit. Moreover, the forces that B exerts ‘on the moving conductors are directed opposite to their velocities. 125 CONDUCTORS IN MOTION THROUGH TIME-DEPENDENT FIELDS ‘When a closed conducting loop is in motion (this includes changes in shape) and also the field B is @ function of time (as well as of position), then the total induced voltage is made up of a contribution from each of the two sources of flux change. Faraday's law becomes 2B Is Oe ‘The first term on the right is the voltage due to the change in B, with the loop held fixed; the second term is the voltage arising from the motion of the loop, with B held fixed. The polarity of each term is found from the appropriate form of Lenz's law, and the two terms are then added with regard to those polarities. vet ras=—[ as fcuxm)-a EXAMPLE 5. As shown in Fig. 12-7a), a planar conducting loop rotates with angular velocity o about the x axis; at_£=0 it is in the xy plane. A time-varying magnetic field, B= B(0)a,, is present. Find the voltage induced in the loop by using the two-term form of Faraday's law. ~ @ Mg. 127 Let the area of the loop be A. The contribution to u due to the variation of B is ae, v=-[ Be as--[F since a, +a, = c0s a ‘To calculate the second, motional contribution to v, the velocity U of a point on the loop is needed. From Fig. 12-7(6) it is seen that y Usroe,=—— on, ‘08 ot so that a cos ov om Xa, = 2 08 sn-n) CHAP. 12] DISPLACEMENT CURRENT AND INDUCED EMF 197 since», 2, =sin ox(-a,). Consequently, @Bsinor "005 a a=fOxm-a frnca ‘Stokes’ theorem (Section 9.8) can be used to evaluate the last integral. Since VX ya, fra-ai= [ coxyny-as= [¢ a ye 28SA pet) Bhesinat Solved Problems 2A. In a material for which o=5.0S/m and ¢,=1 the electric field intensity is E= 250sin 10"t (V/m). Find the conduction and displacement current densities, and the frequency at which they have equal magnitudes. J. a = 1250sin 10% (A/m*) ‘On the assumption that the feld direction does not vary with time, (€o6,250 sin 10%) = 22, 160810" (A/m*) For d= 30 Bax ITF which is equivalent to a frequency f= 8,99 10" Hz = 89.9 GH, 165 10" rad/s o=we oF 12.2. A coaxial capacitor with inner radius $ mm, outer radius 6mm and length 500mm has a electric for which c,=67 and an applied volage 250sin371(V). Determine the 2 displacement current ip and compare with the conduction current i. ‘Assume the inner conductor to be at v=0. Then, from Problem 87, the potential at 0.005 = 1=0006m is From ths, AXA 37%, (Vim) pacts SE asm, (rn) D__MOX10" sm (alm) a 7 Jof2arL) =9.63% 10-*e08 37% (A) ‘The circuit analysis method for i: requires the capacitance, eek in) c = 1,02nF 198 4. ns. DISPLACEMENT CURRENT AND INDUCED EMF [cHap. 12 ‘Then b= CH= (.02 x 10-")250,377( 605371) = 9.63 10°*e08377 (A) Ieis seen thet {=n Moist soil has a conductivity of 10~*S/m and €,=2.5. Find J, and Jp where E=6.0% 10~*sin9.0 10% (V/m) First, J, = 06 = 6.0% 10-*sin9.0x 10% (A/m'). Then, since D=€¢,E, aE Jo~ DB - coe: 20x 10°* 0059.0 x 10 (Afm’) Find the induced voltage in the conductor of Fig. 12-8 where B=0.04a,T and U=25sin 10a, (m/s) E, = UXB=0.10sin 10%(-m,) (V/m) v= [cosins0%-m)- ea, = -0.02sin 10° (V) ‘The conductor first moves in the a, direction. ‘The x=0.20 end is negative with respect to the end at the 2 axis for this half cycle. ae Fig. 12-8 Rework Problem 12.4 if the magnetic field is changed to. B=0.04a, (T). Because the conductor cuts no fed lines, the induced voltage must be zero. This may be verified analytically by use of Problem 1.8. v= [coxp)-a-fu-@xay-0 since B and dl are always parallel. ‘An area of 0.65m? in the z=0_ plane is enclosed by a filamentary conductor. Find the induced voltage, given that See Fig, 12-9. > [sosin01(* 5) -ase, =20sin10% (V) CHAP. 12} DISPLACEMENT CURRENT AND INDUCED EMF 19 ‘The field is decreasing in the first half cycle of the cosine function. The direction of # in a closed Circuit must be such as to oppose this decrease. ‘Thus the conventional current must have the direction shown in Fig. 12-9. 12.7. The circular loop conductor shown in Fig. 12-10 lies in the z=0 plane, has a radius of fg 0.10m and a resistance of 5.002. Given B=0.20sin 10'ta, (T), determine the current = @=B-S=2x10%nsin10'r (Wo) ve -2. 2a 60810 (V) i= Gu -O-Axcos 10% (A) ‘At 1=0+ the flux increasing. In order to oppose this increase, current in the loop must have an instantaneous direction —a, where the loop crosses the positive x axis. Fig. 12-10 12.8. The rectangular loop shown in Fig. 12-11 moves toward the origin at a velocity U= 2500, m/s ina field a B=0.80e°%, (T) Find the current at the instant the coil sides are at y=0.50m and0.60m, if R=2.59. ns. 12.10. DISPLACEMENT CURRENT AND INDUCED EMF ICHAP. 12 Only the 1.0-m sides have induced voltages. Let the side at y=0.50m be J. CU =0.Be9(1)(250) = 1558V = BLU = 482V “The voltages are of the polarity shown. The instantaneous current is 155.8 — 148.2. SS SOA ‘A conductor 1em in length is parallel to the z axis and rotates at a radius of 25cm at 1200 rev/min (See Fig. 12-12). Find the induced voltage if the radial field is given by B=0.5a,T. ‘The angular velocity is (2m Sn) Gos Hence U= r= (025140) B= ia, X050,=5.0n(-a,) Vim ve [saya 5.010% ‘The negative sign indicates that the lower end of the conductor is positive with respect to the upper end. A conducting cylinder of radius 7cm and height 15cm rotates at 600 rev/min in 2 radial field B=0.20a,T. Sliding contacts at the top and bottom connect to a voltmeter as shown in Fig. 12-13. Find the induced voltage. 0 = (600Y()(2x) = 207 rad/s U=(202}(0.07)a, m/s E,= UXB=0.88(—2,) Vim Each vertical element of the curved surface cuts the same flux and has the same induced voltage. These elements are effectively in a parallel connection and the induced voltage of any CHa. 12] DISPLACEMENT CURRENT AND INDUCED EMF 201 clement is the same as the total v= [ ” 88a.) dz -0.13V (at the bottom) 12.41. In Fig. 12-14 a rectangular conducting loop with resistance R=0.20Q turns at a 00 rev/min. The vertical conductor at r,=0.03m isinafield B,=0.2Sa,T, and the conductor at 7, =0.0Sm_ is ina field B,=0.80a, T. Find the current in the loop. U,= (S00)(4)(2)(.03)ag = 0.5srag m/s [stn x028)rde tev Similarly, U;= 0.88719, m/s and vy = ~1.04V. Then , _ 040.20 ja 208 in the direction shown on the diagram. 12.12. The circular disk shown in Fig. 12-15 rotates at (rad/s) in a uniform flux density B= Ba, Sliding contacts connect a voltmeter to the disk. What voltage is indicated on the meter from this Faraday homopotar generator? Fig. 12-15 ‘One radial element is examined, A general point on this fora, so that clement has velocity U= E,, = UXB=orBa, [ota dn and s 2 where a is the radius of the disk. The positive result indicates that the outer point is positive with respect t0 the center for the directions of B and @ shown. 12.13. A square coil, 0.60m on a side, rotates about the x axis at w=60mrad/s in a 2 field B=0.80a,T, as shown in Fig. 12-16(a). Find the induced voltage 12.14. iE DISPLACEMENT CURRENT AND INDUCED EMF [cHap. 12 oat o Fig. 12-16 ‘Assuming that the coli initially in the xy plane, a= ot = 601 (rad) ‘The projected area on the xy plane becomes (see Fig. 12-16(6)} A=(0.6)(0.6c0s 6021) (00) ‘Then $= BA=0.288cos Gxt (WE) and P= S43sin6oat (V) Lenz's law shows that this is the voltage of a with respect to b. ‘Alternate Method Each side parallel to the x axis has a y component of velocity whose magnitude is IU, = {resin of = 118.02 sin 601} (en/s) ‘The voltages BE{U,| for the two sides add, giving ll = 2(BE|U,))=1S4.3sin 6d CV) Lena's law again determines the proper sign. Check Example 5 by means of the original, differential form of Faraday's law. From Fig. 12-7(6) the projected loop area normal to the field is A cos ot, whence $= BO) (Acoso) do_ 4B and WA cos wl + BAw sin aot =v, + v2 ae ae (cis almost always simpler to use the differential form.) Find the electric power generated in the loop of Problem 12.11. Check the result by calculating the rate at which mechanical work is done on the loop. ‘The electric power is the power loss in the resistor: P.=PR=(4.20)'(0.20)=3.53W “The forces exerted by the field on the two vertical conductors are F, =10, XB.) = 4.209(0.50)(0.25)(2, Xa.) = 0.5254, N y= i(h, xB.) = (4.20)(0.50)(0.80\(—0, Xa) = 1.68, N CHAP. 12] DISPLACEMENT CURRENT AND INDUCED EMF 203 ‘To turn the loop, forces -F, and ~F, must be applied; these do work at the rate P= (-B,)* U, + (HF) Us = (~0.525)(0.50n) + (1.68)(0.83n) = 3.55 W ‘To within rounding errors, P= P, Supplementary Problems 12.16. Given the conduction current density in a lossy dielectric as J,=0.02sin 10% (A/m"), find the displacement current density if o=10"S/m and €,=6.5. Ans. 1.15% 10-*eos 10" (A/m*) 12.17. A circular-cross-section conductor of radius 1.Smm carries a current i, =5.5sin4 x 10" (WA). What is the amplitude of the displacement current density, if 9=35MS/m and €,= 17 Ans, T8110" wl? 12.18. Find the frequency at which conduction current density and displacement current density are equal in (@ distilled water, where o=20% 10*S/m and ¢,=81; (6) seawater, where o=40S/m and ¢,=1. Ans. (a) 4.44 10" Hz; (6) 7.19% 10" Hz 12.19. Concentric spherical conducting shells at Smm and r,=Imm are separated by a dielectric for which €,=85. Find the capacitance and calculate i,, given an applied voltage v= 150sin 5000 (V). Obtain the displacement current ip and compare it with i, ‘Ans. i,= ig =7.09* 10°" cos 5000 (A) 12.20, Two parallel conducting plates of area 0.09m? are separated by 2mm of a lossy dielectric for which ¢=83 and ¢=80x10~S/m. Given an applied voltage v=10sin10'r (V), find the total rms current. Ans. 0.192 A 12.21. A parallel plate capacitor of separation 0.6mm and with a dielectric of €,= 15.3 has an applied rms voltage of 25V at a frequency of 15GHz. Find the rms displacement current density. Neglect fringing. Ans. 5.32% 10° A/m* 12.2. A conductor on the x axis between x=0 and x=0.2m has a velocity U=60—,m/s ina field B=0.04a, T. Find the induced voltage by using (a) the motional clece field intensity. () dgidt, and (©) BeU. Determine the polarity and discuss Lenz's law if the conductor was connected 0 closed loop. Ans. 0.08. (x =0 end is positive) 12.23. Repeat Problem 12.22 for B=0.04sinkza, (T). Discuss Lenz’s law as the conductor moves from flux in one direction to the reverse direction. Ans. 0.048sinkz (V) 12.24. ‘The bar conductor parallel to the y axis shown in Fig. 12-17 completes a loop by sliding contact with the conductors at y=0 and y=0.05m. (a) Find the induced voltage when the bar is stationary 30 sin 10'ma, (T). (6) Repeat for a velocity of the bar U= Discuss the polarity. Sos 10'r (V); (b) ~7.5¢0s 10% 2.25sin 0% (V) ” 204 1225, 1226. 221. 1228. B31. DISPLACEMENT CURRENT AND INDUCED EMF ICHAP. 12 ‘The rectangular coil in Fig. 12-18 moves to the right at speed U=2.Sm/s. ‘The left side cuts flux at right angles, where B, =0.30T, while the righ side cuts equal flux in the opposite direction. Find the instantaneous current in the coil and discuss its direction by use of Len2’s law. ‘Ans, IS mA (counterclockwise) a 050m T 8 win f son Fig. 12-18 ‘A rectangular conducting loop in the = plane with sides parallel to the axes has y dimension 1 em and x dimension 2cm. Its resistance is 5.0. At a time when the coil sides are at x 2em and x=22cm itis moving toward the origin at a velocity of 2.5 m/s along the x axis. Find the current if B=5.0e Ma, (1). Repeat for the coil sides at Tem. Ans. 0.613mA,2.75 mA fom and x ‘The 2.0m conductor shown in Fig. 12-19 rotates at 1200rev/min in the radial field B. 0.10sin ga, (T). Find the current in the closed loop with a resistance of 100.2. Discuss the polarity and the current direction. Ans. 5.03 10" sin 4x1 (A) Fig. 12419 Ina radial field B=0.50a,(1), (wo conductors at r=0.23m and r=0.25m_ are parallel tothe 2 axis and are 0.0Lm in length. If both conductors are in the plane g=40.r, what voltage is available to circulate a current when the two conductors are connected by radial conductors? Ans. 126mV In Fig. 12-20 a radial conductor, 3=76cm, is shown embedded in a rotating glass disk. Two 11.2, m@ resistors complete two circuits. The disk turns at I2rev/min. If the field at the disk is B=0.30a,(T), calculate the electric power generated. What is the effect of this on the rotation? Discuss Lenz's law as it applies to this problem. Ans. 46.3 uW RK Fig. 12-20 ‘What voltage is developed by a Faraday disk generator (Problem 12.12) with the meter connections at_j=1mm and r= 100mm when the disk turns at S0Orev/min in a flux density of 0807? Ans. 0.200,V ‘A. coil such as that shown in Fig. 12-16(a) is 75mm wide (y dimension) and 100mm long ( imension). What is the speed of rotation if an ams voltage of 0.25 V is developed in the uniform field 5a, (1)? Ans. 1000 rev/min Chapter 13 Maxwell’s Equations and Boundary Conditions 13.1. INTRODUCTION ‘The behavior of the electric field intensity E. and the electric flux density D across the interface of two different materials was examined in Chapter 7, where the fields were static. A. similar treatment will now be given for the magnetic field strength H and the magnetic flux density B, again with static fields. This will complete the study of the boundary conditions on the four principal vector fields In Chapter 12, where time-variable fields were treated, displacement current density Jp was introduced and Faraday’s law was examined. In this chapter these same equations and others developed earlier are grouped together to form the set known as Maxwell's equations. These ‘equations underlie all of electromagnetic field theory; they should be memorized. 13.2, BOUNDARY RELATIONS FOR MAGNETIC FIELDS When H and B are examined at the interface between two different materials, abrupt changes can be expected, similar to those noted in E and D at the interface between two different diclectrics (Gee Section 7.7) In Fig. 13-1 an interface is shown separating material 1, with properties a, and py , from 2, with @, and... The behavior of B can be determined by use of a small right circular cylinder Positioned across the interface as shown. Since magnetic flux lines are continuous, fa-ds-[ B-ds+f pease [ m-ds.-0 on Now if the two planes are allowed to approach one another, keeping the interface between them, the area of the curved surface will approach zero, giving [ares | p.-as,-0 or HB [dit Bal as.<0 from which Bus = Bra Fig. 131 20s 206 MAXWELL'S EQUATIONS AND BOUNDARY EQUATIONS [cHa. 13 In words, the normal component of B is continuous across an interface. Note that either normal to the interface may be used in calculating By and By. ‘The variation in H across an interface is obtained by the application of Ampere’s law around a closed rectangular path, as shown in Fig. 13-2. Assuming no current at the interface, and letting the rectangle shrink to zero in the usual way, On fH edt HadG—Hadts whenee Ha=He ‘Thus tangential H has the same projection along the two sides of the rectangle. Since the rectangle can be rotated 90° and the argument repeated, it follows that Hy In words, the tangential component of His continuous across a current-free interface. ‘The relation between the angles made by Hj and Hy with a current-free interface (see Fig. 13-2) is obtained by analogy with Example 6, Section 7.7. Fig. 32 13.3 CURRENT SHEET AT THE BOUNDARY If one material at the interface has a nonzero conductivity, a current may be present. This could be a current throughout the material; however, of more interest is the case of a current sheet at the interface. Figure 13-3 shows a uniform current sheet. In the indicated coordinate system the current sheet has density -K and it is located at the interface x=0 between regions 1 and 2. The magnetic field H’ produced by this current sheet is given by Example 2, Section 9.2, Hi=1KxXa,,=iKa, Hy = 1K Xa, = 4Ko(—a,) ‘Thus Hi’ has a tangential discontinuity of magnitude |Kol at the interface. If a second magnetic field, HT’, arising from some other source, is present, its tangential component will be continuous at the interface. ‘The resultant magnetic field, H-H+H will then have a discontinuity of magnitude |Ko| in its tangential component. ‘This is expressed by the vector formula CHAP. 13] MAXWELL'S EQUATIONS AND BOUNDARY EQUATIONS 207 Fig. 13:3 where a, is the unit normal from region 1 10 region 2. The vector relation, which is independent of the choice of coordinate system, also holds for a nonuniform current sheet, where K is the value of the current density at the considered point of the interface. 13.4 SUMMARY OF BOUNDARY CONDITIONS For reference purposes, the relationships for E and D across the interface of two dielectrics are shown below along with the relationships for H and B. Magnetic Fields Electric Fields _ Day = Daz (charge-free) Bu= Bra {(o,-vy-area ams (wih sure charge) { HaHa (current-free) 0, has pa= Given HH, = 4.02, + 3.08, 6.00, (A/m) show that @,=19.7° and that H,=7.12A/m. Proceed as in Problem 13.1 Hy= 40a, + 3.0a,— 600, (A/m) = uf12.00,+ 9.00, ~18.00,) (T) B,= (12.00, + 15.08, ~30.08,) (7) Hy= 2.402, + 3.00, — 6.08, (A/m) VOM FBO + (607 = 7.12 Alm ‘The angle a, between H, and the normal is given by Now I ate co FE eOM or a= 70.5 Then 6,=90°— a,= 19.7". Region 1, where pi Fig. 13-5). In region 2, “2=6. Bi= is the side of the plane y+z=1 containing the origin (see Oa, + 1.08, (T), find B, and Hy. iE 210 MAXWELL'S EQUATIONS AND BOUNDARY EQUATIONS ICHAP. 13 ‘Choosing the unit normal», = (a, + 2,)/V2, Bg 2. 41.0) (a 8) B, =B, ~B,,=20a, +054, 0.5, Hy = FE (O5n, +0125, ~0.1258,)= Hy Boa = HotsHla= 3.00, + 0.75a, ~ 0.758, Now the normal and tangential parts of B, are combined. B, = 3.00, + 1.258, —0.25a, (1) v= Leasty 0215-008) (Alm) 13.4. In region 1, defined by z<0, ju =3 and (0.28, +0.5a, + 1.0a,) (Al Pm +0.5a, + ) (Alm) Find H, if it is known that 0, = 45°. or =F ‘Then, 0,-6.7 and or a= 5.57 tan ass 3 From the continuity of normal By Hafiia= HaHa, and so fs + ia. fm) (02m, +054, 40.540.) (Alm) Ma w= (02,4050, +4100) 13.5. A current sheet, K=6.5e,A/m, at x=0 separates region I, «<0, where Hy= 10a, A/m and region 2, x>0. Find H, at x ~ +0. ‘Nothing is said about the permeabilities of the two regions; however, since H, is entirely tangential, fa change in permeability would have no effect. Since B, and therefore Hy, =0. CHAP. 13) MAXWELL'S EQUATIONS AND BOUNDARY EQUATIONS 2 (H.-H) Xa, (102, — Hay) (10- H,\-2,) =6.59, Ha= 165 (Ale) ‘Thus, a= 16.5a, (A/m). 13.6. A current sheet, K=9.0a, A/m, is located at z=0, the interface between region sig 1, z<0, with pa=4, and region 2, z>0, po=3. Given that H)=14.5a,+ 80a, (A/m), find Hi ‘The current sheet shown in Fig. 13-6 i fist examined slone 10.0), x(-2,)=4.5(-a) © K=908, Fig. 136 From region 1 to region 2, H, will increase by 9.0 A/m due to the current sheet. ‘Now the complete H and B fields are examined. y= 14.50, +808, (Alm) B, = 110(43.5m, + 24.00,) (T) B= (22.08, +2408,) (1) H,=5.58, +6.0a, (A/m) Note that H,, must be 9.0 A/m less than Hi,» because ofthe current sheet. B,, is obtais ‘An alternate method isto apply (Hy — Hi) X ayie= K: (Hua, + Hy, + Hoa) Xa, = K+ (14.50, +8.08,) X09, — Ha, + Hye ~5.50, from which H,,=5.5A/m and Hy,=0. This method deals exclusively with tangential Hi; any ‘normal component must be determined by the previous methods. 25 Hota Hs 13.7. Region 1, z<0, has 4,=1.5, while region 2, z>0, has p.=5. Near (0,0,0), B,=2.40a,+ 10.08, (T) By =25.75a,~17.7a, +1008, (T) If the interface carries a sheet current, what is its density at the origin? [Near the origi 1 pg Oa +6.67,) (Alm) Hy = G5m,~ 3.546, 420) (AY) 212 MAXWELL'S EQUATIONS AND BOUNDARY EQUATIONS (CHAP. 13 Then the lea value of Kis gen by te 152228) Cay Kaiya Ease +25 +46) 0, 82(EN) Cam) Given E=E,, sin (wt ~ z)a, in free space, find D, B and H. Sketch E and Hat ¢= D= 48 = cE, sin (wt — fr), ‘The Maxwell equation VXE= B/at gives a * “ 2 a 2 a oy a 0 E,sin (at fz) 0 Integrating, where the “constant” of integration, which is a static field, has been neglected. Then, Fe asin ot ~ ym Note that E and H are mutually perpendicular. At ¢=0, sin(wt~z)= sin Bz. Figure 13-7 shows the two fields along the 2 axis, on the assumption that E,, and f are positive. 13.9. Show that the E and H fields of Problem 13.8 constitute a wave traveling in the z ig direction. Verify that the wave speed and £/H depend only on the properties of free space. SL antttogenervay as safer $2). A gen se of E and His ten character by (ot — Pr= const. = ay oF But this is the equation of a plane moving with speed 6 quar. 13] MAXWELL’S EQUATIONS AND BOUNDARY EQUATIONS m3 in the direction of its normal, a,. (It is assumed that B, as well as wis positive; for A negative, the direction of motion would be —s,.) Thus, the entire pattern of Fig. 13-7 moves down the 7 axis with toad “The Marvell equation VXH= 90/3 gives C7 ye a 2 ala, 2 FR | Heenan fom, BE, BEsnfae—pe) 00 Fos at ~ fi) = et (ot ~f2), tie oto BP Consequenty, - fh - fo sre = ole (10°°/360)(42 x 10°") 3x 10" (enis) Moreover Fa tte fH2~ 1208 (W/A)= 12082 13.10. Given H= H,e"“"**"a, in free space, find E. vxn-2 Bligh, =D and E=Dle, 13.11. Given E=30nelO%*6g, (Vm) = Hye0**9, (A/m) in free space, find H,, and B (6 >0). This is a plane wave, essentially the same as that in Problems 13.8 and 13.9 (except that, there, E, ‘was in the y direction and H in the x direction). The results of Problem 13.9 hold for any such wave in Tree spe: o1 a=. 3x 10° (m/s) Bo Vea 1") “Thus for the given wave, Ww 1 Fg 3 adm) ‘To fis the sign of Hy, apply VXE= FP30neI0*" Fy, = —j1P gH en, which shows that H, must be negative. 24 MAXWELL’S EQUATIONS AND BOUNDARY EQUATIONS IcHAP. 13 13.12, In a homogeneous nonconducting region where 4, =1, find ¢, and w if a E=30nei- Omg, (V/m) = 1.00! 4g, (Asm) Here, by analogy to Problem 13.9, eA eae ay EL fae fu, B Var ven £- tnt @ Thus, since 4, = @ 3x L Mn 1200 which yield ¢,=16, @=10"rad/s. In this medium the speed of light is ¢/4. Supplementary Problems 13,13. Region 1, where ,=5, is on the side of the plane Gx-+4y+3z=12 that includes the origin. In region 2, Hi2=3 Given 1 H,=+ G00, - 0.5 In na! a) (A/m) find B, and ,. Ans. 12.15a, + 0.608, + 1.58, (1), 56.6" 13.14. The interface between two different regions is normal to one of the three cartesian axes. If 143.50, +24.04,) = pd22.0m, + 24.00,) what isthe ratio (tan @,)/(tan 6,)? Ans. 0.506 IBS. Inside a right circular cylinder, y,,= 1000. The exterior is free space. If B, the cylinder, determine B, just outside. Ans. 2.5a, (mT) Sm, (T) inside 13.46, In spherical coordinates, region fis rb. Regions 1 land Jare free space, while j42=S00. Given B,=0.20a,(T), find H in each region 0.20, ‘4x10 20 ‘Ans, == (alm), (Alem), (Alen) BAT. A current sheet, K=(8.0/u)a, (A/m), at x=0. separates region 1, x<0 and py=3, from region 2, x>0 and p=l.” Given Hy=(10.0/a)\(a, +2.) (A/m), find Hy ans 2 060204) (Aim) 13.18. The x=0 plane contains a current sheet of density K which separates region J, x0, P>0. Ans, See Fig. 138 13.20. In free space, B= Beg, ‘Show that 28, Bm — Be eet ra 1321. In homogencous region where, =1 and €,=50, Ex Done, (Vim) Ba pighlne"-P, (T) Find a and Hq if the wavelength i 1.78m. Ans. 1.5% 10"rad/s, 1.18 A/m Chapter 14 Electromagnetic Waves 14.1 INTRODUCTION ‘Some wave solutions to Maxwell's equations have already been encountered in the Solved Problems of Chapter 13. The present chapter will extend the treatment of electromagnetic waves. ‘most regions of interest are free of charge, it will be assumed that charge density p=0. Moreover, linear isotropic materials will be assumed, with D=«E, B= HH, and J~oE. 142 WAVE EQUATIONS With the above assumptions and with time dependence efor both E and H, Maxwell's ‘equations (Table 13-1) become vx w vx (2) ve (3) v- @ ‘Taking the curl of (1) and (2), Vx (V XH) = (0+ jwe(VXE) ¥x(0 XE) = —jou(¥ XH) Now, in cartesian coordinates only, the Laplacian of a vector VAs (07A,)a, + (V7A,)8, + (V7A,)a, satisfies the identity YX (Vx A)=V(V- A)— VA ‘Substitution for the “‘curl curls” and use of (3) and (4) yields the vector wave equations vi VE jou(o + jwe)H= A =jun(o + joe = 7B “The propagation constant y is that square root of y* whose real and imaginary parts are positive: y=atip “= oo (ie(S)~) o) pros (yie(Z) +) 6 216 CHAP. 14] ELECTROMAGNETIC WAVES 27 143 SOLUTIONS IN CARTESIAN COORDINATES ‘The familiar scalar wave equation in one dimension, oF Ler hhas solutions of the form F=f(z—ut) and F=g(z+ut), where f and g are arbitrary functions. These represent waves traveling with speed u in the +z and —2 directions, respectively. In Fig. 14-1 the first solution is shown at ¢=0 and t=); the wave has advanced in the +z direction a distance of uf, in the time interval f,. For the particutar choices f)=Cerm™ — and g(x) = Den" Fig. 14-1 harmonic waves of angular frequency @ are obtained: Fel) and F = Detorsf) in which B=«w/u Of course, the real and imaginary parts are also solutions to the wave equation. One of these solutions, F=Csin(or—fz), is shown in Fig. 142 at ¢ 0 and ¢=2/2. In this interval the wave has advanced in the positive z direction a distance d=u(x/2m)= 1/28. At any fixed 1, the waveform repeats itself when x changes by 2n/B; the distance da ant 8 is called the wavelength. ‘The wavelength and the frequency f = 2s enjoy the relation su or A=Tu where T= 1/f=2z/@ is the period of the harmonic wave. The vector wave equations of Section 14.2 have solutions similar to those just discussed. Because the unit vectors a,, a,, and a, in cartesian coordinates have fixed directions, the 218 ELECTROMAGNETIC WAVES [cHar. 14 wave equation for H can be rewritten in the form (Of particular interest are solutions (plane waves) that depend on only one spatial coordinate, say z. Then the equation becomes fH aH which, for an assumed time dependence e’, is the vector analog of the one-dimensional scalar wave ‘equation. Solutions are as above, in terms of the propagation constant y. H(z, #) = Hoe* ey The corresponding solstions forthe electric field are E(z, 1) = Ege*™ eng, ‘The fixed unit vectors a), and ag are orthogonal and neither field has a component in the direction of propagation. This being the case, one can rotate the axes to put one of the fields, say I, along the x axis. Then from Maxwell's equation (2) it follows that Hi will lie along the ty axis for propagation in the +z direction. EXAMPLE 1. Given the field E=Eye"ar (time dependence suppressed), show that E can have no ‘component in the propagation direction, +a, ‘The cartesian components of a are found by projection: B= Bee (aes aa + (Be 3,)a, + (ea) ae Feber Meera)=0 which can held only if aya, =0. Consequently, E has no component is ‘The plane wave solutions obtained above depend on the properties 4, ¢, and o of the medium, because these properties are involved in the propagation constant y. 144 SOLUTIONS FOR PARTIALLY CONDUCTING MEDIA For a region in which there is some conductivity but not much (e.g., moist earth, seawater), the solution to the wave equation in E is taken to be E=Eve"“a, lor jue Hef roe jou * The ratio E/H is characteristic of the medium (it is also frequency-dependent). More specifically for waves E=E,a,, H=H,a, which propagate in the +z direction, the intrinsic impedance, n, of the medium is defined by ‘Then, from (2) of Section 14.2, Thus CHaP. 14] ELECTROMAGNETIC WAVES 219 where the correct square root may be written in polar form, |7]/8, with Vale oe tan2@=— and P< 0<45° Inl= (if the wave propagates in the —2 direction, E,/H,=—n. In effect, y is replaced by —y and the other square root used.) Inserting the time factor e! and writing y= a+ j6 results in the following equations for the fields in a partially conducting region: E(z, t) = Ege **ello-Fig, fo. 1 Hz, 1) = 28 earch Beg, On ‘The factor e~** attenuates the magnitudes of both E and Hi as they propagate in the +z direction. The expression for a, (5) of Section 14.2, shows that there will be some attenuation unless the conductivity is zero, which would be the case only for perfect dielectrics or free space. Likewise, the phase difference @ between E(z, t) and H(z, ¢) vanishes only when o is zero. ‘The velocity of propagation and the wavelength are given by oe (Vir) +) If the propagation velocity is known, Af =u may be used to determine the wavelength A. The term (/qe)* has the effect of reducing both the velocity and the wavelength from what they would be in either free space or perfect dielectrics, where o=0. Observe that the medium is dispersive waves with different frequencies w have different velocities u. 145 SOLUTIONS FOR PERFECT DIELECTRICS For a perfect dielectric, 0=0, and so view Since a0, there is no attenuation of the E and H waves. The zero angle on 1 results in being in time phase with E at each fixed location. Assuming E in a, and propagation in a, , the field equations may be obtained as limits of those in Section 14.4: a=0 p=oVue on ny, Eo on, 7" 20 ELECTROMAGNETIC WAVES [cuap. 14 Sotutions in Free Space. Free space is nothing more than the perfect dielectric for which 12 F/gq Om 8.854% 10° F/m~ 5 Fim Wa Wo 4nx107Him — e= For free space, =Mo~ 12022 and u=c~3X 108 m/s 146 SOLUTIONS FOR GOOD CONDUCTORS; SKIN DEPTH Materials are ordinarily classified as good conductors if % we in the range of practical frequencies. Therefore, the propagation constant and the intrinsic impedance are . fo a yeatip a=BpayOavafie n= yas Ie s seen that for all conductors the E and H waves are attenuated. Numerical examples will show that this isa very rapid attenuation, «will always be equal to B. At each fixed location His out of time phase with E by 45° or /4rad. Once again assuming E in a, and propagation in a, , the field equations are, from Section 14.4, Beene, n= Benetennny, Moreover, u=8= 2-05 2nd 3 Vo ‘The velocity and wavelength in a conducting medium are written here in terms of the skin depth or depth of penetration, b=. 1 Vato EXAMPLE 2. Assume afield E=10e"e""s, (V/m), with f'=«w/2x=100MHz, at the surface of a copper conductor, = S8MS/m, located at 2>0, as shown in Fig. 14-3, Examine the attenuation as the wave propagates into the conductor. ete o=0 « £ Propagation Siresion Fig. 143 ‘At depth 2 the magnitude of the field is IE} = 1.00 ** = 1.06-* =661 am 6 Vafuo CHAP. 14] ELECTROMAGNETIC WAVES ma ‘Thus, after just 6.61 micrometers the field is attenuated to e"'= 36.8% of its initial value. At 58 or 33 ‘micrometers, the magnitude is 0.67% of its initial value—practically zero. 14.7 INTERFACE CONDITIONS AT NORMAL INCIDENCE ‘When a traveling wave reaches an interface between two different regions, it is partly reflected and partly transmitted, with the magnitudes of the two parts determined by the constants of the two tegions. In Fig. 14-4, a traveling E wave approaches the interface z=0 from region I, 2< 0. E/andE’ are at 7=—-0, while E’is at z= +0 (in region 2). Here, i signifies “incident,” r “reflected” and ¢ “transmitted.” Normal incidence is assumed. The equations for E and H can be written EG, 1) = Eye ea, ere, Exe ea, Hie es, WG.) = Hees, HG, N= Hie ea, ‘One of the six constants—it is almost always E{—may be taken as real. Under the interface conditions about to be derived, one or more of the remaining five may turn out to be complex. With nominal incidence, E and Hi are entirely tangential to the interface, and thus are continuous across it. At z=0 this implies E+ Eo= Ey Ho+ Ho= Ho ic impedance in either region is equal to +E,/H, (see Section 14.4). Es Furthermore, the intr Fa Es, mo” og ‘The five equations above can be combined to produce the following ratios in terms of the intrinsic impedances: 22 ELECTROMAGNETIC WAVES [cHap. 14 ‘The intrinsic impedances for various materials have been examined earlier. They are repeated here for reference. i a icting medium: partially conducting 1 Not ijoe joe a= ftps i ” € nom f= 12022 € EXAMPLE 3. Traveling E and H waves in free space (region 1) are normally incident on the interface with a perfect dielectric (region 2) for which «,=3.0. Compare the magnitudes of the incident, reflected, and ‘transmitted E and H waves at the interface. 120m, Ve, =21.70 atm 2m ath 14.8 OBLIQUE INCIDENCE AND SNELL'S LAWS ‘An incident wave that approaches a plane interface between two different media generally will result in a transmitted wave in the second medium and a reflected wave in the first. The plane of incidence is the plane containing the incident wave normal and the local normal to the interface; in Fig. 14-5 this is the xz plane, ‘The normals to the reflected and transmitted waves also lie in the plane of incidence. The angle of incidence 0,, the angle of reflection 0,, and the angle of transmission @—all defined as in Fig. 14-5—obey Snell's law of reflection, and Snell's law of refraction, Fig 145 CHAP. 14] ELECTROMAGNETIC WAVES 23 Meccanots 4 A av nen an age of 37m ao tn, «<2, at thane a ‘transmission, and tepeat with an interchange of the regions. Since 14 = Hs sin 6, sin 8, From teflon to air, m= ot 046.43" ‘Supposing both media of the same permeability, propagation from the optically denser medium (e:>«,) results in 6, ‘As 6, increases, an angle of incidence will be reached that results in’ 6,=90. At this critical angle of incidence, instead of a wave being transmitted into the second ‘medium there will be a wave that propagates along the surface. ‘The critical angle is given by ania ff 2 Srcssumes eos ore sie rating tom on fe int heeanee 8, = sin"! a. 68 14.9 PERPENDICULAR POLARIZATION ‘The orientation of the electric field E with respect to the plane of incidence determines the polarization of a wave at the interface between two different regions. In perpendicular polarization E is perpendicular to the plane of incidence (the xz plane in Fig. 14-6) and is thus parallel to the (Planar) interface. At the interface, Eg, 92008 6, ~ m,c08 6, Bi ng00s 8, +7, 008 6, Ey___2nscos 6, Ey 200s 8, +n, cos 6, Note that for normal incidence 6,=6,=0° and the expressions reduce to those found in Section 14.8. and Fig. 146 28 ELECTROMAGNETIC WAVES [cHap. 14 Ik is not difficult to show that, if jy= 2, M2608 8,~n,c0s 6,#0 for any 6, Hence, a perpendicularly polarized incident wave suffers either partial or total reflection. 14.10 PARALLEL POLARIZATION For parallet polarization the etectric field vector E ties entirely within the plane of incidence, the xz plane as shown in Fig. 14-7. (Thus E assumes the role played by H in perpendicular polarization.) At the interface, Ee_ mos 0, ~ 1,608 8, 3005 6, + 7,008 8, 2ns608 6, 1 608 8, ~ nz cos 6, ig. 187 In contrast to perpendicular polarizations, if ,=y2 there will be a particular angle of incidence for which there is no reflected wave. This Brewster angle is given by 1 \2 tant 4/2 « SRecwweis 6 the never ale oe prt plizd wove eng tm no gs fr wih = 3 Suc stanpine waves When waves traveling in a perfect dielectric (c, = a, =0) are normally incident on the interface with a perfect conductor (0; =*, 1 =0), the reflected wave in combination with the incident wave produces a standing wave. In such a wave, which is readily demonstrated on a clamped taut string, ‘the oscillations at all points of a half-wavelength interval are in time phase. The combination of incident and reflected waves may be written Ele, 1) =[EfMOmP 4 Ea, = Ee I + Eset, Since n2=0, Ej/Ej=—-1 and Elz, 1) = e(Eie — Ee), 6 On = tan V5.0 = 65.91° ~2jE3 sin Brea, CHAP. 14] ELECTROMAGNETIC WAVES 25 or, taking the real part, FG, ¢ ‘The standing wave is shown in Fig. 14-8 at time intervals of 7/8, where T=2x/w is the period. At 1=0, E=O everywhere; at 1=1(T/8), the endpoints of the E vectors lie on sine curve 1; at 1=2(T/8), they lie on sine curve 2; and so forth. Sine curves 2 and 6 form an envelope for the oscillations; the amplitude of this envelope is twice the amplitude of the incident wave. Note that adjacent half-wavelength segments are 180° out of phase with each other. Ei sin Bz sin wa, Fig. 148 14.12 POWER AND THE POYNTING VECTOR ‘Maxwell's first equation for a region with conductivity ¢ is written and then E is dotted with each term. aE VxH= 0B + eS 2 aE Es(VXH) = 08 + E65 where, as usual, E7=E+E. The vector identity V+(AxB)=B-(VXA)—A-(VXB) is employed to change the left side of the equation. seg Ho (@xE)—7-(EH) e084 Ee by Manvel second exuation, wate z wecexey =n (14) - Similarly, Substituting, and rearranging terms, 26 ELECTROMAGNETIC WAVES [cHap. 14 Integration of this equation throughout an arbitrary volume v gives [oerav=-[ ($2222) dof cexm-as where the last term has been converted to an integral over the surface of v by use of the divergence theorem. ‘The integral on the left has the units of watts and is the usual ohmic term representing energy dissipated per unit time in heat. This dissipated energy has its source in the integrals on the tight. Because €£?/2 and wH?/2 are the densities of energy stored in the electric and magnetic fields, respectively, the volume integral (including the minus sign) gives the decrease in this stored energy. Consequently, the surface integral (including the minus sign) must be the rate of energy ‘entering the volume from outside. A change of sign then produces the instantaneous rate of energy leaving the volume: P= f Ext 8 §o-as where @=EXH. is the Poynting vector, the instantaneous rate of energy flow per unit area at a point. In the cross product that defines the Poynting vector, the fields are supposed to be in real form. If, instead, E and H are expressed in complex form and have the common time-dependence e™, then the time-average of ® is given by =}Re(EXH*) where H’ is the complex conjugate of H. This follows the complex power of cir }VI*, of which the power is the real part, P=4Re VI*. For plane waves, the direction of energy flow is the direction of propagation. Thus the Poynting vector offers a useful, coordinate-free way of specifying the direction of propagation, or of determining the directions of the fields if the direction of propagation is known. This can be particularly valuable where incident, transmitted, and reflected waves are being examined. analysis, S= Solved Problems 14.1. A traveling wave is described by y= 10sin(z— ax). Sketch the wave at 1=0 and sig at f=t, when it has advanced 4/8, if the velocity is 3x 10*m/s and the angular frequency «= 10°rad/s, Repeat for w=2x 10*rad/s and the same t, “The wave advances A in one period, T=2n/w, Hence uate pee = oxi gee Brn OX 109 gig = 26m The wave isshown at 1=0. and ¢=1, in Fig. 149(@). Attwice the frequency, the wavelength Ais ‘one-half, and the phase shift constant fis twice, the former value. See Fig. 149(b). At 4, the wave has also advanced 236 m, but ths distance is ow A/4. CHAP. 14] ELECTROMAGNETIC WAVES. zz © o Fig. 149 14.2. In free space, E(z,1)=10 sin (wt -fz)a, (V/m). Obtain H(z, 0). Examination of the phase, ww — Az, shows that the direction of propagation is +z. Since EH. must also be in the +z direction, Hi must have the direction ~a,, Consequently, =i sin(we— fe) (Arm =120x2 or mo= 1 inom and He, )=— 22 sinew — pada, (Alm) 14.3. For the wave of Problem 14.2 determine the propagation constant , given that the frequency is f =95.5MHz. In general, y= Vjont(o +e). In free space, 0-0, so that v=o aon i) GI 20) Note that this result shows that the attenuation factor is @=0 and the phase-shift constant is B=20rad/m. 14.4, Examine the field x E(z, 1) = 10sin (wt + Bz), + 10.c0s (ax + Bz)a, inthe z=0 plane, for ot =0, x/4, 2/2, 31/4 and x. ‘The computations are presented in Table 14-1 Table 14-1 E,=1sinar |E,-cosa | B=Ea,+Ea, 0 10 108, 10 10 e|) Sls 5. 166. m7. ELECTROMAGNETIC WAVES IcHap. 16 ‘As shown in Fig. 14-10, E(x, t) is circularly polarized, In addition, the wave travels in the —a, Fig. 1410 An H field travels in the —a, direction in free space with a phaseshift constant of 30.0 rad/m and an amplitude of (1/3) A/m. If the field has the direction —a, when 1=0 and z=0, write suitable expressions for E and H. Determine the frequency and wavelength. In a medium of conductivity o, the intrinsic impedance , which relates E and H, would be complex, and so the phase of E and Hi would have to be written in complex form. In free space this restriction is unnecessary. Using cosines, then HG.) = —Lcos(or+ Beda 1rQ or E, = +40 cos (ax + Be) (V/m) E(z, 1) =40c0s (wt + Bz)a, (V/m) 3x10" _ 45 A alts rteladiad Determine the propagation constant y for a material having 4, 0.25 pS/m, if the wave frequency is 1.6MHz. In this ease, ° 0.25 x oo we” Ex. 6x TOMOKO "=? so that m0 Rm evae= 2a MEE 9.48% 10°* adn and y=a+)B~j9.48%10-'m. The material behaves like a perfect dielectric at the given frequency. Conductivity of the order of 1 pS/m indicates-that the material is more like an insulator ‘than a conductor. Determine the conversion factor between the neper and the decibel. ‘Consider a plane wave traveling in the +2 direction whose amplitude decays according to B= Exe CHAP. 14] ELECTROMAGNETIC WAVES 29 From Section 14.12, the power carried by the wave is proportional to £?, so thet Pare ‘Then, by definition ofthe decibel, the power drop over the distance zis 1010g10(P/P)4B. But R10 hm P2306 "P a0) 8.686(az) ‘Thus, az nepers is equivalent to 8,686(a2) decibels; ie., LNp=8.686 dB 10106. 148. At what frequencies may earth be considered a perfect dielectric, if o=5x 10°S/m, y,=1, and ¢,=87 Can arbe assumed zero at these frequencies? FR, Asse arbi hat marks the cutoff. Then For small o/ae, of 209) = 2 ov S[}(2)]-Syt-3 /E (1202) = 0.333 Npy ‘Thus, no matter how high the frequency, a will be about 0.333 Np/m, or almost 3.db/m (see Problem 14.7); a cannot be assumed zero. 149. Find the skin depth 5 at a frequency of 1.6MHz in aluminum, where o= ig 38.2MS/m and p,=1. Also find y and the wave velocity u. = 6.44% 10" m= 68.4 um Vafuo " Because «=f = 3", 58x 10 + 1.55 x 1Ot= 2.20 x 10/48" (me 5 = 647 (eis) |, 0 =0) to Es, 14.10. A perpendicularly polarized wave propagates from region 1 (€,=8.5, ua= region 2, free space, with an angle of incidence of 15°. Given £)=1.04V/m, fin Ey Hi, He, and He ‘The intrinsic impedances are 230 ELECTROMAGNETIC WAVES [cnap. 14 Ey 12008 6, + 1 608 6, 0.625 Vien E,___ 2nscon, Eq 1,008 8, + 1, 608 Finally, Hy=Ei/n=7.75 nA/m, Hy=483nA/m, and Hy=4.31nA/m. or B4=1.623 nV /m 14.11. Calculate the intrinsic impedance 7, the propagation constant y, and the wave vel conducting medium in which o=S8MS/m, 4=1, ata frequency f = 100MHz. Varn [45° = 2.14% 10° 45° m"! a= (tus =200x 107 u80 ufora a= Ba1sixie =66lym w= 05 =4.15 x10" m/s 14,12. A plane wave traveling in the +z direction in free space (z<0) is normally incident at_z=0 on a conductor (2>0) for which o=61.7MS/m, 4,=1. The free-space E wave has a frequency f=1,5MHz and an amplitude of 1.0'V/m; at the interface it is given by E(0, 1) = 1.0sin2xfta, (V/m) Find H(z, 1) for 2>0. For 2>0, and in complex form, Ez, 1)= 1.06" e""-F%9, (V/m) where the imaginary part will ultimately be taken, In the conductor, a= B= Vihuo = Vin 1.5 * 10 ar 10” WOL.7 X 1) = L.91 x 10" ne 2 jase = 4.3810 tet Then, since E,/(-H.)=n, H(z, 1)=-2.28 x 10e tH, (Aen) jinary part, Hz, 1)= -2.28% We“ sin aft - Be —x/4)a, (Alm) where fy a, and f are as given above. ‘or, taking the i 14.13. In free space E(z, 1)=50cos (ar fz)a, (V/m). Find the average power crossing a ‘circular area of radius 2.5 m in the plane 2 =const. In complex form, = 500K, (Vm) in +2, and since =120% 2 and propagation Soren Im Ha ems, (Alm) Then 9 4= bRe (EXE) = 460 )a, wim? CHAP. 14] ELECTROMAGNETIC WAVES 21 ‘The flow is normal to the area, and so (2.57 =65.1W 14.14 A voltage source, v, is connected to a pure resistor R by a length of coaxial cable, as shown in Fig. 14-11(a). Show that use of the Poynting vector ® in the dielectric leads to the same instantaneous power in the resistor as methods of circuit analysis. From Problem 7.9 and Ampére’s law, g-—"_, Fin 7a) where @ and b ae the radi ofthe inner and outer conductors, as shown in Fig. 14-11(6). ‘Then arn (Ta) ‘This is the instantaneous power density. The total instantaneous power over the cross section of the dielectric is P=EXH=: ‘which is also the circuittheory result for the instantaneous power loss in the resistor. 14.15. Determine the amplitudes of the reflected and transmitted E and H at the interface shown in 232 ELECTROMAGNETIC WAVES IcHap. 14 Fig. 14-12, if E)=1.5% 10-°V/m in region 1, in which €1=85, 4, 0.” Region 2is free space. Assume normal incidence. ma (Ht = 1292 n= 10x Q=378 Ge, -mom = 1.35 x 10-* V/m nem 2m . 2 p-2.24% 10? Vim mem ove = Bho 1.16%107 Alm Mga ate am 2m nem 5.69% 10 Alm Hy 5.9110 * A/mm esta A cote ey & » — , “ Fig. 14:12 14,16. The amplitude of E’ in free space (region 1) at the interface with region 2 is LOV/m. If Hg=—1.41% 10 A/m, €2=18.5 and 0,=0, find pa. From ‘Then 134 or a= 1984 A normally incident E field has amplitude £j=1.0V/m in free space just outside of seawater in which ¢=80, 4,=1, and o=25S/m. For a frequency of 30MHz, at as what depth will the amplitude of E be 1.0mV/m? Let the free space be region 1 and the seawater be region 2. m=37Q 5 =9.73 /43.5°0 ‘Then the amplitude of E just inside the seawater is Es Eo_ 2m a2 Fe tm ope 5.07 10°" Een & " CHAP. 14] ELECTROMAGNETIC WAVES 233 From y= Vjou(o + jwe) = 24.36 /46.53° m '. (r= 24.36 c08 46.59" = 16.76 Np/m ‘Then, from 10=10 *= (5.40710 *)e-™™ 2=0.234m. 14.18. A traveling E field in free space, of amplitude 100 V/m, strikes a sheet of silver of thickness ig SUM, as shown in Fig. 14-13. Assuming o=61.7MS/m and a frequency f 200 MHz, find the amplitudes |F31, |Eyl, and |E«i. Aaa | sum -— Fig. 14.13 For the silver at 200 MHz, 9 = 5.06% 10"? /45° Ey__ 26.06% 10°25") E, 737+ 56x10 aS whence || = 2.68% 10° V/m Within the conductor, a= f= Vafuo= 2.21 10° ‘Thus, in addition to attenuation there is phase shift as the wave travels through the conductor. Since |E,] and |E,| represent maximum values of the sinusoidally varying wave, this phase shift is not involved. VE sh e-°* = (2.68 We 21% = 888 9 10-4 Vr E 237 " ws Eo Wixsoxias (whee 21.78% 10 vim Supplementary Problems 14419. Given EG, 0 10? sin (6 10" Bz)a, (Vm) in fre space, sketch the wave at ‘and at time ¢, when it has traveled 4/4 along the 2 axis. Find fy Brand Ans. G=2@2ns, B=2rad/m, A= nm, See Fig. M14, 24 ELECTROMAGNETIC WAVES ICHAP. 14 Mig. 1614 14.20. In free space, HE, = 107%, (AY) ‘Obtain an expression for (2, t) and determine the propagation direction. Ans. E=317V/m, ~ 1421. In free space, HG, 1) = 1.33 x 10" cos (4 x 10°) - Bz)a, (Am) ‘Obiain an expression for E(z,1). Find Band A. Ans. Eo=S0V/m, ($)rad/m, 15x. 1422. A traveling wave has a velocity of 10° m/s and is described by y= 10cos(2.52 + at) Sketch the wave as a function of z at ¢=0 and 1=f,=0.838us. What fraction of a wavelength is traveled between these two times? Ans. J. See Fig. 14.15. 14.23. Find the magnitude and direction of E(z, 1) = 10 in (wt ~ Br)a, — 15 sin (ax ~ Beja, (Vim) at =O, 2=30/4, Ans. 1803-V/m, 0.5552, —0.8328, 14.24. Determine y at 500 kHz for a medium in which 4,=1, ¢.=15, electromagnetic wave travel inthis medium? — Ans. 4.06% 10-* mr 0. At what velocity will an 7.74 x 10 m/s 14.25. An electromagnetic wave in free space has a wavelength of 0.20m, When this same wave enters a perfect dielectric, the wavelength changes to 0.09m. Assuming that p,=1, determine €, and the ‘wave velocity in the dielectric. Ans. 4,94, 1.35 10" m/s. 14.26. An electromagnetic wave in free space has a phase shift constant of 0.524rad/m. ‘The same wave has a CHAP. 14] ELECTROMAGNETIC WAVES Bs saz. 428, 14.30. 1431, 143. 1434, 14.38. 14.36, 1439. find ¢, phase shift constant of 1.81 rad/m upon entering a perfect dielectric. Assuming that 1, and the velocity of propagation. Ans. 11.9, 8.69 x 10 m/s Find the propagation constant at 400MHiz for a medium in which ¢, 0.6S/m. Find the ratio of the velocity v to the free-space velocity c. ‘Ans. 99.58 /60.34" m"', 0.097 16, 4 =45, and o= In a partially conducting medium, €,=18.5, 14-800, and o=1S/m. Find a, 6, m, and the velocity u, for a frequency of 10° Hz. ‘Determine H(z, (), given E(z,1)= 50.06°** cos (wot Ae)a, (W/m) ‘Ans. 1130Np/m, 2790rad/m, 2100/22.1°Q, 2.25 x 10"m/s, 2.38 x 10-2e"** cos (wt ~ 0.386 Az)(—8,) (Alm) For silver, = 3.0MS/m. At what frequency will the depth of penetration 5 be 1 mm? Ans. 84.4kH2 ‘At a certain frequency in copper (o=S80MS/m) the phase shift constant is 3.71x 10° rad/m. Determine the frequency. Ans. 601 MHz ‘The amplitude of E just inside a liquid is 10.0 V/m and the constants are u,~ 1, €=20, and om 050S/m. Determine the amplitude of E at a distance of 10cm inside the medium for frequencies of (@) SMHz, (b) 50 MHz, and (¢) SOOMHz. Ans. (a) 7.32 V/m; (b) 3.91 Vim; (¢) 1.42 V/m In free space, E(z,1)=1.0sin(or—fz)a, (Vim). Show that the average power crossing a circular disk of radius 15.5m in a z=const. plane is 1 W. In spherical coordinates, the spherical wave 100. 0.265 = sin @ cos (at—firjas (W/m) = ASE sin @cas (ot Arley (Alm) represents the electromagnetic field at large distances r from a certain dipole antenna in free space. Find the average power crossing the hemispherical shell r=1km, 0=0= HI Ans. 55.SW In free space, E(z, )~ 150sin (ot- fz)e, (V/m). Find the total power passing through a rectangu- lar area, of sides 30mm and 15mm, in the 2=0 plane, Ans. 13.46mW {A free space-silver interface has E3=100V/m on the free-space side. The frequency is 15 MHz and the sider constants are_¢=1,=1, o=61.7MS/m. Determine E; and Ey at the interface. Ans, —100V/m, 7.35% 10°*/45* V/m A free space-conductor interface has Hy=1.0A/m on the free-space side. The frequency is 31.8MHz and the conductor constants are <,=4,=1, o=1.26MS/m. Determine Hs and Hs and the depth of penetration of Ans. 1.0 A/m, 2.0 A/m, 80 um. ‘A traveling H field in free space, of amplitude 1.0 A/m and frequency 200 MHz, strikes a sheet of silver Of thickness Sm with @=61.7MS/m, as shown in Fig. 14-16. Find Hi, just beyond the sheet. Ans. 1.7810 A/mm }. A traveling E field in free space, of amplitude 100 V/m, strikes a perfect dielectric, as shown in Fig, 14:17. Determine E, Ans. 59.7. V/m A traveling Kfild in free space strikes a partially conducting medium, as shown in Fig. 14-18. Given a frequency of SOOMHz and E,=100V/m, determine E%, and Hi, ‘Ans, 19.0V/m, 0.0504 A/mn 236 ELECTROMAGNETIC WAVES Icha. 14 Soto foo 056 14.40. A wave propagates from a diclectric medium to the interface with free space. If the angle of incidence is the critical angle of 20°, find the relative permittivity. Ans. 8.55 1441. Compute the ratios £3/E; and E4/E%, for normal incidence and for oblique incidence at @,= 10°. For region 1, €4=85, py=1, and o)=0, region 2is free space. ‘Ans. For normal incidence, E\E,=0.490 and EVE,=1.450. At 10°, E/E;=0.539 and BYE, = 1.539. 1442, A parallel-polarized wave propagates from air into a dielectric at Brewster angle of 75°. Find 6. Ans. 1393 Chapter 15 Transmission Lines (by Miton L. Ku) 15.1 INTRODUCTION Unguided propagation of electromagnetic energy was investigated in Chapter 14. In this chapter the transmission of energy will be studied when the waves are guided by two conductors in a dielectric medium. Exact analysis of this two-conductor transmission line requires field theory. However, the performance of the system can be predicted by modeling the transmission line with distributed parameters and using voltages and currents associated with the electric and magnetic fields. ‘Only uniform transmission lines will be considered; that is, the incremental distributed parameters shall be assumed constant along the line. 15.2. DISTRIBUTED PARAMETERS The incremental distributed parameters per unit length of line are inductance and capacitance as determined in Chapters 7 and 11, the resistance of the conductors, and the conductance of the dielectric medium. It was seen that the parameters depend on the geometry of the configuration, the characteristics of the materials, and in some cases the frequency. In the following summary list the dependence on geometry is represented by a geometrical factor cr. (Capacitance. C=xe{orc) (Fm) [e¢=permittivity of dielectric] Loy (Sim) [a conductivity of diclectric] Inductance (external). LL Be (crt) (Him) [ys = permeability of dielectric ~ uo] DC Resistance (useful for operation up to 10 kHz). Ry=—_(orr) (Qim) —_ [o, = conductivity of conductors} ‘Ac Resistance (or frequencies above 10 kHz). oe 208 R (cn) (aim) [5= = skin depth] 2 Vafice. Inductance (internal). p= [Rah im) for f> 10 “ Uwol4 (Him) for f <10kHz 237 238 ‘TRANSMISSION LINES ICHAP. 15 Inductance (total). Lebt+h~k, For three common line configurations the geometrical factors are as follows: Coaxial Line (inner radius a, outer radius b, outer thickness 1). = _ in(6/a) 1, ot Te+H Parallel Wires (radius a, separation d). orc for (>) 15.3. INCREMENTAL MODEL; VOLTAGES AND CURRENTS ‘The model in Fig. 15-1, where R, L, G, and C are as given in Section 15.2, permits analysis of the line using voltages and currents. For within a cell of length Ax the voltages across the line at points a and b differ by Av(s, 1)=(R Axil, () + Wan ed In the limit as Ar—0, this becomes ave) _ p. if, t) GD pate, 4 1 2D o ka La a wo, par Lae WW BOI $$ WW B00 —$§* SNOT og dew og dew -—— CHAP. 15} ‘TRANSMISSION LINES 29 Likewise, the current at point ¢ differs from that at b by Alte, 1)=(G Auer, + (C Ax) PED from which #e9_ = Gv, + ote » @) ‘The first-order PDEs (1) and (2) imply a single second-order PDE, 2a) RGflx, t)+ (RC + LG) LED. te D4 cP) one. 2» @ for either v(x, 1) or ix, ). Now, (3) is an equation of hyperbolic type, very similar to the wave ‘equation. Indeed, for a lossless line (R ), (3) is precisely the one-dimensional scalar wave ‘equation studied in Chapter 14. Thus it is known in advance that transmission lines support voltage and current-waves which can be reflected and/or transmitted at discontinuities (sites of abrupt parameter changes) in the line. 15.4 SINUSOIDAL STEADY-STATE EXCITATION ‘When the transmission line of Fig. 15-1 is driven for a long time by a sunusoidal source (angular frequency ©), the voltage and current also become sinusoidal, with the same frequency: VG, N=Re[P@IM] iz, )=Relix)e™) Here, the phasors V(x) and {(x) are generally complex-valued; often they are indicated in polar form (with the x-dependence suppressed) as P=WPiidy P= Lb where @ denotes the angle between the complex vector and the real axis. Steady-state analysis of the transmission line is much simplified when all voltages and currents are replaced by their phasor ntations. Figure 15-2 models in the phasor domain a uniform line of length ¢ that is terminated in a {comples ood Za the reeciving end and is riven a the sending end by a generator with internal impedance Z, and voltage V, = Vm, /6. The per-unit-length series impedance and shunt admittance of the line are given by Z=R+jol = ¥=G + joC Distance from the receiving end is measured by the variable x; from the sending end, by d. sending End Receiving End (Load) ie ip t * zy ie an 240 ‘TRANSMISSION LINES IcHap. 15 Equations (1), (2), and (3) of Section 15.3 become ODEs for the phasors P(x) and I(x). a0) = 2ltx) (1 bis) #e = Ye) (2bis) ho. =F) (3 bis) with y= VZY= a+ jB, the square root being chosen to make a and 6 nonnegative. Equation (3 bis) is identical in form to the equation of plane waves (Section 14.3); it has the traveling-wave solutions V(x) = Ve + Doe = Diac(x) + Vien) Mea) = Pre + Fe = hale) + heal) ‘The coefficients P*, etc., are phasors independent of x that are interrelated by the characteristic impedance Zy and the boundary reflection coefficient Cp, defined as Za-Zo Zn+Zo it follows that te Thay = Tye" = A= Boe Similarly, if Z(x)=V(«)/I(z) is the pointwise impedance looking back to the receiving end (=), then 1+1@) =Te@) ‘The conditions at the sending end [rerotate I(é), etc., as Ts, etc.] are Z(e)= Zor Average power received at the load and average power supplied to the sending end are CHAP. 15] ‘TRANSMISSION LINES 2a calculated as Pa=4Re (Dale) =4 Uni? Re (Zn) = Pra = 0) — Pres x = 0) Po=4Re (Pols) = 4 lls? Re (Zs) ‘Simplifications for High-Frequency or Lossless Lines. For frequencies such that R
    1 “| 8, fe] lowe} [a] Fig. 155 (6) Attoad: 2(0)=%+ ix; y(0)=g0+ jbo; VSWR(O)> 1 Before match: 2(x,) =r, +813 y(a1)= 81+ jb; VSWR= VSWR(0) After match: z(x,)=14j0; y(x,)=1+)0; VSWR The matching networks at lower (radio) frequencies can be made with lumped low-loss reactive ‘components; one lumped L-C network is shown in Fig. 15-6. If Zp has a reactive component, a reactance of opposite sign is added in series so that Zq=R+j0. Then, for a match, Y= JOC: + ——— IO RE jak, Ro VR(Ro-R) and If R>Ro, the capacitor should be connected to the other end of the inductor. ‘To minimize dissipation losses at higher frequencies a length of open- or short-circuited line is used for matching, in either a single-stub or double-stub configuration. 15.7 SINGLE-STUB MATCHING ‘The configuration shown in Fig. 15-7 uses one shorted stub, of length ¢,, placed at a distance x1 from the load. To accomplish matching: (1) Determine xy such that y(x1)=1+ jb. (2) Determine ¢ such that y(é,)=0— jb, . After matching, y(x,)=14j0 and VSWR=1 from.x, to é EXAMPLE 1. The above two steps may be accomplished on the Smith Chart (Fig. 15-8). (Plot yx and trace the [Fa [or VSWR(O)] cite i) Mark the intersections of the [Tal circle and the circle = 1. ) From yq move toward the generator to the first intersection, read y=1+)by, and note the distance x, asa fraction of A (or read off angle 26:x,)- CHAP. 15} ‘TRANSMISSION LINES 245 (iv) Mark the point y=0-j, on the |f}=1 circle. From the short-circuit position y= ‘=, move toward the generator to the point y=—jb,. Note the distance ¢, as a fraction of A. If the first intersection is not accessible, the second one can be used by readjusting the stub length for the susceptance at the new position. For matching two cascaded lines with different characteristic impedances the above procedure is used at the connection point where the equivalent load is the input impedance to the second line. 158 DOUBLE-STUB MATCHING ‘A double stub “tuner” has two shorted stub lines separated by a distance d, on the main line, as shown in Fig. 15-9. Stub 1 is nearest the load and frequently is connected at the load (x=0). Common separations for the two stubs are A/4 and 34/8, hence the names “quarter- wavelength tuner,” etc. The Smith Chart solution for the two-stub matching problem involves the ‘construction of the tuner circle for the given d,. This is the circle gr=g(d,)=1, which plays the ‘same role for stub 2 as the g=10 circle plays for the main line. ‘The tuner circle is obtained by clockwise rotation of about the center of the chart, 1-+ 0: a rotation of 180° gives the A/4 tuner circle, rotation of 90° gives the 34/8 tuner circle, etc. See Fig. 15-10. 246, ‘TRANSMISSION LINES IcHap. 15 Tew Fig. 15.9 EXAMPLE 2. Assuming that d,~/4, a five-step sequence is used for two-stub matching, as shown in Fig. ASL. 1. Plot the 4/4 tuner circle. 2. Mark the intersection(s) of the tuner circle and the gq circle through the entry point ye= fat iba. Read by at this point, which may be either intersection. 3. Stub 1 at x=0 is used to change the susceptance bs to br. 4, From yz=8e+jbr move on the [Fpl circle a distance d,= 2/4 toward the generator onto the g=1 circle and read y=1+b, 5. Cancel the susceptance b, by adjusting stub 2 to produce y= 14/0, the matched condition. A problem arises when using the 4/4 tuner to match a load with ge>I, since the ‘conductance circle does not intersect the tuner circle. The 34/8 tuner works for some values of g>1. In any case, a tuner can be displaced from the load by a distance x, to put g in the range for matching. Should g=1 at the displaced position, the single-stub condition holds and stub 2 must be set for 6 =0. CHAP. 15] ‘TRANSMISSION LINES 267 Fig. 15:11 Note the standing waves on the various sections of The shorted stubs each have VSWR=<. For d,a E 1 or=in(S)=s0 Grc= 77 ~0.0 GFR, == 4.7210 mr? GR, =2=971 mr For sir dielectric, y= He and €=€y5 for copper, He~Hy. Hence: He = ‘im 1 = 3 O/m 1 =H#(GRL)=2.0 unt Ry a3 (GFR,) =2.59% 10° Os C= ne,(GFC) = 5.56 pF/m b= 6 Vatu. *™ 1 2x60. G=0S/m (GFR,) = 4.04% 10-7 O/m 18.2. The specifications for rigid air-dielectric coaxial line used in a radar set operating at 3 GHz #25 are: copper material, stub-supported at intervals to maintain the air dielectric; outside FY diameter, J inch; wall thickness, 0.032 inch; inner-conductor diameter, 0.375 inch; charac- teristic impedance, 46.4; attenuation, 0.0664B/m; maximum peak power, 1.31kW; operating peak power, 200 kW; lowest safe wavelength, 5.28cm. Determine the per-meter values of L, C, G, and R, for the line, neglecting internal inductance. CHAP. 15] ‘TRANSMISSION LINES 251 ‘The inner radius a is 4.7mm and the outer radius 6 is 103mm. Then In(b/a)= OTM, GFL =0.386, oFc= 2.59, fon) =0AS¢pHim Co neore)=71.99t/m For copper and a frequency of 3GHz, 8=1.2ym. Then, omen be bezotm! and Rem 5-2 5 (crm,) =0.7020/m For sir disectric, G=0S/m. 15.3. Show that the voltage u(x, t) =A cos (ot + 8)e!® satisfies the transmission line equation 3), for a uniform lossless line, if B= VEC. For the lossless line, R=G=0, so thatthe equation reduces to Fouls.) -Fulst) Tae ae For the given voltage, this requires -B=LC(-wv) oF 15.4, For the parallel-wire line of Problem 15.1, find the characteristic impedance, propagation constant (attenuation and phase shift), velocity of propagation, and wavelength, for ‘operation at 5 kHz, At SH the de resistance may be used. 59x 10°? + j2x(5 x 10°)(2 10-4) = 6.289 x 10-*/87.6" Bim Y= G+ joC= jan(5 x 1OYS.56X 10") = 1.747 x 10-7 [9S Im ane 600-120 (y= VZV = 1.088 % 10 /88.8° = (2.19 10-4) + (1.048 « 10-4) Then @=2.19% 10-°N/m, B= 1.048x 10-*rad/m, u, = w/f =2.998% 10° m/s, 2=2n/f-= 59.95km. 158. A 10-km parallel-wire line operating at 100kHz has Zo=S572, a=2.4x 10-"Np/m, 38 and B=2.12%10-"rad/m. For a matched termination at x=0 and Vp=10/0°V, a evaluate V(x) at x-increments of 4/4 and plot the phasors. ‘The line is matched at the receiving end, so that Ty=O and V(x)= Pre /f. But V()=0* =%q= 10/0°V whence (x) = 10" 1px (V) For x=n(W/4) (n=0,1,2,...,13,13.48), where n= 13.48 corresponds to the 10-km length, ex = (fx) = n0.0178Np) By use of these increments, Table 15-2 is generated. A polar plot of the tabulated results is given in Fig. 15-18. 252 ‘TRANSMISSION LINES Table 152 ‘Quarter Wavelengths | Bx =v, We. from Load ee. | ax.Np | Vv ° 0 | oo 10.00 1 9 | aos | to. 2 10 | 0.0356 | 10.36 3 zm | 0.0536 | 10.35 4 300 | oom | 10.74 5 450, | ose | 10.93 6 sao} 2067 | 1113 7 630 | ozs | 1133 8 mo | os | 1153 9 810 | oto | 1174 10 900 | o1m | an9s u 990 | 01956 | 12.16 2 woo | 02134 | 12:38 B tm | 022 | 12.60 B48 ris | 02 | 27 18.6. Repeat Problem 15.5 if a mismatched load results in Ti [cHaP. 15 0.4 /0°; all other data remain sé Ropes ae Mrcun: to Pobem 55, he ages mw en bye sprain oan nen ana reflected wave: vay ‘The boundary condition at 10/0 or n(x) + gl) = Oe [Bix + TV em ™ [Bie 0 gives (omitting physical units) 40" 14 10° Thus V(x) = 7. 1de™ [Bx + 2.866" [Be CHAP. 15] ‘TRANSMISSION LINES 253, 18.7 Teble 15-3 trom | ax, Load | Np | et | fr Vel) Vud®) vey o [oo o | z14yo 2.86/0° 10.00 /0° 1 | oor78 go | 7.27; 281/90" 4.46 [90° 2 | 0.0356 weer | 740/teor | 2.76/=1 10.16 fia" 3 | o.0s34 zoe | 7s3yzr | 2n;-20F | 4.82/-90 4 | oom 360" | 7.67/36" | 2.66/36 | 10.33/0° 5 | 0.0889 aso | 7.90/50 | 262/=45 | 5.18/90 6 | 0.1067 sar | 79458 | 257/=540° | 10.51/18 7 | ors er | sosjear | 2.52/= $.57/= 8 | 01423 ne | spn | 248j=7F | 10.njr 9 | ote. ser | 83878ir | 2447-81 | 5.94/90" 10 | 0.1779 soo | 8.53 239/-90" | 1092/18" ni | 0.1956 sor | goer | 235-99 | 6.33 /-90 x2 | 0.2134 soso” } 8.g4/toe | 2.31 /=r0ar | 1115/F 3 | 02312 ure | 9.00/17 | 227/17 | 6.73/90" 13.48 | 0.24 raise | 9.08/i2is* | 2.25/=1215° | 9.34 /148.5" where a and 6 are as specified in Problem 15.5. The required calculations are presented in Table 15-3 and Fig. 15-19. Fig. 15.19 Measurements are made at 5kHz on a 0.5-mile-long transmission line. The results show that the characteristic impedance is 94 /-23.2° 2, the total attenuation is 0.06Np, and the phase shift between input and output is ®. Find the R, L, G, and C per mile for the line; the phase velocity on the line; and the power lost on the line when the sending-end power is 3.W and the load is matched. ‘The measured attenuation is_a¢=0.06Np, whence a=0.12Np/mi, the phase shift 8 =0.14rad, so that B=0.28rad/mi. Hence, VZV = y=0.1240.28=0.305 /66.8mi' or ZY = From this and the measured value VZ/¥ = 94 /—23.2°@: Z-= 267 [43.6 = 0.8 + f19.8.0/mi = R+ j2afl ¥=3.24X 107 /90°=/3.24» 10 *S/mi= G+ afc which imply: R=20.89/mi; L=60uH/mi; G=0; C=0103 4E/mi. 093 133.6 mi-* 158, 159. a ‘TRANSMISSION LINES [cHap. 15 ‘The phase velocity isu, = 2nf /B=27(5 x 10°)/0.28= 1.12% 10° mils. For a matched load (no feflections), the received power is given by P= Fee Pf = 30 = 2.66W and the power lost as the result of attenuation is 0.34 W. ‘A 600-2 transmission line is 150m long, operates at 400kHz with a=24x 10? Np/m and i= 0.0212rad/m, and supplies & load impedance Za = 424.345". Find the length of line in wavelengths, Tx, Ts, and Z,. For a received voltage Va = 50/0°V, find Vs, the position on the line where the voltage is a maximum, and the value of (Phease Because A=2n/B=296.4m, €=150m=051%. At x=0, Zn—Zo_ 300 + j300- 600 Za Zo 300+ 300 + 600 T, "= 0.45 (116.6 = ~0.2+j0.4 Therefore, at x= ¢, A Wale [de — Be = 0.486 *” [116.6 — 365° =0.20/113.6 = -0.09 + 020 140) 914/02 and 2A E) = (Feng) “27 BLO From Vq=50/0=V*(14T,), 0 =56.2/-266°V. Then, 7 e* (BOIL + Fe) = (56.2% /-26.6" + 181.55]0.91 + 0.2} =75.0/167.3"°V “To find x where the voltage is a maximum, construct the phasor diagram Fig. 15-20. At 0. the incident and reflected voltages are separated by an angle of 116.6%. When Vy. rotates $8.3" counterclockwise and Vag rotates the same angle clockwise the two phasors add together. The distance x for which fx'= 583° is 48.2m, the position of the maximum. The magnitude is Thane = 56.22" + (0.45)(56.2)0-0""4= 85.5 V For the coaxial line specified in Problem 15.2, determine the actual characteristic impedance and attenuation, and compare the values with the specifications. Determine the length of the shorted stub required to support the center conductor at the operating frequency of 3 GHz and calculate the highest ‘safe” frequency of operation for this line from the specifications. “The characteristic impedance for the high-frequency low-loss line is a 46.33 (specification is 46.49) cuap. 15.10. z 5] ‘TRANSMISSION LINES 255 “The per-meter attenuation is an Pa, 0M Ie 206.33) where the conversion 1Np=8.6864B_ has been wsed 'A stub to support the center conductor must be 4/4 long so thatthe short circuit reflects to an open cteuit atthe point of connection to the mainline. At 3 GHE the length should be ax 10°) “The “safe” highest frequency of operation is determined by the specication for lowest “safe” wavelength = 7.58% 10" Np/m=0.06584B/m (specification is 0.066 4B/m) =0.025m or 25em 3x10 te 68 GHz At frequencies above this value, propagation modes other than the TEM could exist A 70-9 high-frequency lossless tine is used at a frequency where 1=80cm with a load at x=0 of (140+j91)Q. Use the Smith Chart to find: Tx , VSWR, distance to the first voltage maximum from the load, distance to the first voltage minimum from the load, the impedance at Vaux, the impedance at Vin. the input impedance for a section of line that is ‘S4.em long, and the input admittance. ‘On the Smith Chart plot the normalized load Zp/Ro=2+/1.3, 2s shown in Fig. 15-21. Draw ‘2 radial line from the center through this point to the outer A-circle. Read the angle of I on the angle scale: ,=29°. Measure the distance from the center to the z-point and determine the magnitudes of I, and VSWR from the scales at the bottom of the chart. Wal =0.50 VSWR=30 and Ty=0.5/29° Draw a circle at the center passing through the plotted normalized impedance. Note that this rele intersects the horizontal line at 3+ 70. This point of intersection could be used to determine the VSWR instead of the bottom scale, because the circle represents a constant VSWR. Locate the intersection of the VSWR circle and the radial line from the center to the open-circuit point at the right of the z-chart. This imtersection is the point where the voltage is a maximum (the current is a 03K, Mig. 15:21 15.1. ‘TRANSMISSION LINES (chap. 15 minimum) and the impedance is a maximum. The normalized impedance at this point is 3+j0, whence Zna=210+ j0Q. To find the distance from the load to the frst Vea, use the outer scale (wavelengths toward the generator). The reference position is at 0.214 and the max. line is at 0.25 A; s0 the distance is 0.044 toward the generator, or 3.2em from the load. From the Vna Point move 0.25) toward the generator and locate the Vai point. The normalized impedance is 0.330, and Zyuq™23.1 + j0. The distance from the load to the first minimum is (0.254 40.044 = 0.294 =23.2em “To find the input impedance, move #=0.675 wavelength from the load toward the generator, and read the normalized impedance. Once around the circle i 0.58, x0 locate the point that 0.1754 from the load on the ovter sale. The point is at 0.214 + 0.175A~0.385A. Through this point draw 2 radial ine and locate the intersection with the VSWR circle. The normal impedance is 0.56 /0.71 and Z,,=39.2— 7.72. ‘The normalized input admittance is located a diameter across onthe chart, which corresponds to the inversion of a complex number. For 2=0.56~j0-71, y=0.68+ 0.87; therefore, y Yo 2 = 0.714)12.8) ms Zr ons) ‘The high-frequency lossless transmission system shown in Fig. 15-22 operates at 700 MHz with a phase velocity for each line section of 2.1 x 10" m/s. Use the Smith Chart to find the VSWR on each section of line and the input impedance to line #1 at the drive point. (There are three distinct transmission line problems to be solved.) Zq> 1/0. 2u= Mo] 00 Fig. 15-22 FFor the three lines the wavelength is A= (2.1 10°/(7%10")=300m. For line #2 the length is (43.5/30)A= 1.454 and the normalized load is (0+ /70)/70=j1. Plot this value as point 1 in Fig. 15-23. Note the reference position, 0.1254 and VSWR==. Move on the VSWR circle 1.45 toward the generator to point 2 and read the value 2q=0+j0.51. The input impedance to line #2, Zoua= 2nRea = 0+ 735.7 is one part of the load on line #1 For line #3 the length is i} =0.7A and the normalized load is (40+ j0)/90=0.44 +0. Plot this value as point 3 and note the reference position of OA and the VSWR=2.25. Move on the “VSWR circle 0.74 toward the generator to point 4, and read off 2g L624 70.86 OF Zi ™ aR ‘This is the second part of the load on line #1 45.84 j77.4 CHAP. 15] ‘TRANSMISSION LINES 257 Fig. 15:23 For line #1: the length is 1.25/0.30=4.167A__and the load is the parallel combination of Zs and Zs. Normalize each impedance to the 50-9 line, find each admittance, add the admittances for Jn and then find zy. nai(22) 047078 (pans) and y.=0-/L41- (pints BSBA oop. j1.55 (point) and y4=027-14 (pint Y= 0.27-f1.55 (poim9) with VSWR=14 Invert by moving a diameter across to point 10 for zq=0.1+ 0.63. at the reference position 0.092. Now move 4.167A toward the generator from 2, on the VSWR=14 circle to point 11, and read 24=9.5~/63. The input impedance to line #1 is 5019.5 ~ 6.3) = 475 -j315 (a) A high-frequency S0-Q lossless line is 141.6cm long, with a relative diclectric constant ¢,=2.49. At 500MHz the input impedance of the terminated line is measured a8 Zq= (20+ j25)@. Use the Smith Chart to find the value of the terminating load. (6) After the impedance measurement an 8-pF lossless capacitor is connected in parallel with the line at a distance of 8.5m from the load. Find the VSWR on the main line. (@) For «,=249, 3x10" a L9xt0'm/s ante UE. and the length of line is (141.6/38)4=3.726A. The normalized input impedance is 2. (20+ j25)/50=0.4 + 0.5. Plot this value on Fig. 15-24 as point 1, measure the VSWR, draw the VSWR=3.2 circle, and note the reference position at 0.4181 coward the load. From 2 move 3.7264 toward the load on the VSWR circle (a net change of 0.2264) and read the normalized load impedance 2,=0.72—j0.98 at point 2. ‘The load impedance is Z_ = (36— 749) at S00 MHz. (®) Since the capacitor is connected in parallel it is convenient to work on the y-chart, Fig. 15.25. In Fig. 15-24 read the value diametrically opposite zp: ya =0.48+/0.67. Plot yx as point 3 in Fig. 15-25 and draw the VSWR=3.2 circle. The reference position is 0.1054 foward the generator, corresponding to x=0. Move &Scm, or (8.5/38)A=0.2244 toward the 258 ‘TRANSMISSION LINES [CHaP. 15 Fig. 15-24 Fig. 15-25, generator on the VSWR circle and read y(x;) at point 4. Before the capacitor is added, y(x,)=1.04—j1.22. The normalized admittance of the capacitor is Ye = U2MfCYRo = jm (5 X IVS X 10°7)(50) = 0+ j1.26 land the new admittance at x; is y.+y(x;)= 1.044 /0.04. Plot this mew admittance as point 5 and measure VSWR=1.04 (a significant reduction from 3.2). 15.13. A 4-m-long, stub-supported, lossless, 300.2, air-diclectric line (Fig. 15-26) was designed for operation at 300 MHz with a 300-Q resistive load, using shorted (2/4)-supports. With no ‘changes in dimensions or load, the line is operated at 400MHz. Use the Smith Chart to find the VSWR on each section of line, including the supports, and the input impedance at the new frequency. t . tae 028 a Fig. 15:26 ‘The wavelength is A=u,/f = (3 107)/(4 x 10") =75em and distances in terms of 4 are Total length = 4m = 5.3334 Load to stub 1 = S0cm = 0.6674 ‘Stub length = 25 em = 0.3334 ‘Stub separation = 2.5 m= 3.3334 ‘Stub 2 to input = 1 m= 1.334 ‘Since the stubs are in parallel, use a y-chart, Fig. 15-27, for the solution. At x=0, yq=RolZn= 1+j0 (point 1) and VSWR=1. The line is flat to the point of connection of the first stub, (0.667 from the load, with y(x,)=14+ 0. in the absence of stubs. CHAP. 15] ‘TRANSMISSION LINES 259 15.14. we 96 s wl Fig. 1527 To find the admittance of the shorted stubs, plot yc at point 2, move 0.3334 toward the generator onthe VSWR== circle to point 3, and read y=04+/0.58. This value must be added to y= 14/0 to get the admittance at x, with stub 1 connected; thus, y(x,)=1+j0.58 (point 4), VSWR 1.75, and the reference position is 0.148A toward the generator. Draw the VSWR Circle through point 4; move 3.3334 toward the generator from 4 to 5; and read the admittance at x, without stub 2 in place: y(x,) =0.57~j0.08 At this point add the second stub to get _y(x.)= 0,57+ 0.50 (point 6) draw the VSWR=2.3 circle. ‘The reference position is 0.092A toward the ‘generator. From point 6 move 1.333A toward the generator on the VSWR=2.3 circle to point 7 ‘and read the normalized input admittance y,,=0.52—j0.38. Invert this value by moving across a diameter and read_z,=1.23+)0.92 (point 8). The input impedance to the line at 400 MHz is Ry = G69 + 7276) 2 ‘The lossless lumped-parameter network shown in Fig. 15-28 is used to match a S0-@ tine to the input of an RF transistor operating at 1GHz. ‘The input reflection coefficient for the transistor is T= 0.6/~150°, measured for a 50-Q system. Find the values of L and C for the conjugate matched condition. I t cea) Ld on ig. 15-28 Normalizing the reactances of the matching network to the SOQ line gives x= @L/50 and 6=S0@C. ‘The normalized impedance looking back to the network from the transistor 1 OE o Now, the matching criterion is T,=T*, or ar -r Bq een 0.27 + 0.25 @ 15.15. ‘TRANSMISSION LINES IcHap. 15 together (1) and (2) yield b= £1.64. For b=+1.61, x=+0.70 where the positive sign on b corresponds to a capacitance. Then ant aos ont ‘A 15m length of 300-Q tine must be connected to a 3m length of 150-Q line that is terminated in a 150-Q resistor. Assuming the lossless condition for the air-dielectric lines and operation at a fixed frequency of S0MHz, find the Re and the length for a quarter-wave section of line (quarter-wave transformer) to match the two lines for a VSWR=1 on the main line. If no transformer is used, what is the VSWR on the main line? ‘A model for the system is shown in Fig. 15-29. For f=SOMHz and u,, wavelength is 4=6m; a 4/4 section of line must be 1.5 m long. 3x10" m/s, the Ra= 1500 Tom Ineo Fig. 15.29 With no transformer in place the tert Reterminated. The reflection coefficient on tion of the 300-0 line is 1500, since line 2 is ine Lis = 10-300 “+O, Fsjor3007 "3 amt VSWROT =? With the transformer inserted as shown, the reflection coeficient atthe load Re is a= Ror Rake @ and the input impedance at r=", which, asthe load on ine 1, must be 3002, is 14Ts 2q= 9000 Ror ate ‘Substitution of (1), with Rp = 150.Q, in (2) gives 0(150 + Ror + 150 ~ Ror) = Ror(150+ Ror ~ 150+ Ror) or Rey = Vi00X 150 = VRRe; = 212.1 2. @ A generator at 150 MHz drives a 10-m-long, 75-Q coaxial line terminated in a composite load consisting of the parallet connection of two 50-Q lines of lengths 0.5m and 1m, each terminated in a SOQ resistance. Alll lines are lossless with ¢=2.2. With reference to Fig. 15-30, determine the length ¢, and connection point x, of a parallel-connected 75-Q stub that will produce minimum VSWR on the feed line. ‘The stub should be as close as possible to the load. Phase velocity, u,=3 x 10°/V22 = 2.02 x 10 m/s; wavelength, 4=u,/f =1.35m. The input impedance to each of the 50-2 lines is 502 for R,-termination, the composite load on the 75-2 line is 251 or, when normalized, z=0.333+ j0. Plot ze on the Smith Chart and through this point draw the [Tl circle and the radial line to the angle scale. as shown in Fig. 15-31. Read y= 18" and I-05 and VSWR=3 CHAP. 15] ‘TRANSMISSION LINES 261 15.17. Za: = 500 Since the matching stub is in parallel, locate yq_=3+ 0 by projecting a diameter across from 2. Locate the intersections of the g=1 and VSWR=3_ circles and note the distances from the load at x =0: +]11S ata distance 0.25 +0.166 = 0.6164 toward the generator y=1-j115 ata distance 0.3335 0.25=0,08354 toward the generator Locate the 75-2 stub at x,=0.08354=11.3em. The stub length is that which makes y,— O+/1.15, for a net y=1+70, (matched condition). Thus, plot y, and determine the distance from the short-circuit condition y=s> to this point. The length of stub is (0.25+0.1345)A= 51.9em. ‘The VSWR on lines 4 and 4; is 1.0 for the matched loads. On the shorted stub the VSWR is infinite. From x=0 on the main line to x=x, the VSWR is 3.0 and from x=x, to x= 6 the VSWR is 1.0. y Find the shortest distance from the load and the length (both in centimeter) of a shorted stub connected in parallel to a 300-2 lossless air-dielectric line in order to match a Toad Zp = (600+ j300) 2 at 600MHz. The matching stub is the same type of line as the main line. a ‘TRANSMISSION LINES [cHar. 15 For both the line and the stub, u,=3x10"m/s and A=0Sm. Plot zn=(600+ 1300)/300= 2+ 1 on the y-chart, Fig. 1532. Draw the VSWR=2.6 circle, move diametrically ‘cross to yp =0.4—j0.2, and read the reference position 0.4644 toward the generator. Move for ‘yn on the VSWR circle to the frst intersection with the g=1 circle and read y(x,)=1+j1 at the reference position 0.1624. The stub location is. x; = [(0.5 ~0.464) + 0.162]4 = 0.1981 =9.9 em, from the load. To match the line for VSWR=1, the admittance of the shorted stub must be y,=0~j1 to cancel the susceptance at point x;. The required length of stub is 0.1254= 625em, If this position is not accessible, the second intersection with the g=1 circle may be used, =jl and 22= [(0.05 — 0.464) +0.338]4 = 0.3744 = 18.7em ‘The stub would have to be adjusted to give y,=-+/1, for a length of 0.3754=18.75em. A high-frequency lossless 70-2 tine, with «,=2.1, is terminated in Zp= 50/302 at 320MHz. The load is to be matched with a shorted section of S0- line, with ¢,= 2.3, connected in parallel; the stub must be at least Sem from the load. If such matching is possible, find the distance from the load and the length of the stub. For the main line, u, =3X10'/VI1=2.07X 10 m/s, A=uj/f=64.7em; for the stub line, uy, =3 x 10/VZ3 = 1.98 x10" m/s, A,=61.9em, The normalized load is 2— = (50/30)/70= 0162+ /0.36, with VSWR=1.92, and the admittance is y»=1.20~ j0.70 at reference position 0.3272,, toward the generator; see Fig. 15-33. Move on the VSWR circle from yx toward the generator to the frst intersection, y(s,)=1~ 0.66, at 0.3504, of a distance of 0.023.= 1.49 em. This point can not be used due to the S-cm limitation. "Continue on the VSWR circle to y(e,)=1+j0.66 at postion 0.1512,,; the distance 22= ((5=0.327) +0.151 = 0.324, =21.06m {ives the point of connection for the stub. ‘As the stub has a different Ro, itis necessary first to “denarmalize” y(x,): 1+ j0.66, a ¥On)= = (1.4-+ 0.94) x 1077S CHAP. 15] ‘TRANSMISSION LINES 263 Sut 043-0.25= 0.184 93 Fig. 15:33 ‘which shows that for cancellation of susceptance we must have Y= (70.94 x 10-)(50) = —0.47 ‘The length of shorted stub is then (0.43 ~0.25)2, = 0.188 = 11.1.em. 18.19. A complex load is measured with a VHF bridge at SO0MHz; the impedance is 29 [30° Q. This load is connected to a S0-Q air-dielectric line, with a 50-Q 32/8 tuner between the load and line. Find the lengths of each shorted stub to produce a VSWR of 1.0 on the main line. Show both solutions if they exist. ‘The model for the system is shown in Fig. 15-9. For the air line, u,=3x10'm/s and A= u,/f = 60cm. The normalized load impedance is = 0,58 /30" = 0.5 + 0.29 Re On the Smith Chart draw the 3i/8 tuner circle, plot za, draw the VSWR=2.25 circle, locate yq=1.52—j0.88, and find the intersections of the tuner circle and the ge= 15.20. ‘TRANSMISSION LINES (CHAP. 15 1,52 circle. There is a solution for each intersection; first consider y= 1.52~j1.82 (Fig. 15 34a). Here, the first stub must be adjusted to change the susceptance from ~0.88 to —1.82 (point 1); thus y,,=0~j0.94 at point 2. ‘The stub length for this b is read on A scale from y =o toward the generator: (0.380 -0.25)A = 7.8 em_ From point 1 move 32/8 toward the generator to point 3, where y=14/1.53. Stub 2 must add. y'=0-j1.53 (point 4); and the stub length is 617 (0342 ~025)A=5.52n Figure 15-340 presents the second solution, which follows the same pattern. AK: y= 152-7016 RZ: yy =04)0.72 and 2, = 0.25 +0.099)A=21.6em ARs y= 14/045 Ata’: y= 0-70.45 and 5 (0433 —025.= 11.dem “The first solution is preferred because the total length of the stubs with infinite VSWR is 12.63 em, ‘which wil introduce lower losses in a practical system. Use a two-stub quarter-wave tuner (50-2, shorted stubs) located 7.2m from the load of Problem 15.19 in order to match the load to the line. ‘The normalized load admittance in Problem 15.19 is ye =/zq=1.52— 0.88, and the wave- length is A= 60cm. At 7.2cm or 0.12 from the load, "y,=0.54—j0.36; this valve is to be ‘matched to the line with the tuner. ‘Two solutions exist, Solution 1 (Fig. 15.35) Solution 2 (Fig. 15-36) = 0.54 ~ 70.36 y= 0.54 ~ 0.36 As yp = 0.54 70.50 Vs yy 0.54 470.50 % ye jos 2s yy 4/086 = (0478 ~ 0.25) = 13.68em 6, = (0.25 +0.113)A = 21.78.em 3 Move A/4 to y= 1470.95 32 Move 4/4 10 y=1~j0.95 4 ya= 70.98 42 y= +j095 2 (0.3795 -0.25)A=7.71em 67 (0.25 + 0.1205), = 22.23em ‘Total stub length = 21.45 em (preferred) “Total stub length = 44.01 em CHAP. 15] ‘TRANSMISSION LINES 265 15.21 158.22. ‘A. 70-Q double-stub tuner is used to match a load Yp = (4.76+/1.43)mS at 600MHZz to a 70-Q lossless air-dielectric line. The first stub is located at the load and the separation between the stubs is 10cm, Find the shorted-stub lengths for the matched condition. For the air line, 2=(3% 10)/(6X10")=0.50m and the stub separation is 10em= A/S. Draw the A/S tuner circle as shown in Fig. 15-37. Plot the normalized load. ys = YeRo= 0.334 0.10, which determines the VSWR=3.0 circle. Two solutions exist, one for the intersec- tion with the tuner citcle at y,=0.333—j0.18 (VSWR=3.2) and the other at y,=0.333+ f084 (VSWR=49). Fins solution. y3,= ~/0.28 to change ye to y1, for a length of 0.2072 = 10.35em. Move on the VSWR~32 circle 0.24 toward the generaior from yx, 10 y= 1.0+/1.23. The second stub must be adjusted to give ya=—/1.23 fora net y=1+/0, the matched condition. ‘The length i 01091 =5.45em. 4/074, for y= 0.33+/0.84 and a length 0.3514 =17.5Sem. 3 citcle 0.2% toward the generator from J, 10. y'=1.0-j1.75. The second stub must be adjusted for “1.75 to produce 140, the matched condition. The length is 0.417. =20.85em. ‘A50-Q slotted line that is 40.cm long is inserted in a 50-Q lossless line feeding an antenna at 6OOMHz. Standing-wave measurements with a short-circuit termination and with the antenna in place yield the data of Fig. 15-38; the scale on the slotted line has the lowest Min Min mem 3em Ss High VSWR 1 I Max Min mse Bem ——-_ g VSWR = 22, _ Fig. 15.38 266 ‘TRANSMISSION LINES IcHap. 15 ‘number on the load side. Find the impedance of the antenna, the reflection coefficient due to the load, and the velocity of propagation on the line. For the short circuit, minima are separated by a half-wavelength, so A= 50cm. For a frequency of 600MHz the phase velocity is u,=fA=3% 10'm/s (air dielectric). With the antenna in place the minimum shifts Sem=0.1A toward the generator. On the Smith Chart draw the VSWR = 2.2. circle and identify the voltage minimum line as in Fig. 15-39. Locate 25 on the VSWR circle (0.12 toward the load from the Vaie postion: Zn =0.64—j0.52 and Zy= Rote = (32/26) 2 ‘The load Py is read from the chart: @q=—108° and {T'x|~0.375 is the distance from the center (0 zp, as Fead off the external scale. Fig. 15-99 15.23. A 40-m length of lossless 50-Q coaxial cable with a phase velocity of 2 x 10" m/s is connected at £=0 to a source with u(t)=18Vde and R,=100Q. If the receiving end is short-circuit terminated, sketch the sending-end voltage us(t) from t=0 to t=25ys. ‘The delay time of the line is fo 4, Dae ‘The incident voltage at the sending end at ¢=0 is Ro 18% 50 vd)" Re” 100+ 50 “The reflection coefficients at the two ends are y= Ro_ 100-50 1 = Rea Ro 100-90 41 Te RR, isso” 43 T* Figure 15-40 shows the ¢=d plot over a total time of 2.5 4S= 12.Stp. From this, the desired v, plot, Fig. 1541, is easily derived. At any time v, isthe sum ofall incident and reflected waves present at d=, upto and including the last-created incident wave. For example, to = 0.2 us 22 W(A.lip) = 6-6-2424 5 =5 On account of Te=—1. the waves preceding the last incident wave cancel in pairs. CHAP. 15] ‘TRANSMISSION LINES 267 % 2 Fig. 15-41 15.24. A well-designed, lossless, 100-2, 100-us delay line produces a good 10-us pulse at the ‘output 100 us after it is driven at the input, at ¢=0, by a 10-ys rectangular pulse recurring with a period of 2ms. ‘The generator has a 9'V peak open-circuit output and an internal resistance of 50@. Sketch us(‘) and ua(t) from 1=0 to 1=650ys if the termination is a 50-Q resistor. At (20 the sending-end incident voltage is pulse of 10-ns duration with a peak value of (100) ‘50+ 100 ‘The senvding-end and receiving-end reflection coefficients are 50-100 1, = Pa= Rew, == R,+Ry 50+ 3 Ry + Ry 50+100~ 3 Since the pulse period is 2000 us, only the pulse sent out at_¢=0 need be considered in the 1=4d plot of Fig. 15-42, which covers only the first 650 ys. Applying to Fig. 15-42 the summation technique described in Problem 15.23, one obtains the required voltage plots, Fig. 15-43. Vou =: =6V 48) 15.25. 1827. 528. 1529. 150. 1531, ‘TRANSMISSION LINES. [cnap. 15 fat uy 1 sinn Big. 15-43 Supplementary Problems ‘A coaxial cable with the dimensions @=0.5mm, b=3mm, and 1= dielectric materiel having €,=20, 0,=10S/m. “The conductors have 0, the permeter values of 1, C. G, Ry. and R, at SOMEz. Neglect internal inductance. Ans. 0.358 eH/m, 62.0 pE/m, 35.1 4S/m, 0.030 Q/m, 0.743 2/m. Find the per-meter values of L, C, G, and R for a parallelwire line constructed in air of #12 AWG copper wire (dia.=0.081 in., ¢.=52.8MS/m) with a 4-inch separation, Operation is at 1OOKH2. Ans. 1.84 uEi/m, 6.05 pF/m, 0, R, = 0.0268 2/m In a “twin-lead” transmission line, two parallel copper wires (0, =50MS/m) are embedded 0.625iin, apart in a low-loss dielectric with ¢=24 Neglecting losses, determine the diameter of the ‘conductors for a characteristic impedance of 300. For this size of conductor, find the de resistance and the ac resistance at 100MHz. Ans. 0,026in. =0,66mm, 0.117 /m, 2.72.Q/m A high-frequency application uses a coaxial cable with copper conductors, where the diameter of the inner conductor is 0.8mm and the inside diameter of the outer conductor is 8.0mm. The dielectric material has ¢,=2.35, and the thickness of the outer conductor is much greater than the depth of penetration at the operating frequency. The engineer wants to use a new cable having the same Ry , bbut with a larger outer conductor such that b,~a~=1,5(b,~a). Find ¢, for the new cable and. calculate Ry and the capacitance per meter for each cable, Ans. 3.18, 90, 568 pF/m (old), 66.1 pF/m (new) For the cosxiat cable of Problem 15.28, calculate the line chorateristcs Ze a, tps for operation at WOKLI2, Ans, 32.7 /15.2°, 1.04% 10 *Np/; 4.2% 10 * rad/m, 1.49 10 m/s, 14.9 km Find the characteristic impedance, propagation constant, velocity of propagation, and wavelength for the parallel-wite line of Problem 15.26. Ans, 582.5 /-0.65°Q, (24+ j210)10-* m', 2.99% 10" m/s, 2.99 km A transmission line is 2 miles tong, operates at 10 kHz, and has parameters R=30Q/mi, C= SOnE/mi, L=2.2mH/mi, and G=20nS/mi. Find the characteristic impedance, attenuation per CHAP. 15] ‘TRANSMISSION LINES 269 rile, phase shift per mile, phase velocity, and wavelength. What is the received power to a matched oad when the sending-end power is 1.2 W? ‘Ans, 167.7 [6.1 Q, 0.0896 Np/mi, 0.838 rad/mi, 7.5 x 10' mi/s, 7.5 mi: 838.6mW 15.32. A transmission line 250m long operates at 2MHz with a load impedance of 2009. The line characteristics are Z,=300/0°O, @=4%10"*Np/m, P=0.06rad/m. If the sending-end voltage is 30 0° V, find the receiving-end voltage, power to the load, sending-end current and power, and the reflected power from the load, Ans. 22 /—130" V; 1.21 W; 105.4 /18.3° mA, 1.5W; SOmW 15.33, One method of determining the characteristics of line is to measure the input impedance at_ x= (line disconnected from sourcs) when the receiving end is opened for Zoc , and when itis shorted for Zac. From the product ZocZac and the ralio Zoc/Zac the characteristic impedance and the Propagation constant per Unit length can be calculated. If measured values at SkHz are Zoc= 141.9/-84.1°Q and Zsc=62.0/37.7° for a 2-mile length of line, use the equation for Zs to find Zp, «(per mile), and B (pet mile). Ans. 93.8 /—23.2° Q, 0.12Np/mi, 0.28 rad/mi 1524, A 200-m length of 300-2 transmission line has 5% 10Np/m and B=0.02rad/m when operating at 200kHz. If V,=20/0°V and Z,=350/20°2, find the distance from the receiving tend to the first impedance minimum. What is the value Of this Zme? Ans. 107.2, 239.40 15.35. A 500-0 line is connected to a 10-KHz generator rated at 80/0°V open-circuit with an internal resistance of 6008. The line is 3 miles tong, with a@=0.05Np/mi and f=0.9rad/mi at 10kHz. For @ matched load at_x=0, find the sending-end power, the receiving-end power, and Vp. If the line is opened at the receiving end, what is the sending-end power? Ans. 1.32 W, 0.98 W, 31.3 /=154.7° V; 0.65 W 15.36. For the line of Problem 15.35, find the receiving-end current and the sending-end power if the line is shorted at x=0. Ans. 0.12/-157.6° A, 0.55 W 15.37. ‘The rigid coaxial line of Problems 15.2 and 15.9 would be classified as a low-loss line. (a) What are the reflection coefficient at the load and VSWR if the load is a 40-9 resistor? (b) Determine the maximum ‘and minimum load resistance for VSWR-=1.5. (c) Calculate the reflection coefficient 3m from the load, if Z_=(55+/0) 2 (consider attenuation). ‘Ans. (2) ~0.073, 1.16; (b) 69.45, 30.89 Q; (c) 0.0856 /—216° 15.38. A 909, lossless, high-frequency, coaxial line, with €,=2.1, operates at 1S0MHz. Of interest is the sensitivity of the VSWR to small changes in terminating resistance. (a) Tabulate T, and VSWR against Re=(90+2n)Q, for integral values of m from 0 to 5. (6) If the specifications for an application limit the maximum VSWR to 1.025, find the maximum and minimum values of terminating resistance. Ans. (a) See Table 15-4. (b) 92.24 Q, 87.812 15.39. For the transmission line of Problem 15.38, (a) find the phase velocity, wavelength, and the phase shift per meter. (b) If the terminating resistance is 100, find the input impedances for line lengths 4/2, 2/4, and A/S. Ans. (a) 2.07 10" m/s, 1.38 m, 4.55 rad/m; (6) 100/—0°Q, 81 /-0° 2, 90/-6'2 15.40. Use the Smith Chart to find (a) Tp, (6) VSWR, and (c) y, for the following (Zn, R,)-pairs, in ohms: (100 + 150, 50); (28 ~/35, 70), 90 (30° , $0); (120 /90° , $0); (0,70); (50-+ 18, 0). ‘Ans. (@) 0.75/26.5", 0.53 /121", 0.27 [-90, 1.0/45", ~1.0, 0.05 /90" (6) 7, 33, 175, © & LT (€) 0.16 ~ 0.225, 0.97 + 1.23, 0.87 + 0.52, j0.415, o, 1~ 70.1 1SA1. Find (2) yx. (6) VSWR, and (c) Yq (in mS) for the following (Fx Repairs: (0.5 60,509); (1{=80" , 902); (0.1 (0° ,70.Q); (-0.6 {30° , 90); (0.8 + 0.4, 70.Q). Ans. (@) O.44~ 0.495, 0+ 70.84, 0.83470, 2.0 j1.85, 0.06 — 0.238 (8) 30, &, 12, 40, 17 (©) 88~/9.9, 0+]9.3, 1.940, 40-37, 0.86—j3.4 15.82. 15.43, 15.46, TRANSMISSION LINES [cHar. 15 ‘Table 15-4 P 0.0826 | 1.089 0.0465 | 1.098 0.0526 | 1.111 0.0588 | 1.125 A ossless high-frequency ine 3m long, with Ry=50Q and ¢=1.9, is operated at 3S0MHz. ‘The VSWR on the line is 2.4 and the first voltage maximum is located 7em from the load. Use the Smith Chart to find the load impedance, the reflection coefficient at the receiving end, the location of the first voltage minimum, and the input impedance. Ans. (40-439) Q, 0.42 /81° , 22.6¢m, (22.3—f11) 2 28|as|/ex|e2)as| 8 8 A702 lossless ine, with «,=2.2, is 2.5m long and operates at 625MHz. ‘The VSWR on the line js 1.7 and the frst voltage minimum is located Sem from the load. Use the Smith Chart to find the load. admittance, the reflection coefficient at x=0, and the input admittance to the Tine. Ans. (10.4 + j5.4) mS, 0.27 68° , (16 ~ 7.9) mS ‘An airiclectric line with Ro~ 150 is terminated in a load of (150 —j150) at the operating frequency, 75MHz. (a) Use the Smith Chart to find the shortest length of line for which the input impedance is (150+ 150). (6) What are the VSWR on the line and the reflection coefficient at the load? What is the shortest length of line for which Z,=R+j0, and what is the value of RP Ans. (a) 1.3m; (b) 26, 0.47 [-64°; (c) 64.8em, 572 ‘Two lines are connected in parallel at the input to a 250-MHz source. Each line is 2m long and is terminated in a 70-2 resistance. Line #1 has Ry=S0Q, €,=1.9, line #2has Ry=%0Q, ¢, 23. Use the Smith Chart to find the input impedance to the parallel combination. "(Be careful in ‘combining the two input impedances/admittances.) Ans. (24.6+/3.5)Q A lossless 50-2 line, with a phase velocty 2.5 x 108m/s, is 05cm long and is terminated in a load Yq=(20-/16)mS at SOOMH2. A shorveireuited line, 17.85cem long and also having Ra: 50, & connected across Y, as shown in Fig. 15.44. Use the Smith Chat (0 find the VSWR on the rain line and the input impedance. What is the equivalent capacitance (or inductance) of the short-circuited line? Ans. 1.0, 502%; 5.1 pF CHAP. 15] ‘TRANSMISSION LINES m 1s.a7, 15.48. 15.49. 15.50. 1551. 15.82, 15.54. 1857. 1558, In the line of Problem 15.46 the short-circuit on line two is inadvertently changed to an open circuit, Use the Smith chart to find the VSWR on the main line and the input impedance. Ans, 62, (26.5 +j725) 2 A parallel-wire line of the type in Problem 15.1 is operated at 20 MHz to supply a resistive load of 500 through a quarter-wave matching transformer connected at the load. Neglect losses on the main li fand the transformer section. (a) For the transformer calculate the length of line and characteristic impedance required for matching. (b) If the same sized wire is used for the main line and the transformer, find the separation d (in inches) required for matching. Ans. (a) € =3.75 m, Ror = 347.79; (6) dy =7.77 in. A lossless 70-2 line is terminated in Zp = 60.3 /30.7°2 at 280MHz. Use the Smith Chart to find the value of the inductance or capacitance to connect in parallel with the load for minimum VSWR on the line, What length (in centimeter) of shorted line would give the desired value, if <, = 2.1? Ans. C=48pF (VSWRei= 1.0); 24.86m ‘A 2000 airdielectric line is terminated in Y»=(3.3~ (1.0) mS at 200MHz. (a) Find the VSWR. ‘and the position nearest the load where the real part of the normalized admittance is unity, using the Smith Chart. (6) What value of susceptance (in milisiemes) should be connected at this point to make VSWR=1 on the line? Ans. (a) 1.65, 29.3 cm; (b) ~2.55 (inductive) ‘Two 72-Q resistive loads are connected in parallel as the termination for a 120- air-dielectric lossless line at 150MHz. Find the location nearest the load and the length (both in centimeters) of a single shorted parallel-connected stub to match the line for a VSWR = ‘Ans, 16cm, 78.6cm {@) In Problem 15.51, if the maximum length of the adjustable shorted stub is SOcm. can the load be matched to the line?’ (6) If the answer to (a) is Yes, find the position and length of stub for the matched condition. (c) If the stub were left at its orginal position and set to the 50-em maximum, what ‘would be the VSWR on the line? Ans. (a) Yes; (6) 83.8em, 21.4em; (c) 3.3, ‘A 90-0. lossless line with €,=1.8 operates at 280MHz and is matched to the termination with a single shorted stub that produces VSWR-= 1.0. The stub is located 15.8em from the load and is of length 10cm. Find the ohmic value of the terminating impedance. (Hint: Remove y,,, find the VSWR, and move back toward the load.) Ans. 201.2/26.6°Q A 50 air-dielectric lossless line has Z_= (25 j30) 2 at 120MHz. An adjustable shorted stub is located 45cm from the load (fixed in position). Find the length of stub for the best match on the line. What is the minimum VSWR on the line? Ans. 96cm, 1.08 (@) An air-dielectric lossless 70-2 line is matched at 200 MHz to a 140-0 load by means of a shorted Parallel stub. Find the position nearest the load and the length of the stub (both in centimeter) for the ‘matched condition. _(b) The line is now used at 220 MHz without changing the position or length of the stub. Find the VSWR on the main line at the new frequency. ‘Ans, (a) 22,65 cm, 22.80 em; (6) 1.22 . The termination on a 90-G lossless air-diclectric line is Zq = (270+ 0) at 600MHz. A double stub 0.254 tuner is connected with the first stub at the load for matching. Find the lengths for the shorted stubs (both solutions). Which solution is preferred? ‘Ans. 9¢m and 4.95 em (preferred); otherwise, 16m and 20.05 em. A 34/8 tuner is connected at the load to match Zp, = (50~/50)Q at 400MHz to a 50-Q lossless ‘ir-dielectrc line. Find the lengths of the shorted stubs for both solutions and indicate the preferred solution. Ans. 4.5em and 4.3m (preferred); otherwise, 12.1 em and 26.1.m. ‘A S0-Q sirdielectric line, with a load Yq =(0.024—/0.02)S at 470MHz, has a 3/4 tuner with the first stub located 7-cm from the load. Find both solutions for the lengths of the shorted stubs to match the load to the line. Indicate the preferred solution. ‘Ans. 14.5 em and 7.6m (preferred); otherwise, 23.1 em and 24.3cm. m 59. 15.60, 15.63. 15.64. 15.65, 1567. ‘TRANSMISSION LINES ICHAP. 15 ‘A two-stub 32/8 tuner is constructed of 70-9 line with €=2.0 for use at 272MHz on the same type Of line and a certain load. For the matched condition the shorted stub at the load is 4.76cm long and the other shorted stub is 4.60cm long. Find the ohmic impedance of the load at the operating. frequency. Ans. (58.1 ~ j58.1)Q A 225-2 resistive load is matched 10 a 90-@ air-dielectric line at 300MHz. The matching is via two shorted stubs separated by 30cm, with the first stub connected at the load, Find the lengths of the stubs (both solutions) and indicate the preferred solution. ‘Ans. 13.66m and 8.5cm (preferred); otherwise, 28.1 cm and 37.5.em. ‘A.50.Q slotted line is used to determine the load impedance at 750 MHz on a lossless 50-Q line. When the line is terminated in a short circuit, the high VSWR has adjacent minima at 30¢m and 10cm (the scale has the low numbers on the load side). With Z, connected the VSWR is 3.2, a minimum is located at 13.2em and the adjacent maximum is at 23.2cm. Find the ohmic value of Z, at the operating frequency. Ans, 31.24/-$0.2°2 Find the load impedance and the operating frequency for a 90-9 air-dielectric system that has the following slotted-line measurements: With load: VSWR=1.6 and a voltage minimum at 10cm (high numbers on load side). With short circuit: VSWR > 100, minimum at 40cm, maximum at 10cm. ‘Ans, 1449; 250MHe ‘A 50 slotted line is used to measure the load impedance at 625MHz on a 50-2 lossless coaxial Tine. Adjacent voltage minima are found at 10mm and 250 mm (high numbers at the load side) when the termination isa short circuit. With the load connected, VSWR >100 and a minimum occurs at 172.7 mm. Find the ohmic value ofthe load impedance. Ans. 80 (capacitative) ‘A tuner is connected at the load to match the load to a S0-2 lossless air-dielectrc line at 517 MHz. To check the quality of the matching a slotted line is inserted in the system. With the tuner and load connected, VSWR=1.15, with a Vu at 253.4mm. When the tuner and load are both removed and. replaced by a short circuit, adjacent minima are found at 40 mm and 330 mm (low numbers on the scale are at load side). Find the residual normalized admittance on the line that results from the “best match.” Ans. 0.98 — 0.14 {A 60 mong, lostess, 50-2 coaxial cable, with a phase velocity of 2x 10* m/s, is terminated with a short circuit. The line is connected at ¢=0 to 8.30-V de source having internal resistance 252, Plot the Scndingend voltage from 1=0_ up to the time when the voltage drops below 0.1V, Ans. See Fig 154. nv, 667 22 04 038, 008 Fig. 15.45, A 90-2 lossless line, with «,=2.78, is connected at 1=0 to a 70-V de source with an internal resistance of 1209.” If the line is 135m long, find the time when the open-circuit voltage at the receiving end is 97% of the steady-state value. When is the voltage 99.95% of the steady-state value? Ans. 2.25 ys, 5.25 ps ‘A pulse generator with internal resistance 150 produces a 20-ns pulse with an open-circuit amplitude CHAP. 15] ‘TRANSMISSION LINES m3 Of +8V. ‘The generator is connected to a 50-Q, lossless, 200-ps delay line that is terminated in a 100-2 resistance. If the period of the recurring pulses is 4ms, sketch the voltage at the input to the delay line from pulse onset at ¢=0 to f=14ms Ans. See Fig. 15-46. we 10 oar oe ms Fig. 15.46 15.68. Sketch vs and vp versus time, from 1=0* to ¢=300ps, when 4 70-0, lossless, 50-ps delay line is terminated with a 30-Q resistor and driven by a pulbe generator. The generator has an internal resistance of 70 and produces a 2-us pulse with a peak open-circuit voltage of +10V at a repetition rate of 1000 pulses per second. Ans. See Fig. 15-47. wv eM tL . a 20 Fig. 15.47 15.69. In Fig. 15-48 a line is used to produce a short rectangular pulse of width 12ns and peak value 800 V. With S-2 open, 5-1 is closed to charge the line to Vai after charging, S-1 is opened. ‘Then, at £0, S-2 is closed to discharge the line through Ry and form the pulse. Find the length of line and Vy. Ans. 1.2m, 1600V 2k Fig. 15-48 15.70, Sketch us and vp versus time, from 1=0° to 1=30s, when a 220-m-long, 90-Q, lossless line, with .65, "is terminated in a S0-Q resistance and driven by a pulse generator. ‘The generator has an internal resistance of 909 and produces a 5-ys pulse with open-circuit peak value +140 V, at a repetition rate of 100 pulses per second. Ans. See Fig. 15-49. nv ve * Sys OTE ¥ os Chapter 16 Waveguides (by Milton L. Kult) 16.1 INTRODUCTION ‘The electromagnetic waves of Chapter 14 can be guided in a given direction of propagation using several different methods. For instance, the two-conductor transmission line, supporting what are essentially plane waves at megahertz frequencies, was considered in Chapter 15. The present chapter is restricted to single-conductor (hollow-pipe) waveguides, of rectangular or circular cross section, which operate in the gigahertz (microwave) range. ‘These devices too support “plane waves”—in the sense that the wavefronts are planes perpendicular to the direction of propagation. However, the boundary conditions at the inner surface of the pipe force the fields to vary over a wavefront. 16.2 TRANSVERSE AND AXIAL FIELDS “The waveguide is positioned with the longitudinal direction along the z axis. In general the guide walls have 0, = (perfect conductor) and the diclectric-filed hollow has @=0 (perfect dielectric), = ot,, and €=€€,. IL is further supposed that p-0 (no free charge) in the dielectric. The dimensions for the cross section are inside dimensions. In Fig. 16-1(a) the @Xb rectangular waveguide is shown in a cartesian coordinate system, Fig. 16-1(b) shows the Gircular or cylindrical waveguide of radius a in a cylindrical coordinate system. 4 ‘As in Chapter 14 the time dependence e* will be assumed for the electromagnetic field in the dielectric core; this time-factor will be suppressed everywhere in the analysis (as in phasor rotation), Thus we have the following expressions for the field vector F (which stands for either E. or H), assuming wave propagation in the +z direction, rectangular coordinates F=F(x, y)e"A* where Fr, 9) = Ra, ya + B(x yay + Ee Ya = Fr(x,y)+ EG ya, cylindrical coordinates F=F(r, pe" where FOr, 9) = Fr, Pa, + Fel, Oe + F(x, Oa: Fr(r, 0) + Els om, Fig. 161 24 CHAP. 16] WAVEGUIDES m5 the wave propagates without attenuation; hence the wave number k=2z/2 (in rad/m) is constrained to be real and positive. Note: In the other chapters of this book, unbounded dielectric media are considered, for which the wave number, notated B, depends on frequency and dielectric properties only. However, as will soon appear, the wave number in a bounded dielectric depends additionally on the geometry of the boundary. This important distinction is emphasized by the employment of a new symbol, k, in the present chapter. ‘The reason for decomposing the field vector into a transverse vector component Fy and an axial vector component Fa, is two-fold. On the one hand, the boundary conditions apply to Ey and Hy alone (see Problems 16.1 and 16.2). On the other hand, as will now be shown, the complete E and H fields in the waveguide are known once either cartesian component D, or H, is known. ‘Transverse Components from Axial Components, Assume a rectangular coordinate system. Maxwell's equation (2) of Section 14.2 yields the three scalar equations ; veg 4 3E “jon, = ke, +E ia) aE, —jopH, = —jkE, me (ib) 3E, dE, ~jont, = 52 -S (te) ‘Maxwell's equation (1) of Section 14.2, with o=0, gives thece additional scalar equations: aH, jweE, = jkH, + (2a) jek, = jkH, +5 (2a) (2) ey (3a) (3%) in which k2= w?ye - 2. The parameter k, (also in rad/m) functions as a critical wave number; see Problem 16.3. Finally, stide (3) and (32) back into (2a) and (25), to find y= ay as (&) = ik at , je 3, Hee Bae eS (3a) By exciting the waveguide in suitable fashion it is possible to force either E, or H, (but not both) to vanish identically. The nonvanishing axial component will then determine all other components via Equations (3). See Problems 16.4 and 16.5 for the analogous results in cylindrical coordi 26 WAVEGUIDES IcHap. 16 16.3 TE AND TM MODES; WAVE IMPEDANCES ‘The two types of waves found in Section 16.2 are referred to as transverse electric (TE) or transverse magnetic (TM) waves, according as E,=0 or H,=0. When carrying such waves, the guide is said to operate in a TE or TM miode. For any transverse electromagnetic wave, the wave impedance (in ohms) is defined as =/Exl 1H, (compare Chapter 14). For a waveguide in a TE mode, (1a) and (16) imply ” @ HEr/?=1E.P +16)? ee) e+) or we mete % Because (4) only involves lengths of two-dimensional vectors, n must be independent of the coordinate system. Problem 16.6 confirms the value of yz by recalculating it in cylindrical coordinates. In Problem 16.7 it is shown (using rectangular coordinates) that ow = DE 6 164 DETERMINATION OF THE AXIAL FIELDS {All that remains for a complete description of the TE and TM modes is the determination of the respective axial fields: F,=H, for TE; F,=E, for TM. The good word is that F,e"**, being 1 cartesian component of F (in either rectangular or cylindrical coordinates), must satisfy the scalar ‘wave equation found in Section 14.2, Vie) = —a7ne(he*) ” together with appropriate boundary conditions which are inferred from the boundary conditions on the components of F;. Warning: Transverse components such as Hye"#* are not cartesian ‘components and do not obey a scalar wave equation.) Explicit Solutions for TE Modes of a Rectangular Guide. “The wave equation (7) becomes oH, PH, Bxt * By? oye — Kix. Solving by separation of variables (Section + KapH, =0 where, as previously defined, Ki; 8.7), H(z, y) = (A, cos kx + B, sin k,x)(A, cos k,y + B, sin k,y) ®) where K2+K2=K2re. The separation constants k, and k, are determined by the boundary conditions (review Problem 8.19). Consider first the x-conditions £,(0, y)=E,(a,y)=0; in view of (32) and E,=0. these translate into oH, - 0 ar | 3H, Applying these conditions to (8) gives B,=0. and ox sinka=0 or CHAP. 16] WAVEGUIDES m By symmetry, the boundary conditions in y force B,=0 and Each pair of nonnegative integers (m, n}—with the exception of (0,0) which gives a trivial solution—identifies a distinet TE mode, indicated as TE,... This mode has the axial field Hl, Y)= Hu 608 cos 7) from which the transverse field is obtained through (3). ‘The critical wave number for TE my is in terms of which the wave number and wave impedance for TE eq ate Krenn = VOTHE = Kite ap ou Tome Toe — Rien ” See Problem 16.9 for the TM, modes of a rectangular waveguide; it is shown there that kerminn = Kertmn- Consequently, the subscripts Te and Tw can be dropped from all modal parameters of rectangular guides save the wave impedance. This is not the case with cylindrical guides; see Problem 16.12. 16.8 MODE CUTOFF FREQUENCIES In practice one deals with frequencies, not wave numbers; it is then desirable to replace the concept of critical wave number (k.) by one of cutoff frequency (f.). This is accomplished in the definition (see Problem 16.3) (3) In terms of the cutoff frequency f. and the operating frequency f= w/2x>f, (10), (11), and (12) become 4) (rectangular waveguide) (10bis) © eR (ibis) to tm ET (12bis) where y= o/f isthe wavelength of an imaginary uniform plane wave at the operating frequency and where j= Vu/e is the plane-wave impedance of the lossless dielectric. The second form of (1bis) exhibits the relation between the operating wavelength Ay and the actual guide wavelength Inn. FOr TMy, Waves, (126i) is teplaced by {see (6)] Tre = not = (Ea) (us) 28 WAVEGUIDES Icuap. 16 ‘The phase velocity of a TE; OF TM, wave is given by Yn = Donn f = «sy If (10bis) is replaced by a similar expression involving a Bessel function (see Problems 16.10 and 16.11), all formulas remain valid for cylindrical guides. ‘The meaning of cutoff is made particularly clear in (15). As the operating frequency drops down to the cutoff frequency, the velocity becomes infinite—which is characteristic, not of wave propagation, but of diffusion (instantaneous spread of exponentially small disturbances). 16.6 DOMINANT MODE ‘The dominant mode of any waveguide is that of lowest cutoff frequency. Now, for a rectangular guide, the coordinate system may always be oriented to make a2b. Since (Problem 169) Ho my? (n\? fom = (3) + (6) for cither TE o TM, but neither m nor can vanish in TM, the dominant mode of a rectangular guide is invariably TE, with dye WH Ouray hoo From (9), E,o=0, and the equations of Section 16.2: dyo=: Hso= Hig cos = Ho i( 72) Mosin = ~ind 6) Hyp=0 For Hio real, the three nonzero field components have the time-domain expressions Ho= Hyoc0s (=) c0s (ot — ky) Hiso= -(32) sin (2) sin (ot — ke) a Eno= n(24) I) sin (2) sin (ot- koe) Plots of the dominant-mode fields (17) at_ t=O are given in Figs. 16-2 and 16-3. Both |E,| and |H,| vary as sin (ztx/a). This is indicated in Fig. 16-2 by drawing the lines of E close together near x=a/2 and far apart near x=0 and x=a. The lines of H are shown evenly spaced because there is no variation with y. This same line-density convention is used to indicate the local value of E]=1E,| in Fig. 16-3(a) and of = VA in Fig. 16-3(b). Observe that the lines of Hare closed curves (div the H field may be considered as circulating about the perpendicular displacement current density Jp (Section 12.1). CHAP. 16] WAVEGUIDES 279 4 Fig. 162. ‘Transverse cross section 2=—Del4 (hut = 2/2) 2 Re (Ox a2 (0) 2 y= const Fig. 163. Longitudinal cross sections Figure 164 illustrates how the TE. mode can be initiated in a rectangular waveguide by inserting a probe halfway across the top wall (y=b, x=a/2), at a distance z=Ay/4 from the end of the guide. Higher-order modes are present in the vicinity of the probe, but they will not Propagate if the frequency-size condition is selected correctly. See Problem 16.13 for the dominant mode of a cylindrical waveguide. I. aa * nw Fig. 164 16.7 POWER TRANSMITTED IN A LOSSLESS WAVEGUIDE ‘The time-average power transmitted in the +2 direction is calculated by integration of the z ‘component of the complex Poynting vector over a transverse cross section of the guide (cf. Section 14.12): a, dS (18) ne [f ee 280 WAVEGUIDES IcHap. 16 Substituting the field components from (16) and writing A,=ab, we obtain for the dominant mode of a lossless rectangular waveguide: (9) AAs expected for a lossless system, independent of 2; moreover, the power is proportional to the square of the field amplitude and to the cross-sectional area of the guide. Since the excitation of a guide is commonly specified through the electric field amplitude, xd = no) Ha it is useful to rewrite (79) as Relations similar to (19) and (19is) exist for the higher-order modes. For the lossless cylindrical guide, sce Problem 16.15. 16.8 POWER DISSIPATION IN A LOSSY WAVEGUIDE When the conductivity of the guide dielectric is nonzero (but small) and/or the conductivity of the guide walls is noninfinite, the wave in any propagating mode will be attenuated, and transmitted power will decrease exponentially with z. An approximate treatment of these dielectric and wall losses is possible on the assumptions that the two types may be analyzed separately and that the fields which interact with the walls ate those which would be present if the dielectric were lossless. To keep the mathematics as simple as possible, only the TEy mode of a rectangular waveguide will be treated. Dielectric Loss. Maxwell's equations (1)-(4) of Section 14.2 are unchanged if 04, the dielectric condue- tivity, is replaced by zero and the dielectric permittivity, is replaced by its complex permittivity ene, ‘Therefore, the field equations for the lossy dielectric may be obtained from those for the lossless dielectric by formal substitution of é for €g._In particular, the z dependence of the field vectors in the lossy TE, mode is exp (—y102), where, by (11), kwh @) = Veo" hae — Kero = 70" Hata — Keo) — FOO " jomses)'” _ (emaae Ba ee) (SE) + Bo (20) In), Bro Va" Hea Fon = hk) = VI= Gol @ and the binomial approximation presumes that 9, and w ate small enough to make @p,0,-< Bio. To this order of approximation, then, the wave numbe CHAP. 16] WAVEGUIDES 281 Re yo, which governs the power loss in the dielectric, is given by mee __ (Vial eadou__ 00 Fhe Gag me (Np) (@2y Wall Loss. ‘The attenuation factor a, governing the wall loss may be determined indirectly, as follows. Because power varies as the square of the field strength, the time-average transmitted power in the TE mode must obey Paol2) = Pryge 7 where the entrance power Pug is as given in (19). The power dissipated in the walls per unit “length is thus , Paal2) = ~Pi(2) = 202) PowlZ) _ Posf0) whence “2P(2)~ 2Pao oe All that remains is to calculate P.u(0), the power flowing into the first 1m of wall inner surface. Now, it is not hard to show that, at a wall surface, tangential H—which by hypothesis can be obtained from (16)—sets up a Poynting vector, of time-average magnitude Sone = ARs [Heangl? (24) and directed into the wall. Here, R,=Re 1. = Vafu./0, (Section 14.6) is the surface resis- tance () of the wall material at the given frequency f. Integrating the appropriate expression (24) ‘over the first 1m of each wall surface and adding the results yields finally PoalO)= Re ht [b+ 204 Hf] (Wm) @) From (23), (19), and (25), «Se Stee oom ta in which R,-1 is the surface resistance at the cutoff frequency of TE; and o=Vitaléz is the plane-wave impedance of the (lossless) dielectric. Combined Losses. ‘The total attenuation factor is iq = a +44. To convert from Np/m to the more usual dB/m, see Problem 14.7. Solved Problems 16.1. Give the boundary conditions on E and Hat each perfectly conducting wall of the waveguide of Fig. 16-1(2) ‘At a perfect conductor tangential E and normal Hi must vanish. Therefore: top wall E,(x,b)=E,(x,)=0 and Hy(x,b)=0 Heft wall E,(0,y)=E,(0,y)=0 and H.(0,) right wall E2(0,y)=E,(@,y)=0 and H.( bottom wall E,(x,0)=E,(x,0)=0 and H,(x,0) 16.2, 165. WAVEGUIDES Repeat Problem 16.1 for the guide of Fig. 16-1(b) ‘At the single cylindrical wall, E,(a, ¢)=E,(a, 9)=0 and H(a, $)=0 What is “critical” about the number k,? For propagation through a lossless dielectric, the wave number k must be real. But k= Value — Kis VBR IcHap. 16 where ky is the wave number of a uniform plane wave in the unbounded dielectric at the given ‘o. Thus k.is critical wave number in the sense that a guided wave's same-frequency “twin” must have a wave number exceeding ke. Stated otherwise, the frequency fof the guided wave must exceed the quantity (ug/2m)k,, where uy 1/Vue is the wave velocity in the unbounded dielectric. Express Maxwell's equations (J) and (2) of Section 14.2 in scalar form in a cylindrical coordinate system. For the curl in cylindrical coordinates, see the Appendix. Equation (1) yields (0 = 0): 1aH, jooet, = + tty jeacE, = —jat, — Ee @ w i) w @) (wi) 1a 1a, jock, 13 0) Equation (2) yields: 13E., 756 tHE aE, ~iontty = ~jkE, - 1a mont, = 2 (£,)— Using the equations of Problem 16.4, find all cylindrical field components in terms of E, and H, ‘From (i) and (v), From (if) and (iv), From (1) and (i), From (2) and (ii), Hk1AE, | jonah, Fe= Ep ag* or « @ @ ® CHAP. 16} WAVEGUIDES 283 16.6. Calculate re from the field components in cylindrical coordinates. With E,=0, (iv) and (uv) of Problem 16.4 yield l= VOR FTE = (2) eg + (2) et Elon Trl kee whence 16.7. Calculate nr, from the field components in rectangular coordinates. With H,=0, (2a) and (25) give lee +16 = (22) ante ie) or teri = “in| = Eel oa kn 46.8. Show that E and Hare mutually perpendicular in any TE or TM wave (as with ordinary plane waves). For either type of wave E,= nH, and E,=—nH,; therefore, since nis real, Ey Hy =Re(E,H! + E,H3) = Re(nH,H: ~ nHHs) 16.9. (C08 kx +, sinkx\(C, cosk,y + D, sink) where K+ R= Kine ope — Ki But now the boundary conditions, £,0,y)=Eda.y)=0 and E,(x,0)~E,(x,b)=0 require that Gao G0 where m,n=1,2,3,.... Note that neither m nor n is zero in a TM mode. ‘The required formulas are @ @ @ @ 16.10. Determine the TM modes of a lossless cylindrical waveguide. a WAVEGUIDES (CHAP. 16 ‘The Laplacian in cylindrical coordinates is given in the Appendix; the wave equation (7) for E,fr, ¢) becomes 1aE, 18, rar Pag subject to the boundary conditions (i) E,(r, 6 +2m)=E,(r, $); Gi) Ex(a, @)=0. Following Section 8.8, one solves by separation of variables to find Eelts 0)= Eaoalkrnee) 20500 where 20.1.2... and Where Xap koneya is Of Jf) =0. (The Fist few such roots are listed in Table 16-1.) + KiwE, = 0 Teble 16-1. Roots x, of J,(2) =0 n=0 n=2 0 n=3 pel | 2405 3832 5.136 6.380 p22 | 550 7016 R417 9.761 p=3 | 8645 10.173 11.620 12015 ‘The expression (1), together with H, =0, Problem 16.5. The cutoff frequency of TM,, is given by Determine the TE modes of a lossless cylindrical waveguide ‘The solution by separation is therefore: Halt, 9) = Hooda KerergF COSMO where m=0,1,2,... and where xi, =keey of Jz(x)=0. See Table 16-2. Table 162. Roots x, of J.2)=0 n=0 n=3 p=1| 3832 1841 3054 4201 p=2| 7016 S331 6706 8015 p=3| 10.173 8536 9.969 11.346 In a TE mode the axial field H,(r, ) obeys the wave equation and the conditions (i) and Problem 16.10. As a consequence of (2) of Problem 16.5, condition (ji) must be replaced by (kira one — Kine) Gi) E,@, 4) bounded; @ the pth positive root (p=1,2,...) determines all transverse field components in TM vi @ @ ) of w isthe pth positive root (p=1.2,...) CHAP. 16] WAVEGUIDES 285 16.12. 16.13. 16.14. a 16.15. ‘The analogues of (2) and (3) of Problem 16.10 are: @ Discuss the relative magnitudes of ferey and fersinp For each fixed n, the 2er0s x4, of J(x) and the stationary points x,,—where J,(x) is a maximum or ‘2 minimum—alternate along the © axis; this sine-wave-like behavior is clear in Fig. 8-3(a). For n> 0, the function starts at 0, and the first stationary point precedes the first positive zero; thus, x1, Kermee and fereap> fermen (a) What is the dominant mode of a lossless cylindrical waveguide? (b) List the first five ‘modes in order of increasing cutoff frequency. (@)_ By Problem 16.12, the dominant mode is either TMo, or the TE,; with the lowest cutoff, Tables 16-1 and 16-2 indicate (and analysis establishes) that the winner is TE, (©) TE; TM), TEx TEoy, and TM), (8 tie). [The first column of Table 16-2 is identical to the second column of Table 16-1 because Jg(x) = —J(x).] (Obtain the transverse fields for the TE,, (dominant) mode of a cylindrical waveguide. For m=p=1, Equation (1) of Problem 16.11, £,=0, and (1)-(4) of Problem 16.5 yield o @ @ @ in which kere = Calculate the time-average power transmission in the TE,, mode of a lossless cylindrical guide. Follow Section 167, with the transverse fils as given by Problem 16.14 fg — Eg th) eres tal (MOY cnt 9 + Urorreng} o ‘which the integration variable v= Kermur has been introduced. In the integration of (1) over the 16.16. 16.17. WAVEGUIDES (CHAP. 16 cross section 0<@ <2" and O- 0) eo) ‘This is @ one-parameter family of curves, where the parameter po gives the radius at which a curve cuts the horizontal axis sing =0. Note thatthe right side of (3) does not change when @ is replaced by =6 or by $+ x; hence the field pattern is symmetric about both the horizontal and vertical axes, and only the quadrant O=@=7/2 need be considered. As one moves along a field line through increasingly positive -values, the right side of (3) increases through positive values. Consequently {see Fig. 8 3(a)], p/P increases through values greater than 1. This, together with the constraint that the field line hit the boundary p= 1 orthogonally, shows that the field line must bend away from the origin, as shown in Fig. 16-5. The line p.=1 “degenerates into 2 single point. CHAP. 16.23. 16.24, 16.28. 16) WAVEGUIDES 289 ‘The lines of H are plotted as the orthogonal trajectories of the E lines; see Problem 16.8. By Problem 16.14 both H, and H, vanish at the points p indeterminate there. A lossless air-dielectric waveguide for an S-band radar system has the dimensions a= 7.2l4em and b=3404em. The dominant mode propagates in the +z direction at 3GHz. Find the average power transmitted ifthe excitation level of the E field is 10 kV/m. ‘The cutoff frequency for TE is finn 3X10 is ono 208 Oe and (19bis) yields 07707.214).404)10 467) In a lossless air-dielectric cylindrical waveguide with a 1cm radius the transmitted power in the dominant mode at 15 GHz is 2W. Find the level of excitation for the magnetic field. ‘The cutoff frequency for TE, is (see Table 16-2): hy 3x10" 2na*" “Im (0x10) 0 that (4) of Problem 16.15 becomes (see also Problem 16.16): few 1.841) = 8.79 GHz ZZ raat e10- 0518-797 VE= ETT 10.29) Solving, \Hul=0.11 A/em, A section of X-band waveguiude with dimensions a=2.286cm and b=1.016em has perfectly conducting walls and is filled with a lossy dielectric (0, = 367.5 uS/m, ¢, 2.1, p=). Find the attenuation factor, in 4B/m, for the dominant mode of propagation at a frequency of 9 GHz. ‘The cutoff frequency of TE is fi My _ 310/21 _ 5 = 453 Ot and (22) gives (second form): = STV AYG67-5 x 10-*) i- 33 ‘The reader should verify that the underlying approximation, cytg4<€ Bly, holds forthe data (4B/e) 8.69 = 0.48 ‘An X-band airdielectric rectangular waveguide has brass walls (ity —Ho, Om = 16MS/m) with @=2.286cm and b=1.016cm. Find the dB/m of attenuation due to wall loss when the dominant mode is propagating at 9.6 GHz. ‘At the cutoff frequency of the dominant mode, 3x10" 2a" F572 x 107 56GHz Sew the surface resistance of the brass is Vv 16x16 Reon’ 40.24 ma 20 WAVEGUIDES IcHaP. 16 " 04024 / (9.6) 0.02286 + 2(0.01016)6.56/9.6)? - wenn) = 2504) Bo ce 16.27 An airdielectric cylindrical waveguide (a=Smm) operates in the TMg mode at frequency 3ferwor - Find the dB/m of attenuation due to wall loss in a short section ‘of copper (0, = 38MS/m). First_ derive an expression for Ru(0), following Section 16.8. By (1) of Problem 16.10, E(t, @)~ Eoddtur/a). Then (3) of Problem 165 gives the tangential magnetic Held at the wall Hilo) = HOO: tern Enda) _iEoea) (_f. Hao 9) = ee a ae and, since Hye is constant, (24) gives Eos! Fixer ") 40) = sn [Eat Ad (LY Iernay o Next find Perse by the method of Problem 16.15. By Problem 16.15, esl) Li OP) jE {Hermon " Hi while Ho =Eyo)=0. Thus the time-average Poynting vector is [Eo fersen'V1= Ceol ea) 5 = 1B oiHin = The * Integrating over a eross section, [ [2 Se drdg = 2a Teyudu = ATi) Combining these results, a“ ie meat “imi ° For the data, = 2.9GHz 1=(1.3)22.99) = 29.89 GHz [afi _ [29.89 x 10)(4n X10) _ R= ee er o0sie 0.0451 GME })VI-G 0.0374 Npim = 0.325 ¢B/m Supplementary Problems 16.28. Determine the condition(s) under which a magnetic field with Hay. 2,0) = K cos873x cos 92.Ay cos 2nft— 108.12) can exis in free space, Anz. f = 80GHz CHAP. 16] WAVEGUIDES 291 16.29, 16.30, 1631. 16.33. 1634. 16.41. 16.42, Obtain the critical wave number for a 4-GHz wave propagating in a medium with ,=1 and ¢,= 2.2, ifthe phase shift constant (wave number) is 54° perem. Ans. 81.1 rad/m If H.(x, y, 2,0) in Problem 16.28 represents the axial field of a TE,, wave in a rectangular waveguide, find (a) the guide size, (6) the critical wave number, (c) the guide wavelength Ans. (a) 72cm by 3.em; (6) 127.1 rad/m; (€) 5.76em ‘The S-band waveguide of Problem 16.18 is used in the X-band at 9GHz. Identify the modes that could [propagate in the guide. Ans. ‘TE, , TEg:, TE » TE,» TEs», TE21» TEw, TEs,» TExsi TMi» TMs» Mn, In Problem 16.19, what other modes could propagate at the given frequency? Ans, TE, , TEs, , TE: » TEx} TMos ‘A Cand waveguide for use between 3.95 and 5.85GHz measures 4.755cm by 2.215em. For air dielectric, calculate the dominant mode cutoff frequency and the guide wavelength when the operating frequency is 4.2GHz. Ans. 3.185 GHz, 10.82¢m ‘The WC-50 cylindrical waveguide with air dielectric is used in the frequency range 15.9-21.8 GHz for dominant-mode propagation. Calculate the cutoff frequency for an inside diameter of 1.270em. Also obtain the cutoff frequency for the TMs, mode. Ans. 13.84 GHz, 18.08 GHz ‘An air-diclectric L-band rectangular waveguide has 4/6 =2 and a dominant-mode cutoff frequency of 0.908 GHz. If the measured guide wavelength is 40cm, find the operating frequency, the guide dimensions, and the wave number. Ans. 1.18Ghz, 16.52cm by 8.26¢m, 15.7 rad/m. For the waveguide in Problem 16.35 find the lowest frequency at which a TE,, mode would propagate, Ans. f >2.569GHz ‘A Veband waveguide for use between 26.5 and 40GHz has inside dimensions 0.7i1cm by 0.356em. (a) Calculate the dominant-mode critical wave number for air dielectric. (6) If the ‘measured guide wavelength is 1.41 cm, what is the operating frequency? ‘Ans. (a) 441.86rad/m; (6) 29.98 GHz ‘The WC-19 airdiclectric cylindrical waveguide is used for dominant-mode operation in the 42.4~ S8.10GHz range. Find the inside diameter for the specified cutoff frequency of 36.716GHz, Ans. 0.478em A Ku-band airdielectric guide with a/b =2 is used in the 12.4~18.8GHz range for dominant-mode ‘operation with a cutoff frequency of 9.49GHz. What are the inside dimensions? ‘Ans. 1.S8em by 0.79em Find the radius and guide wavelength in an air-tielectric cylindrical waveguide forthe dominant mode at f= 30GH2= 1Sfernn . Will he TMy, mode propagate under these conditions? ‘Ans, O.Adem, 1 34cm, No Solve Problem 16.40 for the guide with a lossless dielectric of €,=2.2. ‘Ans. 0.296em, 0.93 ern; NO ‘A K-band rectangular waveguide with dimensions 1.067cm and 0.432cm operates in the dominant mode at 18 GHz. Find the cutoff frequency, guide wavelength, phase velocity, and wave impedance, if the dielectric is air. Ans. 14.06 GHz, 2.67 em, 4.81 x 10" m/s, 604.22 Solve Probiem 16.42 ifthe guide is filled with a lossless dielectric of, =2.0. Ans. 9.93GHz, 1.44em, 2.54 x 10" m/s, 319.62 292 6s, 1648, 16.47, 16.48, 16.50. 5. In a cert WAVEGUIDES IcHar. 16 Calculate the radius and guide wavelength for a TMy mode at f=30GH2=15faq, in an sit-dieleetric cylindrical waveguide. (Compare Problem 16.40.] Ans. 0.915em, 1.3420m For an (m,n) mode operated below its cutoff frequency, the cutoff atenuation factor is defined 38 Gm ~Hhen- Calculate eran, in Blom when a lossless sit-diclectric guide, 2.286cm by 1.016 cm is operated at 9.4GHz. Ans, 23.9 cross section of a rectangular waveguide the instantaneous components of E are eonin(S)aa(Z) aotea()m(Z) ne ‘Sketch this E field and idemtify the mode of operation. Ans. See Fig. 16-6; TE,, EIS Le. Fig. 16-6 ‘The air-dielectric waveguide of Problem 16.23 transports 200 W of average power at 2.6GHz. Find the excitation level of the field. Ans. 143.V/em If a lossless dielectric having ¢,=1.8 is inserted in the waveguide of Problem 16.47, calculate the excitation level for the transport of 200W. Ans. 106.8 V/em ‘The air-diclectric waveguide of Problem 16.24 is filed with a lossless dielectric having ¢,=2.1. Find the power transported in the dominant mode, if the excitation level and frequency are unchanged. Ans. 0.09 A/em Show that result (2) of Problem 16.27 can be rewritten as a= GaBcmagy, WME & is the (frequency dependent) skin depth. G0 8.Rnn Chapter 17 Antennas (by Kai-Fong Lee) 17.1 INTRODUCTION Maxwell's equations 2s examined in Chapter 14 predict propagating plane waves in an unbounded source-free region. In this chapter the propagating waves produced by current sources or antennas are examined; in general these waves have spherical wavefronts and direction-dependent amplitudes. Because free-space conditions are exclusively assumed throughout the chapter, the notation for the permittivity, permeability, propagation speed, and characteristic impedance of the medium can omit the subscript 0; likewise the wave number (phase shift constant) of the radiation will be written B= oVuE= w/e. 17.2 CURRENT SOURCE AND THE E AND H FIELDS ‘The vector magnetic potential A defined in Section 9.7 gives the phasor fields in the region outside of the current source as H 1 1 u o VX H=—— VX UXA=ZUKYXA. jwe joue iB in which u=3X10' m/s and = 12072. ‘The phasor A is itself given by af BG). Oe ® In (2), r is the distance between the observation point and the source current element J, dv. ‘The significance of the factor e~™ becomes clear when A is transformed to the time domain: a-[ HY, 008 (6 — Fle) 4 hat ‘4nr ‘Thus A at the observation point properly reflects conditions at the source at earlier times—the lag, for any given source element being precisely the time r/u needed for the condition to propagate to the observation point. 17.3. ELECTRIC (HERTZIAN) DIPOLE ANTENNA ‘The vector potential set up by the infinitesimal current etement of Fig. 17-1 is, by (2), ie AP) = dba, In spherical coordinates, , = cos 6a, ~sin Oag; relations (1) yield rae . 1 1d Bin as a ad 293 294 ANTENNAS [cHAP. 17 lll other components are zero. Attention will be restricted to the far field, in which terms in 1/r? or 1/r° are neglected G) It is clear that (3) represents a diverging spherical wave which at any point is traveling in the +2, direction with an amplitude that falls off as 1/r. ‘The power radiated by of the Hertzian dipole is obtained by integrating the time-averaged Poynting vector, Pog = }Re(EXH*) (Section 14.12), of the far field over the surface of a (large) sphere. Pam [ [a sin odo doa, = rf "[b Re (EoH)\P*sin dodge wy ° 17.4 ANTENNA PARAMETERS ‘The radiation resistance Ri is defined as the value of a hypothetical resistor that would dissipate ‘2 power equal to the power radiated by the antenna when fed by the same current, thus, Pag MiRea OF Riga 2Pra/ EG where &y is the peak value of the feed point current. For the Hertzian dipole, from (4), 2 ‘The pattern function F(8, ) gives the variation of the far-zone electric or magnetic field magnitude with direction. For the Hertizian dipole this reduces to F(8)=sin 8, since (E| and (H| are independent of ¢. (Q) CHAP. 17} ANTENNAS 298 ‘The radiation intensity U(8, ) is another measure of antenna performance; it is defined as the time-averaged radiated power per unit solid angle. From Fig. 17-2, Pa _|Pa dS" Pon as Ses— as i an Fig. 17-2 Because U is independent of r (by energy conservation), the far field may be used in its evaluation. For the Hertzian dipole, ue)=2 (AY sere (5) Polar plots of the pattern function and radiation intensity distribution for the Hertzian dipole are given in Fig. 17-3. (nore) @) Wo) Mig. 17-3 17-3(b), the half-power points are at 6=45° and @=135° and the half-power beam width is therefore 90°. In general, the smaller the beam width (about the direction of Ua:), the more directive the antenna. gain D(6, @) of an antenna is defined as the ratio of the radiation intensity U(8, ) to that of a hypothetical isotropic radiator that radiates the same total power Us. For the isotropic radiator, Poa be 4x UO, 9) _ 4nU(6, 0) ‘Then D(A, o)= Us Pra ‘The directivity of an antenna is the maximum value of its directive gain: Se 296 ANTENNAS, [cHap. 17 For the Hertzian dipole, (4) and (5) give © ‘The radiation efficiency of an antenna is na Pas/Po., where Pa is the time-averaged power that the antenna accepts from the feed. The (power) gain G(6, ) is defined as the efficiency times the directive gain: 4nU(6. $) _4xU(6, 6) Pa maa Pr where P, is the ohmic loss of the antenna. A lossless isotropic radiator has a power gain Go=1. At times the power gain of an antenna is expressed in decibles, where G(G, ¢)= enaD(6, $) G Fan = 101080 Can = WloBi0 1010810 G 17.5 SMALL CIRCULAR-LOOP ANTENNA Also known as the magnetic dipole, a small loop in the z=0 plane, carrying a phasor current In, , produces radiating E and H fields with characteristics similar to those of the Hertzian dipole, but with the directions of E and H interchanged. In the far zone, _ (na? le sin @ The radiation resistance of the small loop antenna is found as part of Problem 17.6: Rrsa= (20.2)(*na°Y. oy * 17.6 FINITE-LENGTH DIPOLE ‘The expression (4) for the radiated power of the Hertzian dipole contains the term (d¢/A)? which suggests that the length should be comparable to the wavelength. The open-circuited two-wire transmission line shown in Fig. 17-5(a) has currents in the conductors that are out of phase, so that the far field nearly cancels out. An efficient antenna results when the line is opened out as CHAP. 17] ANTENNAS 27 weil ue w o Fig. 17-5 shown in Fig. 17-5(b), producing current phasors A(z’) asin p(E = ) @L and r>A. Hlnse - nr F(@) Eo= nly where the pattern function is given by FO)= sin 6 ‘The antenna can also be assigned an effective length [write 1(2')= ly sin B(L/2— lz"): sin@ (7 cos 6 coy = [eye onde ‘which has the units of length and contains all the pattern information. For L up to about 1.2A the antenna patterns resemble the figure eight, becoming sharper as L approaches 1.24. In the other limit, as L42.52 ‘When a second antenna is placed adjacent to a first antenna, @ current in one will induce a voltage in the other. Consequently, a mutual impedance Z21=Vzi/hh=Ra+iXo exists in the system. For two side-by-side half-wave dipoles with very small conductor size, Ra, and Xz, vary with the separation d as shown in Fig. 17-9. CHAP. 17] ANTENNAS 299 pin = 0.0005 =F qoms Length of monopole. 172 Mig. 178. (Source: Edward C. Jordan/Keith G. Balmain, Electromagnetic Waves and Radiating Systems, ‘2nd e., © 1968, p. $48. Reprinted by permission of Prentice-Hall, Inc., Englewood Ciffs, NJ.) Ry) 06 Xa 0 Mig. 17-9. (Source: Weeks (1968), Antenna Engineering. Reprodaced by permission of McGraw-Hill, Inc.) 17-9 THE RECEIVING ANTENNA ‘An antenna in the far field of a transmitter extracts energy from what is essentially a plane wave and delivers it to a load impedance Z,._In Fig. 17-10(a) the dipole antenna lies along the z axis and the incident wave has a Poynting vector &. The open-circuit voltage is equal to the product of the effective length h,(6) and the magnitude E of the projection of E onto the plane of incidence. {For the coordinate system of Fig. 17-10(a), E= VE?+E?] Voc=h(8)E 300 ANTENNAS (CHAP. 17 mE al w wo) Mig. 17-10 ‘The pattern for the receiving antenna is identical to that of a similar transmitting antenna. The available power P, is the maximum power which the receiving antenna can deliver to a load, which occurs when Z,= Zz. From the equivalent circuit of Fig. 17-10(6), h {BYE BR, ‘The effective area A,(0) for an antenna is a hypothetical area such that when multiplied by the power density of the incident wave, E/2n, it results in the available power. PB A(O(5e) = B= BOE 4.6) =h0)(;2-) Itean be shown that the effective area is related to the directive gin by AO )_% DO, 6) 4 ‘When both a transmitting and a receiving antenna are considered, the power Pag radiated by antenna 1 and the available power P.z at the receiving atenna 2, are related by the Friss transmission formula, DilOr Or)Aer( 82. 2) Proat 4x Here, r is the separation of the two antennas. Angles @, and ¢, specify the direction of the receiving antenna as seen from the coordinate system of antenna I. Similarly 6, and @2 specify the direction of the transmitting antenna as viewed from the coordinate system of antenna 2. ss 17.10 LINEAR ARRAYS A far-field pattern with a narrow beam width and high gain can be achieved by forming an array of identical antenna elements, each with the same orientation as shown in Fig. 17-11. The pattern function of the array is equal to the pattern function of an individual element multiplied by an array factor f(x). In Problem 17.15 it is shown that, for a uniformly spaced array of N elements where d is CHAP. 17] ANTENNAS 301 Mig. 17.11 the spacing, FQ) 2, heaters ‘The angle x is the angle between the array axis and the line OP; by geometry, cosy = sin 6 cos. If the elements are progressively phased so that J, =a,e”* (n=0,1,...,N—1), Fay = E aerearteon or, defining u= a+ fid cos x, Siu = Bane ” ‘The overall pattern function will be a maximum when [fi(u)| is a maximum, which occurs for If a =O. (the individual antennas are all in phase), then u=O° implies x= £90"; ie., peak radiation occurs at right angles to the line of antennas. This is called a broadside array. On the other hand, if the phasing a=—fd isimposed, u=O implies z=0"; this is an endfire array ‘A uniform array has all antenna currents equal in magnitude. For a= 1, (7) becomes in(Nu/2) sin (u/2) ‘Thus, the main peak or lobe of the radiation pattern, centered on u=0, has “height” f(0)|= N. The two first nulls of the pattern [zeros of |f(u)l], occur at u=+2x/N. The separation of the two first nulls can be used to define the beamwidth. "Concentrating on the plane 6=90", one finds: Alu)= me (8) endfire uniform Ag BNd Vind where the approximations are for the case Nd >>. ‘The sidelobes occur approximately midway between the nulls. The ratio of the main lobe to the first sidelobe is Nsin(3xr/2N) which approaches the value 3/2 for large N. 32 ANTENNAS IcHap. 17 17.11 REFLECTORS ‘The gain of an antenna element can be enhanced by means of a reflector. Gains of from 6 to 124B can be obtained by using a half-wave dipole and a comer reflector such as that shown in Fig, 17-12(a). (A fiat sheet reflector results when y = 180°.) Conducting sect, y Tage *1 Corner ceflector we ' 0 aipule Fig. 17-12 ‘The effect of a reflector with y=180°/N (N=1,2,3,...) can be calculated by the method of images. ‘The actual reflector is replaced by 2N — 1 image dipoles, which together with the actual driven dipole, constitute an evenly spaced circular array, alternating in polarity [Fig. 17-12(6)]. Superposition of the far fields yields = ola thgy (ayessineuatie Oy © For high gain applications, the parabolic reflector driven by a source located at its focus, as shown in Fig. 17-13, is widely used. ‘The directivity of the parabolic reflector is proportional to the aperture radius a and the aperture efficiency #: ‘Qna\?, Poa (8)€ The aperture efficiency depends on a variety of design factors; a reasonable value is 55%. The half-power beam width can be estimated from the formula HPBW = 117°(4/2a). Fig. 17-13, CHAP. 17) ANTENNAS 303 Solved Problems 17.1. A center-fed dipole antenna with a z-directed current has electrical length L/A< @ Show that the current distribution may be assumed to be triangular in form. (6) Find the components of the vector magnetic potential A. (@) Since lew) 0), Ans, HT For the Hertzian dipole of Problem 17.21, determine Hi, 8, @) under the assumption |e|> ur. Consider a Hertzian dipole at the origin with angular frequency «w. Find the phases of E, and Ey relative to the phase of H, at points corresponding to (a) Br=1, (6) r= 10. Assume 0<0< 9" Ans. (a) E, lags H, by 90°, Eg logs H, by 45°; (b) E, lags H, by 90°, Ey and H, are almost in phase A redirected Hertzian dipole 1, dé and a second that is x-directed have the same angular frequency ©. If, leads 1, by 90°, show that on the y axis in the far zone the field is righthand, circularly polarized. Find the radiated power of the two Hertzian dipoles of Problem 17.3, if d<in scale factor | you change the equation of the plane, you may need to adjust the scale factor to get a better view of the plane. The equations which generate this picture may be found to the right of this screen. 320 MATHCAD SAMPLES Itis evident from this sketch that the unit vector on the side of the Plane containing the origin is produced by a negative sign. The electric field anywhere on this side ofthe plane is oa E rag (@) =| 13:6 2 les | ™ 2 wag ( tn)“ [3 | Editor's Note: The equations which generate the live picture ofthe plane are not shown here, for simplicity ‘They would, of course, be accessible in the Electronic Book companion MATHCAD SAMPLES 324 Flux Density (Schaum's Electromagnetics Solved Problem 3.11, p. 40) ‘Statement Determine the flux crossing a 1 mm by 1 mm area.on the surface of ‘a cylindrical shel a a point P given a flux density D. 2x Syston Dexy.z) | 21 y) :2 This is the given flux raters ‘4 | ™ density in cartesian coordinates. At point P, which is given in cylindrical coordinates: 53.2-deg =0.929-rad 10 P-=|0929 2 area = (1-mm)* HC =10°%cout Solution First convert the point P on the shell to Cartesian coordinates. Pyeas(P, 6 P =| Pysin(P,) p=(8 P, 2 Then, at P, 2 - cout Dp=D(PyP,,P,) Dp={-14]: 322 MATHCAD SAMPLES If the radius r ofthe cylinder is large, the 1 mm? area is essentially planar, with a directed area vector normal to its surtace (no z component) beginning at P. ° 06 s=|P, ‘sy @S=araag dS =| 08 |+mm’ o ‘The flux is given by the dot product of the flux density and the area vector: dv =Dpds. oY =-4.1¢HC ‘The negative sign indicates that flux crosses this differential surface in a direction toward the z axis, rather than outward in the direction of aS. ‘This is a good example of the type of simplifying assumptions used by engineers. Given the capabilities of Mathcad, the problem could have been solved exactly, rather than assuming the small surface to be planar. If you'd lke to see this sort of problem solved exactly, ‘examine Chapter 3. Eaitor's Note: The boldface, undeslined text in the paragraph above indicates a hyperlinked piece of text. If you ‘were working on a computer, double-clicking on the bold text with the mouse would take you to the file indicated by the text. Also, the slight difference between the answer here and on page 40 is due tothe difference in numerical accuracy used in the two calculations. MATHCAD SAMPLES 323 Method of Images (Schaum's Electromagnetics Solved Problem 7.22 and Supplementary Problem 7.34, pp. 109 and 111) Statement (a) Find the capacitance per unit length between a cylindrical conductor Of radius @ and a ground plane parallel to the conductor axis, at a distance hi from it. The conductor carries a charge distribution p. (b) Double the conductor diameter to see the effect on the capacitance, System a =25em h =60m p=108.ce! Parameters m PF =10 fara Permittvity of free space: eq =8.854-10°'2 farad Solution ‘Ausetul technique in problems of this kind s the method of images. Take the mirror image of the conductor in the ground plane, and let this image ‘conductor carry the negative of the charge distribution on the actual ‘conductor. Now suppose the ground plane is removed. It is clear that the ‘electric field of the two conductors obeys the correct boundary condition at the actual conductor, and by symmetry, has an equipotential surface where the ground plane was. Thus, the field found in this way is the field in the region between the actual conductor and the ground plane. See the figure below for clarification. @ h rt AUS ‘Approximating the actual and image charge distributions by line charges at their centers, Potential atradusadue yp (See Chapter § for to the actual conductor: weal one the potential from a line charge.) 324 MATHCAD SAMPLES To find the potential due to the image conductor, find the distance from the imaginary line charge to any point on the circumference of the real ‘conductor. Potential at point P due p to the image conductor: V image Ieey” 1-10 -volt V real + V image ‘Similarly, the potential of the image conductor is-V,, Thus, the potential difference between the two conductors is 2V,. and the potential diference at the ground plane is 2V,/2 = Vy. ‘The capacitance per unit length is then given by = Qi(LV): =9.01-2F cet Ya ‘Observe that the capacitance per unit length for the whole source-image system (and more generally, for any pair of parallel cylindrical ‘conductors with center-to-center separation 2h) is one-half the value found above, since it contains the same charge, but twice the voltage drop. (b) Ifthe radius is doubled in the original problem, asda the resulting capacitance, Cy, is PF Cox =1015-% 2x m Editor's Note: As in the previous example, differences in numerical accuracy will affect how closely these answers match those on pages 110 and 111. MATHCAD SAMPLES 325 | Magnetic Potential (Schaum's Electromagnetics Solved Problem 9.12, p. 147) Statement Given the general vector field A, in cylindrical coordinates, find the curt of Aat point P. ‘System 5-<%c0s(9) Coordinates for P: Parameters Ane =| 0 : Seeos() 1 jo jo ag a, =(0 \o, 1 Solution The curl of Ais given by: out (Each,)-2aeo, © \ag a arly la a eA 4 nve) 29 dr owl, 11S. (eaea) SMe Hos curl Purl + curl g + curt, cul =| 0 0.34 ‘Try creating different functional dependencies for A to see the effect on the curl of the vector field. Remember when changing the variables in the vector to also change the variables in the left-hand side of the definition. It A were a magnetic field, does the resulting current density agree with ‘what you'd expect to find? 326 MATHCAD SAMPLES Magnetic Work and Power (Schaum's Electromagnetics Solved Problems 10.13 and 10.14, pp. 163 and 164) | Find the work and power required to move the conductor shown in the figure one full tum in the positive direction at a rotational frequency of N revolutions per minute, it B is @ radial field. Find the work to move the conductor over some portion of the cylinder at a constant speed. If the Current direction is reversed over half the cylinder, what isthe total work. required for one revolution? ‘System = testa ok Svs Bo +3510 408 sm = 35pm 225mm L'=100-mm $1 =Orad 9's W = 10° joule sino) By4(@) =Bo| 0 o 0 The applied force to move the conductor is equal in magnitude and opposite in direction, end so itis in the positive ¢ direction. The work required to move the conductor through one full ravolution is given by the line integral over the force. MATHCAD SAMPLES 327 1. : Multiply by N revolutions per minute to arrive at the total power. 1.60410 wart rBglLN| Toind the work done for only part of a revolution, and for a magnetic field that varies with angle, find the applied force over part of a rotation. 92 (1 (4). agr Was7sq 1 Ifthe current direction changes on the side of the conductor between + and 2x, the work will be the same, because the sign of the integral changes. 269 we] [LenxBya}agre — wesrsws 92 ‘The total work fora single revolution is DW =175q0 and the power is N-2-W= 1.02110 +watt 328 MATHCAD SAMPLES Underconstrained Magnetic Circuits (Schaum's Electromagnetics Solved Problem 11.16, p. 185) Statement ‘The magnetic circuit showin below consists of nickel-iron alloy in part 1 and cast steel for part 2. In part 1, the mean length is ¢ and the area S,. In part 2, the mean length is d, and area is Sq, Find the flux densities B, and By. ‘system 2 Parameters dy = Wem dy -8em F-40amp S$) -225em* Sy ~3em? Solution Given the information, itis possible to tell what the sum of the magnetomotive forces (mmfs) are in the structure, but not the individual magnetic fields. Since there isnt an explicit function to solve simultaneously for the nonlinear flux density, this problem will require an iterative solution. The following expressions must hold true: NISFEH pd) + Hod © Hp s peters noel HH Sy eS a Sa HrSo Making a guess thal the grealer percentage of the mmf drops in the material of lower permeability (not a bad guess given the results of the analysis in Chapter 11) let's suppose: F\ The variable percent is Hy percent (i percent = 0,2 defined globally below: “1 with the final resutt MATHCAD SAMPLES 329 Read in the data points: ata = READPRN( bhlow) V1 = Ispline( data” data”) NiFe(H) Sinterp( v1 data , data? tesla Now use the function NIFe to find the flux density. iFe(H) By =1.079esta From the flux density, find the magnetic field in part 2: Bra data? = READPRN( bhhigh) v2:=Ispine (ata? data”) ‘steel(B) = interp(v2,data2> data” .p).2™P High led data Hp wea(B 9) Hy =46195g-88 used - always m check what range isin effect! Percentage drop in part: percent=20:% Hydy+ Hyd) =45.197:amp —— F=40-amp By =1.079"esla B 7 =0.809-testa Adjust the value of percent until the mmfs around the circuit and from the coil match (they aren't matched now!) 330 MATHCAD SAMPLES Itis also possible to let Mathcad do the iterating for you automatically. The until instruction causes values of the percentage to be calculated in ‘a vector until the mmf matching condition is met. First element in the vector: perc, "= 100% 0.1000 deen Number of iterations required: —_m=last(perc) NiFe| ~ Fyperc, — 1-% mona Correct percentage: percent Bperc,, , percent = Flux densities at this percentage: pape Pecent By own 4 ‘) By =1.028-testa By =0.771-tesla Editor's Note: The two data files used in this example, bhhigh and bhlow, are supplied withthe Electronic Book companion. The data are taken from the graphs shown on page 175 of this Schaum's Outline. Differences in interpolation and numerical accuracy will affect how closely this answer matches that inthe original solved problem on page 185. MATHCAD SAMPLES 331 Traveling Waves (Schaum's Electromagnetics Solved Problem 14.1, p. 226) ‘Statement Solution Attraveling wave is described by the function y(z, t, ©, B). Sketch the wave at t= 0, and at t = t,, when it has advanced one eighth of a ‘wavelength in free space. Repeat for a wave with twice the frequency and the same ty = 10¢Hz yo =10 ¥(2,t,0,8) =y gsin(Bx - wt) Permittvity of free space: eq =8.854-10''2.farad henry Permeability of free space: jig 84--10 First calculate the necessary wave parameters for both plotted frequencies. Brsefiges By =333.6107 1 m The wave number is __ twice as large for By -Befugeg Bz =6orr10 twice the frequency. ey =1.8810 +m “The wavelength is , half ts former value. dg =2® dp 294210? +m 332 MATHCAD SAMPLES ‘The waves will be plotted at times ©. at 20sec ty = fuge. 1 = Hof 0-5 Pict the wave over a single cycle of By. 2 =0.m,005-™... Br By 18002000 Attime t,, the first wave has advanced ay dy = =235.46¢m — =255.46-m 8 4 ‘The second wave has advanced by the same distance, but this, distance is now a quarter of a wavelength. es Ealtor’s Note: The ellipsis used to define the variable z indicates that it ‘a Mathcad range variable. The ‘variable z assumes all the values in the given range, in steps of 0.5%/B,, so that the traveling wave can be plotted over this range. As in previous examples, differences in numerical accuracy used will affect how closely this answer matches that in the original solved problem on page 226. ‘AC resistance, of transmission lines, 237 ‘Ait gap line. negative, 13, 184 ‘i emps, cores with, 177 “Ampere (unit) 76, 174 “Ampere turns (uit), 174 ‘Ampere's aw, 35-183, 137 for magnetic circuits, 174, 176-17 Antenna parameters, 24-296 “Antennas, 293-313, availble power of, 300 directivity of, 295-296 ffctive aes for, 300 effective length of, 297 clectrc dipole, 293-294 Tinea areays of, 300-301 monopole, 298 ‘ohmic toss of, 296 power gain of, 296 Faciation efciency of, 296 receiving, 299-300 selFimpedance of, 28-209 small creula- loop, 296, “Array factor, 300 “arrays: endlie, 301 linear of antennas, 300-301 uniform, 301 ‘Associative la, 1 “Attenuation, per-meter, 255, ‘Attenuation factor, 280-281 total, 281 Available power of antennas, 300 ‘Avogadro's umber, 86 ‘Axial components, transverse components from, 75 ‘Arial elds, 274-275. determination of, 276-271 B (see Magnetic fux density) BH curve, 174, 175 Back-voltage in inductor, 172 Beam width, half-power ‘actos interface of two dielectrics, 207 At nterface of two dielectrics, 100101 conductor-ciclectric, 8-85 Boundary reflection coeficient, 240 ‘Boundary relations, for magnetic fields, 205-206 (Capacitance, 95-113 ‘efinition of, 96-97 equivalent, 97 ‘of transmission lines, 237 Capacitors nergy sired in, 98 ‘multipleielecic, 97-98 paralle-pat, fringing of, 43-34 (Cartesian coordinate system, 3-4 ‘cual in, 138-139, INDEX CCanesian coordinate system (Cont): ‘del operator in, 29-30 ‘iferential displacement vector in, 59. ‘divergence, carl, gradient, and Lapiscian in, 314 divergence in, 47-89 through time-independent fields, 195-196 parallel, inductance of, 171 perfect, imaging in, 307 ‘Conservative fields, 60 ‘Conservative property of electrostatic field, 60 ‘Constant currents, 75 (Continuity of current, $2-£3 ‘equation of, 82 Contour, closed, 60 ‘Convection curtent, 77 Convection current density (J), 77 ‘Coordinate sytem, divergence, cul, gradient, and [Laplacian in, 314 Coordinate systems, 3-4, (See also Cartesian coordinate ‘jem, Cirularcjindrieal coordinate system; Core lengths, 174 Cores, with ar gaps, 177 Coulomb (uit), 13 ‘Coulorn forces, 13-31 ‘Coulomb's tw, 13-14, 101 Ctcal wave number, 282 (Gross product of two vectors, 2 in coordinate systems, 314 divergence of, as zero scalar, 139 of gradient ss zero vector, 139 ‘of vector fel, 137-139 Curcents) (79 ‘constant, 76 Continuity of (see Continuity of curren) {isplacement (se Displacement current) time-variable, 76 Current density. 76 92 Current elements, magnetic force on, 155-156 ‘Current filament, vector magnetic potential for, 41 Curent law, Kircihos, #2 Current sheet, 81 at boundary, 106-107 334 (Current sheet density, 81-82 CCartent source, phasor fields outside, 293 (Cutof frequency, 277-278 Cutoff wavelength, 286, (Cylindrical conductors, inductance of, 17L (see Electric ux density) D’Arsonval meter movement, 161 DC resistance, of transmission lines, 27 Decibel (unit), 229, Del operator, 89-50 Delay time, 348 oh charge (see Charge density) ‘current (see Current density) ee ‘x (ce Fax density) eee Ra Beant tin 8 ey Bae en cm eee tain Bee Sete ernie 3 re are eee pavloiteenmnate an Reece ceememet ely meeecmemaes obeaeae seas Seasons semi ee Bice. an Ea of ea oe ee ae ae Beare an See aah sina temca ay, seers es eerie see a a Leroeneehe anaes eae eo Se cm ues ses Eo oe he SRT wen cera Bamaeetasee Sanaa (see lect fed itensty) [Eflective area for antennas, 300 Eifective length of antennas, 297 Bleciic component of force, 185 INDEX lectrc current (see Curent entries) Electric dipole antennas, 293-294 Electric dipole moment, 95. Electric field intensity (E), 13-31, 114 definition of, 16-15 duc to point charges, 21 ‘otra fnction and, 63-64 tangential component of, 100 ‘definition of, 32-33 [Bectre fax density (D), 33-34 ‘ntisymmmetrical, 127-128 divergence of, 49 select field intensity and, 34-35 fixed-charge, 9 Farad (unit), 96 Faraday homopaar generator, 20 Faraday’ aw, 17 mega frm 186 {worerm form of, 196 Ferromagnetic material, 178 Field lines, 288-289, Field vector, 774 Fels: ‘axial (see Axial fields) ‘conservative, 60 clectric (se Electric fields) lecrosatic, 38-75 magnetic (see Magnetic elds) radial, 148 time-dependent, conductors in motion through, 196-197 Fields (Cont): time-independent, conductors in motion through, 195-196, transverse, 274-275 ‘veetor (see Vector fields) Finite-length dipole, 296-298 ‘First ull, 301 ‘Fixed charge electric field intensity and electric fax ensiy, 98 Fied-voltage electric field intensity and electric ux ‘density, 98-99 Fix: ‘isplacement, 32 sfetirc (se Electric ux) ‘magnesic, 140 lux density: clectric (ee Electric ux density) magnetic (fee Magnetic Aux density) Puutlines, 32-33 lux linkege, 169 Forces ‘Coulomb, 13-31 electromotive, 172 in magnetic fields, 154-168, ‘magnetomotive, 174 ‘moment of, 157 Fourier sine series, 130 Free charge, &3 Free space, Maxwell's equation solutions in, 220 Free-apace interface, dielectric, 102-103, Freespace permeability, 140 Frequency of harmonic wave, 217 Fringing of paalle-plate capacitors, 344 ‘ris transmission formuls, 300 {Gaus civegeee theorem, 50 wat Sain bias special, 35-36 Generator, Faraday homepolar, 201 Geometrical factor, 257 Gradient, 62-63 in coordinate systems. 34 ‘ur of, as zero vector rept of of potcatalfnction, 116-113 ‘Guide wavelength, 277 H (eee Magnetic field strength) Half power beam width, 295 Henry (unit), 140, 169 Hertzian dipcle antennas, 293-204 “High frequency lines, 241 Homopolar generator, Faraday, 201 1 (see Curent) Imaging in perfect conductor, 307 Impedance: “characteristic, 240 intrinsic, 218,222 ‘mutual, of antennas, 298-299 ‘selFimpedance, of antennas, 298-299 wave, 276 ‘angle of, 222 ‘normal, interface conditions at, 221-222 INDEX 335, Inductor, back voltage in, 172 afte ine charge, 17 Infinit plane charge, 18 Infinity, 2er0 relerence at, 61 Instantaneous power, 231 Interface conditions at normal incidence, 221-222 Internal inductance, 172-173 Intrinsic concentration, 87 Intrinsic impedance, 218, 222 Intrinsic semiconductors, 78, Inversesquate law of point charge, 17 Tron-core magnetics, 174 otropic radiator, 295, 4 (see Conduction current density; Convection current

    You might also like